0% found this document useful (0 votes)
338 views286 pages

General Aptitude Sheet Sloution - 2022

Uploaded by

as5626531
Copyright
© © All Rights Reserved
We take content rights seriously. If you suspect this is your content, claim it here.
Available Formats
Download as PDF, TXT or read online on Scribd
0% found this document useful (0 votes)
338 views286 pages

General Aptitude Sheet Sloution - 2022

Uploaded by

as5626531
Copyright
© © All Rights Reserved
We take content rights seriously. If you suspect this is your content, claim it here.
Available Formats
Download as PDF, TXT or read online on Scribd
You are on page 1/ 286

ANALYSIS OF GENERAL APTITUDE IN GATE PAPER

Years Marks
2017(Set-1) 15
2017(Set-2) 15
2018 15
2019 15
2020 15
2021(Set-1) 15
2021(Set-2) 15
GENERAL APTITUDE GATE SYLLABUSVERBAL

APTITUDE

Basic English grammar: tenses, articles, adjectives, prepositions, conjunctions, verb-noun


agreement, and other parts of speech

Basic vocabulary: words, idioms, and phrases in context

Reading and comprehension

Narrative sequencing

QUANTITATIVE APTITUDE

Data interpretation: data graphs (bar graphs, pie charts, and other graphs representing
data), 2- and 3-dimensional plots, maps, andtables

Numerical computation and estimation: ratios, percentages, powers, exponents and


logarithms, permutations and combinations, and series

Mensuration and geometry

Elementary statistics and probability

ANALYTICAL APTITUDE

Logic: deduction and induction

Analogy

Numerical relations and reasoning

SPATIAL APTITUDE

Transformation of shapes: translation, rotation, scaling, mirroring, assembling, and


grouping Paper folding, cutting, and patterns in 2 and 3 dimensions
QUANTITATIVE APTITUDE GATE REFERENCE BOOKS
1. Quantitative Aptitude by R.S.Agrawal.
2. A modern approaches to Verbal and Non-VerbalReasoning
3. Quantitative Aptitude by ArunSharma
4. Quantitative Aptitude book of Pearsonpublication
5. https://www.sawaal.com/aptitude-reasoning/quantitative-
aptitudearithmetic-ability-questions-and-answers.html
6. https://byjus.com/govt-exams/quantitative-aptitude/
7. https://www.careerbless.com/aptitude/qa/home.php
8. https://www.toppr.com/guides/quantitative-aptitude/
9. https://www.indiabix.com/aptitude/questions-and-answers/
10. https://www.fresherslive.com/online-test/aptitude-test/questions-and-
Answers

TABLE OF CONTENTS

S. No. CHAPTERS P. No.

1 QUANTITATIVE APTITUDE 1

2 SPATIAL APTITUDE 85

3 VERBAL APTITUDE 121

4 ANALYTICAL APTITUDE 154


QUANTITATIVE APTITUDE
Number System (Divisibility, Cyclicity, HCF and LCM)
Q1. When335isaddedto5A7,theresultis8B2.8B2isdivisibleby3.Whatissumofthe smallest
and largest possible value ofA?
(a)8 (b)13 (c)1 (d)5
Answer: D

Q2. Ifthenumber5107k3204iscompletelydivisibleby9,thenthesmallestwholenumber in
place of k willbe
(a)2 (b)6 (c)5 (d)7
Answer: C

Q3. Thegreatestamongthefollowingnumbers(3)1/3 ,(2)1/2,1,(6)1/6 is:

GENERAL APTITUDE PAGE 1


(a)(2)1/3 (b)1 (c)(6)1/6 (d)(3)1/3
Answer: d

Q4. On dividing a certain number by 9342 we get 44 as remainder. If the same number is
divided by 18, what will be the remainder?
Answer: 8

GENERAL APTITUDE PAGE 2


Q5. If the number 93215k6 is completely divisible by 11, then the smallest whole number
in place of k willbe
Answer: 7

Q6. The largest 4-digit number exactly divisible by 88is


Answer: 9944
Solution: The largest 4-digit number is 9999.
So, required answer = 9999 – 9999 mod 88 = 9999 – 55 = 9944

Q7. What least number must be subtracted from 13601, so that the resultant number is
divisible by87?
Answer: 29
Solution: 13601 mod 87 = 29

Q8. [MSQ]
5349isaddedto3957.Then7062issubtractedfromthesum.Theresultis/aredivisible by
(a)4 (b)3 (c)7 (d)11
Answer: a, b, d
Solution: 3957 + 5349 – 7062 = 2244; which is not divisible by 7.

GENERAL APTITUDE PAGE 3


Q9. If k = (5x+11y) is a prime number for positive integral values of x and y, then whatis
theleastvalueof(x+y+k)?
Answer: 36
Solution:Ifx=4andy=1thenk=31;whichisaprimenumberforleastvalueofx
andy.So,x+y+k=31+1+4=36.

Q10. On multiplying a number by 7, the product is a number each of whose digits is 3. The
smallest such numberis
Answer: 47619
Solution: The smallest such number = 333333 /7 = 47619

Q11. Thedifferenceofthesquaresoftwoconsecutiveevenintegersisdivisiblebywhichof the


followingintegers?
(a)3 (b)4 (c)6 (d)7
Answer: B
Solution:Sinceevennumberismultipleof2,hencetheirsquareisdivisibleby4.
So,theirdifferenceisalsodivisibleby4.

Q12. [MSQ]
The difference of the squares of two consecutive odd integers is always divisible by
which of the following integers?

GENERAL APTITUDE PAGE 4


(a)3 (b)4 (c)6 (d)8
Answer: b, d
Solution: Let two consecutive odd integers are 2k + 1 and 2k+3. Then the difference
oftheirsquare=(2𝑘+3)2−(2𝑘+1)2=8(𝑘+1);whichisivisibleby8.So,itisalso divisible by4.

Q13. [MSQ]
If n is a natural number, then (6𝑛2+ 6𝑛) is always divisible by
(a)3 (b)12 (c)18 (d)24
Answer: a, b
Solution:(6𝑛2+6𝑛)=6𝑛(𝑛+1)=6×2𝑘=12𝑘;𝑤𝑕𝑒𝑟𝑒𝑘𝑖𝑠𝑎𝑛𝑎𝑡𝑢𝑟𝑎𝑙𝑛𝑢𝑚𝑏𝑒𝑟
So, it is divisible by both 3 and 12.

Q14. [MSQ]
If n is a natural number, then 4(𝑛3− 𝑛) is always divisible by
(a)3 (b)6 (c)12 (d)24
Answer: a, b, c, d
Solution:4(𝑛3−𝑛)=4(𝑛−1)𝑛(𝑛+1)=4×3!)𝑘= 24𝑘;𝑤𝑕𝑒𝑟𝑒𝑘𝑖𝑠𝑎𝑛𝑎𝑡𝑢𝑟𝑎𝑙𝑛𝑢𝑚𝑏𝑒𝑟
So, it is divisible by 3, 6, 12 and 24.

Q15. Whatwillbetheremainderwhen(6767+67)isdivisibleby68?

GENERAL APTITUDE PAGE 5


Answer: 66
Solution: (6767 + 67) 𝑚𝑜𝑑 68 = ((68 − 1)67𝑚𝑜𝑑 68 + 67 𝑚𝑜𝑑 68) 𝑚𝑜𝑑 68
= ((−1)67+ 67) 𝑚𝑜𝑑 68 = 66.

Q16. [MSQ]
Whichofthefollowingnumber(s)willcompletelydivide(4949−1)?
(a)12 (b)24 (c)48 (d) None of these
Answer: a, b,c

Q17. Whatwillbetheremainderwhen(172022+1)isdividedby288?
Answer: 2
Solution: (172022 + 1) 𝑚𝑜𝑑 288 = ((17)2)1011 + 1)𝑚𝑜𝑑 288
=(288+1)1011 mod288+1mod288=2

Q18. Which one of the following is the common factor of


(4743+4343)and(4747+4347 )?
(a)90 (b)4 (c)17 (d) None of these
Answer:A

GENERAL APTITUDE PAGE 6


GENERAL APTITUDE PAGE 7
Q19. [MSQ]
Which one of the following number(s) will completely divide
(481+482+483+484 )?
(a)68 (b)20 (c)170 (d)80
Answer: a, b, c, d

GENERAL APTITUDE PAGE 8


Q20. Anumberwhendividedby7leavesaremainder4.Whenthesquareofthenumberis
dividedby7,theremainderis
Answer: 2
Solution: The remainder is 42 𝑚𝑜𝑑 7 = 2.

Q21. On dividing a number N by 6, 7, 8 if we get 4 as remainder always, then what is the


smallest valueofN? [Assume N is a naturalnumber]
Answer: 4

GENERAL APTITUDE PAGE 9


Q22. On dividing a number N by 6, 7, 8 if we get 4 as remainder always, then what is
thesecond smallest valueofN? [Assume N is a naturalnumber]
Answer: 172

Q23. On dividing a number N by 6, 7, 8 if we get 5, 6 & 7 as remainder respectively, then


what is the smallest valueofN? [Assume N is a naturalnumber]
Answer: 167

GENERAL APTITUDE PAGE 10


Q24. Whatisthe3rdsmallestnaturalnumber,whichafteradding8isdivisibleby10,11and 12?
Answer: 1972
Solution: Required number =3× LCM (10, 11, 12) – 8 = 3 × 660 – 8 = 1972

Q25. A number was divided successively in order by 4, 5, 6. The remainders were


respectively2,3and4.Thenumberis
Answer: 214
Solution: Required number = (((6 × 1 + 4) × 5) + 3) × 4 + 2 = 214.

Q26. What is the largest number N, such that the number 210, 315 and 525 are divisible by
N?
Answer: 105

GENERAL APTITUDE PAGE 11


Q27. WhatisthelargestnumberN,suchthatwhenthenumber45,101and157aredivisible by N
then we get remainder 3 in eachcase?
Answer: 14

Q28. What is the largest number N, such that when the number 151, 193 and 424 are
divisiblebyNthenwegetsameremainderineachcase?
Answer: 21

GENERAL APTITUDE PAGE 12


Q29. WhatisthesecondlargestnumberN,suchthatwhenthenumber155,205and305are
divisiblebyNthenwegetsameremainderineachcase?
Answer: 25

Q30. What is the third largest number N, such that when the number 93, 128 and 199 are
divisible by N then we get remainder 3, 2 and 1respectively?
Answer: 6

GENERAL APTITUDE PAGE 13


Q31. Whatisthevalueof3510𝑚𝑜𝑑8?
Answer: 1

Q32. Whatistheunitdigitof1111+1212+1313+1414+1515+166?
Answer: 7

GENERAL APTITUDE PAGE 14


Q33. Whatisthevalueof(𝑋mod5);ifX=(3254)236×(178)17×(327)323 ?
Answer: 4

GENERAL APTITUDE PAGE 15


Q34. Amanwroteallthenaturalnumbersstartingfrom1inaseries.Whatwillbethe100 thdigit of
thenumber?
Answer: 5

GENERAL APTITUDE PAGE 16


Q35. Whatisthevalueof(310.2−33 .3+1)𝑚𝑜𝑑(26)?
Answer: 4

Q36. [MSQ]
LetXbeanaturalnumbersuchthatX=465374−235374,thenwhichofthefollowing statement
is/aretrue?
(a) X mod 35 =0
(b) X mod 23 =0
(c) (𝑋 + 1)2mod 5 = 1
(d) 𝑋 mod 161 = 0
Answer: a, b, c, d

Q37. The number 7105 + 8105 is always divisible by


(a)3 (b)5 (c)15 (d) All
oftheseAnswer:d

GENERAL APTITUDE PAGE 17


Q38. How many divisorsof16800? _
Answer:72

Q39. Whatwillbethesumofalldivisorsof16800?

Answer: 62496

Q40. Let H.C.F and L.C.M of 4/14, 6/21 and 8/42 are X and Y respectively, then
X/Y is equal to
(a)8 (b)24 (c)24 (d)1
7 147 72
Answer: D

Q41. Whatwillbetheunitdigitof2626(263)37(45)63?

Answer:0
Q42. Whatwillbetheunitdigitof2121+2222+2323+2424?

Answer:8
Q43. LetN=23253159whatistheremainderwhenNisdividedby17?
Answer: 4
Q44. The value of 9(999) mod 8is _

Answer: 1
Q45. Two numbers differ by 5. If their product is 336, then the sum of the two numbers
would be:
(a)41 (b)43 (c)37 (d)39
Answer: C
Q46. The multiplication of two numbers is 9375 and the quotient, when the greater is
divided by the smaller is 15. The sum of the numbers is:
(a) 600 (b)125 (c)275 (d)400
Answer: D
Q47. The product of the two prime numbers is 493. What will the L.C.M of these two
numbers?
(a)493 (b)17 (c)29 (d)Can‟t bedetermined
Answer: A
Q48. What is the remainder when: 1661 + 1551 + 1441 + 1331 + 1221 is divided by 20?
(a)25 (b)5 (c)11 (d)1
Answer: B
Q49. The sum of two numbers is 45 and their product is 500. The HCF of the numbers is:
(a)5 (b)9 (c)10 (d)15

GENERAL APTITUDE PAGE 18


Answer: A
Q50. Which of the following is a pair of co-primes?
(a)23,69 (b)37,111 (c) 14,9 (d) 51,34
Answer: C
Q51. The sum of two numbers is 216 and their H.C.F is 27. The numbers are:
(a)81,135 (b)27,189 (c)154,162 (d)both a &b
Answer: D

Q52. Which of the following number which when divides 1265 leaves quotient 84 and
remainder 5?
(a)15 (b)18 (c)23 (d)61
Answer: A

Q53. A number gives a remainder 5 when it is divided by 8. What will be the remainder
when the square of the same number is divided by 4?

(a)2 (b)0 (c)1 (d) can‟t be determined


Answer:C

Q54. A hundred digit number is formed by writing first 54 natural numbers one after the
other as 123456…………….5354. What will be the remainder when this number is
divided by 8?
(a)1 (b)2 (c)4 (d)0
Answer: 2

Q55. Ifn=1+x,wherexistheproductoffourconsecutivepositiveintegers,thenwhichof
thefollowingstatementsis/aretrue?

(i) nisodd (ii) √𝑛isprime (iii) n is a perfectsquare


(a)ionly (b) i,iionly (c) i,iiionly (d) All are true
Answer:d

Q56. Iftheunitdigitintheproduct(459×46×28*×484)is2,thedigitinplaceof*is:
Answer: 7

Q57. A person divided 100 rupees between his friends, if there will be 5 more friends than
each will get 1 rupee less. How many friend he had?
(a)20 (b)25 (c)30 (d)35

GENERAL APTITUDE PAGE 19


Answer: A

Percentage
Q58. 50% of a% of b is 75% of b% of C. Which of the following is C?
(a)1.5a (b)0.667a (c)0.5a (d)1.25a
Answer: B
Solution: 50% of a% of b = 75% of b% of C
0.5𝑎𝑏 0.75𝑏𝐶
⟹ =
100 100
⟹ 0.50𝑎 = 0.75𝐶
0.50
⟹𝐶= 𝑎 = 0.667𝑎
0.75
Q59. x% of y + y % of x =?
(a) 3%ofxy (b) 2%ofxy (c) 5%ofxy (d) None ofthese
Answer: B
Q60. If 90% of A = 30% of B and B= x% of A, then the value of x is
(a)800 (b)300 (c)700 (d) None ofthese
Answer: B
GENERAL APTITUDE PAGE 20
Q61. The price of cooking oil has increased by 15%. The percentage of reduction that a family
shouldeffectintheuseofcookingoil,soasnottoincreasetheexpenditureonthisaccount is
2 1 1
(a)15 % (b) 13 % (c) 17 % (d)None ofthese
23 23 23

GENERAL APTITUDE PAGE 21


Answer: B
Q62. A number is increased by 20% and then decreased by 20%, the final value of the number
(a)Does not change
(b) Decreases by2%
(c) Increases by4%
(d) Decreases by4%
Answer: D
Q63. The population of a town is decreased by 20% and 25% in two successive years. What
percent of population is decreased after two years?
(a)50% (b)40% (c)60% (d)None ofthese
Answer: B
Q64. When the price of an article is reduced by 15% the sales increases by 35%. The
percentage change in the total amount of receipts is
(a)Decrease (b)Increase (c)Decrease (d) None ofthese
Answer: B
Q65. If the side of a square is increased by 30%, its area is increased by
(a)49% (b)69% (c) 79% (d)None ofthese
Answer: B
Q66. For a rectangle, the length and breadth are increased by 10% and 20%, respectively.
The percentage increases in area is
(a)24% (b)48% (c)32% (d) None of these
Answer:C
Q67. Water tax is increased by 20% but its consumption is decreased by 20%. The increase
or decrease in the expenditure is
(a)4%decrease (b)4% increase (c) 8% decrease (d) 8 % increase
Answer:A

Q68. The population of a town increases by 12% during first year and decreases by 10 %
during second year. If the present population is 50400, what it was 2 years ago?
(a)40000 (b)35000 (c)50000 (d) None ofthese
Answer: C

GENERAL APTITUDE PAGE 22


Q69. The salary of an employee is increased twice successively by 25% and 20%
respectively. What are the two successive deductions that should be made to bring
back his salary to the original level?
(a) 20%,25% (b) 25%,50% (c) 20%,50% (d) 22%,23%
3 3

Answer: C

Q70. The salaries of A and B together is equal to Rs 10,000. A spends 80% of his salary and B
spends 70% of his salary. If now, their saving arethe same, what is A′s salary?
(a)6000 (b)5000 (c)4000 (d)7000
Answer: A
Q71. A man save 40% of his monthly salary. On account of an increase in price, he has to
increasehismonthlyexpenditureby40%,andisabletosaveRs40permonth.What is his
monthly salary?
(a)Rs1000 (b)Rs500 (c)Rs200 (d) Rs250
Answer: D

Q72. Duetofallinmanpower,productioninafactorydecreasesto40%.Bywhatpercentageshould
the working hours be increased to restore the original level ofproduction?
(a)662% (b)461% (c)25% (d)40 %
3 3

Answer: A
Q73. Afterspending20%onmachinery,10%onbuliding,9%onrawmaterialsand7%on other,
Chandra had a balance of Rs. 2700, then the money with him initially was.
(a)Rs5000 (b)Rs6000 (c)Rs7000 (d) Rs8000
Answer: A

Q74. The population of a city decreases by 10% at the end of first year and increases by 10
at the end of second year and again decreases by 10% at the end of the thridyear .If
thepopulationofthecityattheendofthirdyearis4,455,thenwhatwasthepopulation
ofthecityatthebeginningofthefirstyers?
(a)5000 (b)4500 (c)4950 (d)1000

GENERAL APTITUDE PAGE 23


Answer: A
Q75. Twonumbersarelessthanathirdnumberby40%and46%respectively.Bywhatpercentis
the second number less than the first, is
(a)5% (b)15% (c)20% (d)10%
Answer: D
Q76. IfRaju‟ssalaryisincreasedby40%,bywhatpercentageshoulditbebroughtdownto bring it
back to the originallevel?
(a)191% (b)162% (c)284% (d) 231%
3 3 7 7

Answer: C
Q77. Theproductionofricewentupfrom2000milliontonnesin1998to2300milliontonnes
in1999.Whatisthepercentageincreaseintheproductionofriceform1998to1999? (a)20%
(b)15% (c)25% (d)12½%
Answer: B
Q78. In an examination 35% of students passed in maths, 48% passes in science and 23%
passed in both the subject . What is the percentage of the student who failed in both
the subject?
(a)50% (b)40% (c)60% (d)53%
Answer: B
Q79. In an election contested by two candidates. The candidates who gets 43% is rejected
by a margin of 2856 votes. What is the total number of votes polled given that all the
votes polled were valid votes?
(a)20,400 (b)20,820 (c)16,500 (d)40,800
Answer: A
Q80. In an election a candidate secures 37% of the total votes polled. The only
othercandidate defeated him by 520 votes. If all the votes polled are valid, the
number ofvotes polledwas
(a)1500 (b)4000 (c)2000 (d)4400
Answer: C

GENERAL APTITUDE PAGE 24


PROFIT AND LOSS

GENERAL APTITUDE PAGE 25


Q81. AnarticleisboughtforRs.675andsoldforRs.900.Findthegainpercent?
(a)302% (b)311% (c)331% (d) 331%
3 3 3 6

Answer: C
Q82. A person buys a horse for 15 pounds. After one year, he sells it for 20 pounds. After
one year, again he buys the same horse at 30 pounds and sells it for 40 pounds. What
is the overall profit percent for that person over both the transactions?
(a)30.33% (b)33.33% (c)35.33% (d)40.33%
Answer: B
Q83. Atradersells85mofclothforRs.8,925attheprofitofRs.15/mofcloth.Whatisthe cost price
of 1 m ofcloth?
(a)Rs.84 (b)Rs.90 (c) Rs.96 (d) Rs.102
Answer: B
Q84. By selling an article at Rs. 800, a shopkeeper makes a profit of 25%. At what price
should he sell the article so as to make a loss of 25%?
(a)Rs.460 (b)Rs.480 (c)Rs. 500 (d) Rs.520
Answer: B

Q85. Ankit bought 160 shirts at the rate of Rs. 225/shirt. The transport expenditure wasRs.
1,400. He paid an octroi at the rate of Rs. 1.75/shirt and labour charges were Rs. 320.
Whatshouldbethesellingpriceof1shirt,ifhewantsaprofitof20%?
(a)Rs.260 (b)Rs.275 (c) Rs. 280 (d) Rs.285
Answer: D
Q86. A man sold two houses for Rs. 8 lakhs each. On the one, he gained 6% and on the
other, he lost 6%. What percent is the effect of the sale on the whole?
(a) 0.36%loss (b) 0.36%gain (c) 36%loss (d) 36 %gain
Answer: A
Q87. If two articles are bought for the same price (i.e., the cost prices are equal) and one is
sold at a profit of 30% and the second is sold at a profit of 20%, then the overall
percentage of profitis
Answer: 25

GENERAL APTITUDE PAGE 26


Q88. If two articles are bought for the same price (i.e., the cost prices are equal) and one is
sold at a profit of 20% and the second is sold at a loss of 30%, then the overall
percentage of profit/loss is
(a) 25%profit (b)10%loss (c)5%loss (d) none ofthese
Answer: C
Q89. A shopkeeper sells an article for Rs 960 and gains 20%. At what price should he sell
the article to gain 40%?
(a)Rs.800 (b)Rs.1200 (c) Rs.1120 (d) Rs.1100
Answer: C
Q90. Ashopkeeperprofessestosellthearticlesatadiscountof25%,butmarksthelistprice
ofeacharticle40%abovethecostprice.Whatishisgain(in%)oneacharticle?
(a)15% (b)10% (c) 5% (d)3.75%
Answer: C
Q91. Amerchantgives25%discountonanarticleandstillgain20%.Howmuchpercentage above
the cost price is the articlemarked?
(a)62.5% (b)15% (c)37.5% (d)60%
Answer: D
Q92. 400 tomatoes were bought at Rs 125 per hundred and were sold at a total profit of Rs
100.Thesellingpriceofadozentomatoesis
(a)Rs20 (b)Rs18 (c)Rs25 (d) none of
theseAnswer:B
Q93. By selling a book for Rs 150, a shop owner lost 1/16 of what it costs. What is its cost
price?
(a)Rs170 (b)Rs180 (c)Rs150 (d) Rs160
Answer: D
Q94. By selling 44 meters of ribbon I gained the cost price of 11 meters of ribbon. My
percentage of gainis
1
(a)33 % (b)25% (c)30% (d) None ofthese
3

Answer:B

GENERAL APTITUDE PAGE 27


Q95. What will be the percentage of profit after selling an article at a certain price, if there
is a loss of 10% when the article is sold at half that price?
(a)20% (b)80% (c)50% (d) None ofthese
Answer: B
Q96. Amanbuys200orangesforRs10.Howmanyorangesarupeecanhesellsoastogain 25%?
Answer: 16

Q97. Thecostpriceof30lemonsisthesameassellingpriceof24lemons.Whatispercentage
ofgain/loss?
(a) 25%profit (b)20%profit (c) 25%loss (d) 20%
profitAnswer:A

Q98. Thecostpriceofacertainnumberofarticlesisthesameasthesellingpriceof1/3𝑟𝑑
of the same number of articles. What is the gain/loss percent?
(a)66.67%gain (b) 200%gain (c)33%gain (d) 66.67%loss
Answer:B

Q99. A dealer weighs only 950 gm per kg. What is his error percentage?
5
(a) 5 % (b)5% (c)15% (d) None ofthese
19

Answer: B
Q100. A dealer professes to sell his goods at cost price and uses an 880 gm weight instead of
a kg. What is his percentage of gain? (Approximately)
(a)13.13% (b) 13.33% (c)13.36% (d) 13.64%
Answer: D
Q101. Atradergainsthecostof2kgofoilbyselling10kgofoil.Whatishisgainpercentage?
2 2
(a)20% (b)25% (c)18 % (d) 11 %
3 3

Answer: A
Q102. Aftertwosuccessivediscountsof10%eachareallowed,anarticleissoldat Rs
2,430. What is the discountallowed?
(a)Rs510.30 (b)Rs570 (c)Rs486 (d) Rs461.70

GENERAL APTITUDE PAGE 28


Answer: B
Q103. If,aftersuccessivediscountsof5%,10%and25%,acustomerhastopayRs1282.50for
anarticle,thenwhatistheapproximatelistpriceofthearticle?
(a)Rs2200 (b)Rs2250 (c)Rs2000 (d) Rs2400
Answer: C
Q104. If 10%, 20%, 30% are three successive discounts offered for an article, then what is %
effective discount/overall discount?
(a) 50% (b) 15.6% (c)50.4% (d)49.6%
Answer: D

Q105. If30%and50%aretwosuccessivediscounts,thenitisequivalenttoasinglediscount
percentage , which is equalto
(a)80% (b)40% (c)65% (d)60%
Answer: C

Q106. The listed price of a bag is Rs 160. A merchant bought it for Rs 122.40. He got two
successive discounts of which one is 10%. What is other discount?
(a)10% (b)15% (c)20% (d)25%
Answer: B
Q107. The sale price of a „T‟ shirt, listed for Rs 400, after successive discounts 10% and 5%, is
(a)Rs340 (b)Rs341 (c)Rs342 (d) Rs343
Answer: C

Q108. XsellsYanarticleataprofitof20%.YsellsittoZatalossof20%.IfZpaidRs960,
thenwhatisthesellingpriceofX?
Answer: 1000

Q109. Ifthepriceofanappleisreducedby20%,apersoncanbuy5applesmoreforRs100,
whatisthereducedcostofeachapple?
(a)Rs5 (b)Rs4 (c)Rs1 (d)
Rs4.80Answer:B

GENERAL APTITUDE PAGE 29


Q110. Atrader‟spricesyieldagainof25%.Heweighsonly950gmforeachkilogram.What is his
percentage gain?(Approximately)
(a)23.5% (b)30.25% (c)28.6% (d)31.6%
Answer: B

CI and SI
Q111. Whatistheinterest(inRs)onasumofRs80,000atarateof20%peranumforthree years under
simpleinterest?
Answer: 48000
Solution: Simple interest = 80,000 × 0.2 × 3 = 48000 Rs.

Q112. A sum triples in 16 years at simple interest. What is the rate of interest per anum?
(a)10% (b)5.33% (c)12.5% (d) none ofthese
Answer: C
Solution:Rateofinterest=200×100=12.5%
16×100

Q113. AsumofRs60,000amountstoRs80,000infouryearsatsimpleinterest.Whatwillthe sum


amount (in Rs) to in 11years?
Answer: 115000

Q114. AcertainsumofmoneyamountstoRs1,250intwoyearsandRs1500fouryears.What is the


rate of simpleinterest?
(a)6.25% (b)12.5% (c)18% (d)25%
Answer: B

GENERAL APTITUDE PAGE 30


Q115. AsumofRs1,200amountstoRs1,800in10yearsundersimpleinterest.Whatshould
betherateofinterestsothatthesamesumamountsto1800ineightyearsundersimple
interest?
(a)6.25% (b)12.5% (c)18% (d)25%
Answer: A
Q116. HowmuchwillRs12,000amounttoin3yearsatarateof25%perannum
Compoundinterest?
(a)Rs25000 (b)Rs21000 (c) Rs 23437.5 (d)none
Answer: C

Q117. A sum of money invested at compound triples itself in four years. In how many years
will it becomes 27 times itself at the same rate of compound interest?
(a)1.33years (b)8years (c)12years (d) 20
yearsAnswer:C

Q118. AsumofmoneyamountstoRs36,000inthreeyearsandRs48000infouryearsat
compoundinterest,compoundedannuallyineachcase.Whatistherateofinterestper
annum?
(a)66.67% (b)33.33% (c)20% (d)25%
Answer: B
Q119. There is 60% increase in an amount in 6 years at simple interest. What will be the
compound interest of Rs. 12,000 after 3 years at the same rate?
(a)Rs.2160 (b)Rs.3120 (c) Rs.3972 (d) Rs.6240
Answer: C
Solution:

GENERAL APTITUDE PAGE 31


Q120. Rs.1,00,000wasborrowedfor3years.Whatwillbethecompoundinterestiftherate of
interest for first year is 3% per annum; second year is 4% per annum and for third
year is 5% per annumrespectively?
(a)Rs120000 (b)Rs112476 (c)Rs150000 (d) none of these
Answer:B

Q121. Asumofmoneybecomes16timesofitselfin2yearsifcompoundedhalf-yearly.How
muchtimeitwilltaketobecome27timesifcompoundedyearly.
(a)3years (b)4years (c) 5years (d) 6years
Answer: A
Solution:

Q122. ThedifferenceofS.IandC.IonanamountofRs.30000for2yearsisRs.147.Whatis the rate


ofInterest?
(a)8% (b)10% (c)9% (d) 7%

GENERAL APTITUDE PAGE 32


Answer: D
Solution:

Q123. If the ratio of difference of S.I and C.I. for two years and 3 years is 4:13. Find the rate
of interest.
(a)20% (b)25% (c)30% (d) 40%
Answer: B
Solution:
The ratio of difference of S.I and C.I. for two years and 3 years is
𝑟 2 𝑟 2 𝑟
𝑃( ):𝑃( )(3+ )=100:(300+𝑟)100100 100
But in question, it is given that 4 : 13.
Therefore,100:(300+𝑟)=4∶13
So, r = 25.

Q124. The difference between SI and CI for 2 years at the rate of 20% per annum is Rs 16.
What is the principal value?
(a)Rs.200 (b)Rs.400 (c)Rs. 500 (d) Rs.800
Answer: B
Solution: Using the formula: Difference = P (R/100) 2
16 = P[20/100]2
On Solving, P = Rs 400

Q125. Whatisthedifferenceofcompoundinterestsaccruedat20%perannumonRs10,000 for one


year when interest is compounded half-yearly and when it is compounded quarterly?
[Round off upto four decimaldigits]
Answer: 55.0625
Solution:
Requireddifference=10000×[{(1.05)4−1}−{(1.1)2−1}]=55.0625

GENERAL APTITUDE PAGE 33


Q126. A certain sum amounts to Rs 6,300 in two years and to Rs 7,875 in three years nine
months at simple interest. What is the rate of interest per annum?
(a)10% (b)15% (c)18% (d)20%
Answer: D

Q127. ThedifferencebetweenthesimpleinterestandcompoundinterestonasumofRs.5,00,000
fortwoyearsisRs2450.Whatistherateofinterest,ifitiscompoundedannually
(a)6% (b)7% (c)8% (d)10%
Answer: B

Q128. AsumofRs.7,500investedatcompoundinterestdoublesinfiveyears.After20years
itbecomes.
(a)Rs1,50,000 (b)Rs60,000 (c)Rs 1,20,000 (d)Rs 90000
Answer: C

Q129. In how many years will a sum of money , when invested at 25% p.a. Compound
interest compounded annually, becomes 1.5625 times (approximately one and half
times) itself ?
(a)1 (b)2 (c)3 (d)4
Answer: B

Q130. A sum amounts to Rs 8640 in three years at 20 % per anum compound interest. What
was the value of the sum?
(a)Rs7500 (b)Rs5000 (c)Rs 6000 (d)Rs4500
Answer: B

Q131. Findthedifferenceinthecompoundinterestat40%p.awhenitiscompoundonRs
20,000foroneyear,whentheinterestiscompoundedhalfyearlyandquarterly.
(a)Rs284 (b)Rs364 (c)Rs 482 (d) Rs436
Answer: C

GENERAL APTITUDE PAGE 34


Q132. AsumofRsPamountstoRs9P/4infiveyearsatsimpleinterest.Iftherateofinterest is
decreases by ten percentage points how much interest will the same sum yield in the
sametime?
(a)Rs3𝑃 (b)Rs3𝑃 (c)Rs5𝑃 (d) Rs𝑃
8 4 8 8

Answer: B

Q133. ThedifferenceincompoundinterestaccruedonRs20,000foroneyearat6%p/awhen
interestiscompoundedyearlyandwhenitiscompoundedhalfyearlyis
(a)Rs32 (b)Rs15 (c)Rs 18 (d)Rs21
Answer: C

Q134. The difference between the simple interests received from two different banks on a
sumofRs60,000fortwoyearsisRs1,500.Whatisthedifferenceoftheinterestrates of the
two banks in percentagepoints?
(a)1.25 (b)1.5 (c)0.5 (d)0.75
Answer: A

Q135. If the difference of the simple interest and the compound interest on a certain sum at
15% p.a. for two years is Rs 360. Find the sum.
(a)Rs2,400 (b)Rs20,000 (c) Rs 16,000 (d) Rs4.800
Answer: C

Q136. Ifthesimpleinterestonasumofmoneyat10%p.aforfouryearsisRs2,400thenthe
compound interest on the same sum for the same period at the same rate, when
interest is compounded annually,is
(a)Rs 2847.60 (b)7248.80 (c)2784.60 (d)Rs4284.70
Answer: C

Q137. A sum of money doubles itself at a certain rate of compound interest, compounded
annually in six years. In how many years (approximately) will it triple itself at the
same rate of compound interest? (log 3: log2 = 1.5850)

GENERAL APTITUDE PAGE 35


1 1
(a)11 (b)9 (c)9 (d)12
3 2

Answer: B

Q138. Ifthesimpleinterestonasumofmoneyat5%peranumforfouryearsisRs6,000then what


will be the corresponding compound interest on the same sum at the same rate of
interest for 3years?
(a)Rs4,278.25 (b)Rs 4,728.75 (c)Rs 3,975.75 (d) Rs4,4,75.75
Answer: B

Q139. Hari gave a loan of Rs 20 to Ram and recover at a rate of Rs 3.50 for eight months,
commencingfromtheendof1stmonth.Whatistheeffectiverateofsimpleinterestper
annum?
(a)60% (b)80% (c)40% (d)90%
Answer: A
Q140. A sum of Rs 800 amount to Rs 1,040 in four years at simple interest. By how many
percentage points should the rate of interest per annum be increased so that Rs 1,200
would amount to rs 2,400 in 8 years?
(a)4 (b)8 (c)5 (d)None of these.
Answer:C

Q141. The effective annual rate of interest corresponding to a nominal rate of 6% per annum
payable half-yearly is:
(a)6.06% (b)6.07% (c)6.08% (d)6.09%
Answer: D
Solution:Effectiverateofinterest=100×[(1+0.03)2−1]= 6.09%

Q142. JohninvestsRs.5,000inatermdepositscheme.Theschemeoffersaninterestrateof
6%perannum,compoundedquarterly.Whatistheeffectiverateofinterest?
(a)0.06% (b)61.3% (c)6.13% (d)6%
Answer: C
Solution: We know that,
Principalamount=P=Rs.5,000
Actual rate of interest = i = 6% p.a. = 0.06 p.a. = 0.015 per quarter

GENERAL APTITUDE PAGE 36


Number of conversion periods = n = 4
(since we are calculating for one year and compounding happens every quarter)
The effective rate of interest (E) is,
E = (1 + i)n– 1 = (1 + 0.015)4 – 1 = 0.0613 or 6.13%.

RATIO, PROPORTION AND VARIATION


Q143. Three numbers are in the ratio of 2 : P : 5. The third number is greater than the first
oneby21.FindthevalueofP,ifthesumofthenumbersis70?
Answer: 3
Q144. If a : b = 2 : 3; c : b = 5 : 4; d : c = 2 : 7 and d : e = 2 : 3, then a : b : c : d : e is equivalent
to
(a)56:84:105:30:45 (b)56:84:100:36:45
(c)60:80:100:30:50 (d) 84:56:30:105:40

Q145. The expenditure and savings of a person are in the ratio of 4:1. If his savings are
increased by 25% of his income, then what is the new ratio of his expenditureand
savings?
(a)8:5 (b)7:5 (c)16:9 (d)11:9

Q146. A bag containing twelve goggles is dropped from the first floor of a building. Which
of the following cannot be the ratio of the number of goggles broken to unbroken?
(a)1:5 (b)5:7 (c)2:1 (d)4:3

Q147. The ratio of marks obtained by Ram and Das is 4 : 3. If Ram and Das get 210 marks
together and the average of their percentage is 70, find their individual marks and the
total marks for which the exam was conducted.
(a)160,120,200
(b)180,135,200
(c)150,120,200
(d)120,90,150

GENERAL APTITUDE PAGE 37


Q148. The ratio of ages of a son and a father is 1:4.If the present age of the son is 15 years,
then after how many years their ages will be in the ratio of 4:13?
(a)20 (b)10 (c)5 (d)15

Q149. Find the three numbers in the ratio 6:8:3 such that the difference of the square of the
first and the third number is 108.
(a)6,8,3 (b)18,24,9 (c)9,12,4 (d)12,16,6

Q150. The ratio of A‟s salary to B‟s was 4 : 5. A‟s salary is increased by 10% and B‟s by 20%,
what is the ratio of their salaries now?
(a)14:11 (b)15:14 (c)11:15 (d) None ofthese
Answer: C
Solution: Present ratio = 4 : 5. Increase of 10% and 20%.
New ratio of salaries will be 4 × 1.1 : 5 × 1.2 = 11 : 15.

Q151. 300 coins consists of 1 rupee, 50 paise and 25 paise coins, their values being in the ratio
of 10 : 4 : 3. Find the number of coins of each type.
(a) 100,80,120 (b) 80,90,100 (c) 100,100,80 (d) 60, 80,100
Answer: A
Solution:
Value of rupee coins = Rs10 i.e. 10 coins.
Value of 50 p coins = Rs 4 i.e. 8 coins.
Value of 25 p coins = Rs. 3 i.e. 12 coins.
∴ Ratio of coins = 10 : 8 : 12 ⇒ 5 : 4 : 6.
∴Numberofrupeecoins=5/15×300=100.
Numberof50Pcoins=4/15×300=80and
Numberof25Pcoins=6/15×300=120

Q152. Ifxvariesinverselyas(y2–1)andisequalto24wheny=10,thenthevalueofxwhen
y=5willbe?
Answer: 99

Q153. Rs.590 is divided amongst A, B, C so that 5 times A's share, six times B's share and
eight times C's share are all equal. What is C's share?

GENERAL APTITUDE PAGE 38


(a)Rs.130 (b)Rs.150 (c)Rs.170 (d)Rs.175
Answer: B
Solution:
A+B+C = 590
5A = 6B = 8C = x
A:B:C = 1/5 : 1/6 : 1/8 = 24 : 20 : 15
15/59 * 590 = Rs.150

Q154. The salary earned by a man for three months in the ratio of 2 : 4 : 5. If the difference
between the product of salaries of the first two months to that of the last two months
(in Rs) is 4,80, 00, 000, then what is the salary of the man for the second month?

Answer: 8000
Q155. Theforceofattractionbetweentwobodiesvariesinverslyasthesquareofthedistance
betweenthem.Whenthedistanceis20feet,theforceofattractionis18N.Whatisthe
forceofattraction,whenthedistanceis30feet?
(a)8N (b)20N (c)40N (d) 80N
Answer: A
Q156. The length of the edges of two cubes are in the ratio 5 : 4. What is the ratio of their
diagonal?

(a) 25: 16 (b) 16:25 (c)√2∶√3 (d) 5 :4


Answer: D
Q157. The ratio of surface area of two spheres is 64 : 81. What is the ratio of their voumes?
(a) 2048:2407 (b) 8:9 (c) 512:729 (d) 256 :243
Answer: C
PARTNERSHIP
Q158. A, B and C invested Rs.6300, Rs.4200 and Rs.10500 respectively, in a partnership
business. What is the share of A in profit of Rs.12100 after a year?
(a)Rs.3630 (b)Rs.2840 (c)Rs.3200 (d)Rs.5600
Answer: A

GENERAL APTITUDE PAGE 39


Q159. Amit, Chintu, and Deepak decided to start a new business by investing Rs. 1,20,000,
Rs.1,35,000,andRs1,50,000respectively.IfthetotalyearlyprofitofthebusinessisRs.
56,700thanwhatwillbethesumofshareofAmitandDeepak?
Answer: 37800
Q160. A and B invest in a business in the ratio 3: 2. If 5% of the total profit goes to charity
and A's share is Rs. 855, the total profit is :
(a)500 (b)1000 (c)1500 (d)2000
Answer: C
Solution: Let the total profit be Rs. 100.
After paying to charity, A's share = (95*3/5) = Rs. 57.
If A's share is Rs. 57, total profit = Rs. 100.
If A's share is Rs. 855, total profit = (100/57*855) = 1500.

Q161. A, B, C started a business with their investments in the ratio 1:3:5. After 4 months, A
invested the same amount as before and B as well as C withdrew half of their
investments. The ratio of their profits at the end of the year is :
(a) 1 : 2:3 (b) 3 : 4:15 (c) 3 : 5:10 (d) 5 : 6 :10
Answer: D
Solution: Let their initial investments be x, 3x and 5x respectively. Then,
A:B:C = (x*4+2x*8) : (3x*4+(3x/2)*8) : (5x*4+(5x/2)*8)
20x : 24x : 40x = 5 : 6 : 10

Q162. A, B, C rent a pasture. A puts 10 oxen for 7 months, B puts 12 oxen for 5 months and
Cputs15oxenfor3monthsforgrazing.IftherentofthepastureisRs.175,howmuch must C
pay as his share ofrent?
(a)Rs.45 (b)Rs.50 (c)Rs.55 (d) Rs.60
Answer: A
Solution:
A:B:C = (10×7):(12×5):(15×3)
= 70 :60 :45
= 14 :12 :9

C's rent = Rs.(175×9/35)


=Rs. 45

GENERAL APTITUDE PAGE 40


Q163. A began a business with Rs. 85,000. He was joined afterwards by B with Ks. 42,500.
For how much period does B join, if the profits at the end of the year are divided in
the ratio of 3: 1?
(a)4months (b)5months (c)6months (d)8months
Answer: D
Solution: Suppose B joined for x months .
Then, (85000 * 12)/(42500 * x) = 3.
or x = (85000 * 12) /(42500 * 3) = 8.
So, B joined for 8 months.

Q164. A, B and C enter into a partnership and their shares are in the ratio 1/2 : 1/3 : 1/4.
After2months,Awithdrawshalfofhiscapitalandafter10months,aprofitofRs.378 is
divided among them. What is B'sshare?
(a)144 (b)169 (c)225 (d)339
Answer: A
Solution:
Ratio of initial investments = 1/2 : 1/3 : 1/4 = 6 : 4 : 3.

Lettheirinitialinvestmentsbe6x,2xand3xrespectively.

A : B : C = (6x * 2 + 3x * 10) : (4x * 12) : (3x * 12) = 42 : 48 : 36 = 7 : 8 : 6.

B's share = Rs. (378*8/21) = Rs. 144.

Q165. A invested Rs x for 5 months and Rs y for the remaining period. B invested Rs y for
thefirst8monthsandRsxfortheremainingperiod.Ifattheendoftheyeartheyshare
theprofitsequally,thenwhatisthevaluex–y?
(a)x (b)2x (c)–y (d) 0
Answer:D

Q166. In a business, Ajay puts in Rs 10,000 and Karishma puts in Rs 20,000. Ajay received
20% of the profit for managing the business and the rest of the profit is divided in the
ratio of their capitals. What is Ajay‟s share of the profit in a profit of Rs 6000?

Answer: 2800

GENERAL APTITUDE PAGE 41


Q167. A and B enter into partnership with capital as 7:9. At the end of 8 months, A
withdraws. If they receive the profits in the ratio of 8:9, find how long B's capitalwas
used?
(a)6months (b)8months (c)10months (d) 7
monthsAnswer:D

Q168. A, B, C invested their capitals in the ratio of 4 : 3 : 2. At the end of the year they shared
the profits in the ratio 8 : 9 : 8. Who among A, B and C stayed in the business for the
longest period?
(a)A (b)B (c)C (d) Can‟tsay

Q169. Kamal started business with Rs 6000 and was later joined by Bharat with Rs 8, 000.
After how many months did Bharat join, if the profit at the end of the year were
divided in the ratio of 3: 2?
(a)6 (b)8 (c)9 (d)10
Q170. A and B are partners in a business. If A stayed in the business for one year and
contributed1/3rdofthecapitalwhileBreceived3/5thoftheprofit,thenforhowlong
B‟smoneywasinthebusiness?
(a) 8months
(b) 6months
(c) 10months
(d) 9months

AVERAGES
Q171. Thereare50,60and40studentsinthreesectionsA,BandCofaclassrespectively.If the
average marks of the students in sections A, b and C are 65, 70 and 75 Respectively,
then find the approximate average marks of all three sections put together.
(a)70 (b)80 (c)60 (d)50
Answer: A

GENERAL APTITUDE PAGE 42


Q172. Inaclassof20students,theaverageweightofthe10studentsis„a‟kgandthatoflast
10studentsisbkg.Whatistheaverageweightofallthestudents?

(a) (a +b)kg (b)(10𝑎+20𝑏)kg (c)(𝑎+𝑏)kg (d) Can‟tsay


𝑎+𝑏 2

Answer: C
Q173. Theaverageageofagirlandherparentsis28years.Iftheratioofthepresentagesof
thefather,themotherandthegirlis7:6:1,thenwhatwillbegirl‟sageafter2years?
(a)7years (b)9years (c)6years (d) 8
yearsAnswer:D

Q174. The average of five numbers is 39.20 and the average of three of these numbers is 41.
What is the average of other two numbers?
(a) 35.5 (b)36.5 (c)37.5 (d)38.5
Answer: B
Q175. In a class of 60 students, the average weight of 30 students is x kg and that of the
remaining is y kg. What is the average weight of the class?
𝑥𝑦
(a)(x +y)kg (b)30kg (c)(𝑥+𝑦)kg (d)( )kg
2 2

Answer: C

Q176. Theaverageageoftheeightboys‟increasesbyoneyear,ifaboyof12yearsofageis
replacedbyanewboy.Whatistheageofthenewboy?
(a)9years (b)15 years (c)10years (d) 20
yearsAnswer:D

Q177. The average weight of A, B and C is 80 kg. If D joins, the average becomes 82 kg. If
anotherpersonE,whoseweightis3kglessthanD,replacesA,thentheaverageofB,
C,DandEbecomes78kg.WhatistheweightofA?
(a)101kg (b)92kg (c)85kg (d) 97kg
Answer: A
Q178. The average weight of 8 person's increases by 2.5 kg when a new person comes in
place of one of them weighing 65 kg. What might be the weight of the new person?
(a)76kg (b)76.5kg (c)85kg (d) Datainadequate

GENERAL APTITUDE PAGE 43


Q179. The average monthly income of P and Q is Rs. 5050. The average monthly income of
Q and R is Rs. 6250 and the average monthly income of P and R is Rs. 5200. The
monthly income of P is:
(a)3500 (b)4000 (c)4050 (d)5000
Answer: B
Solution:
Let P, Q and R represent their respective monthly incomes. Then, wehave:
P+Q=(5050x2)=10100 ......... (i)
Q + R = (6250 x 2)=12500 ........ (ii)
P + R = (5200 x 2)=10400 ........ (iii)
Adding (i), (ii) and (iii), we get: 2(P+Q + R) = 33000
or P + Q+ R=16500 .......(iv)
Subtracting(ii)from(iv),wegetP=4000. P's
monthly income =Rs. 4000.

Q180. A car owner buys petrol at Rs.7.50, Rs. 8 and Rs. 8.50 per litre for three successive
years. What approximately is the average cost per litre of petrol if he spends Rs.4000
eachyear?
(a)Rs.7.98 (b)Rs.8 (c)Rs. 8.50 (d) Rs.9
Answer: A
Solution:

Q181. The average mark of 600 students in an examination is 42. Among them, the average
of the top 150 students is 76, while that of the last 250 students is 28. What are the
average marks of the remaining 200 students?

GENERAL APTITUDE PAGE 44


(a)43 (b)34 (c)51 (d)22.66
Answer: B

Q182. Alibraryhasanaverageof510visitorsonSundaysand240onotherdays.Theaverage number


of visitors per day in a month of 30 days beginning with a Sundayis:
(a)250 (b)276 (c)280 (d)285
Answer: D
Solution:

Q183. Among P, Q, R, S and T, P weighs 2 kg less than Q, Q weighs 2 kg more than R orS,
individually. But T weighs 6 kg more than Q. the average weight of these five will be
equaltotheweightofwhoamongthefollowing?
(a)P (b)Q (c)T (d)
RAnswer:B

Q184. Fouryearsago,theaverageageofAandBwas20years.IftodayaverageageofA,B
andCis25years,whatwillbeageofCafter7years?
(a)32 years (b)34years (c)36years (d)38years
Answer: B
Solution:TotalageofAandBfouryearsago=20∗2=40years Present
total age of A and B would be = 40 + 4∗2 = 48 years
PresenttotalageofA,BandCis=25∗3=75years
∴ Present age of C would be = 75 - 48 = 27 years
Age of C after 7 years = 27 + 7 = 34 years

Q185. Inanexam,theaveragemarkfor80studentsofClassVis35.Theaverageofmarks
insectionAoftheclassis55,whiletheaverageofmarksinsectionBis30.Findthe

GENERAL APTITUDE PAGE 45


number of students in Class VB.
(a)45 (b)50 (c)64 (d)70
Answer: C
Solution: Total students i.e. Section A + Section B = 80
Section A = 80 - Section B = 80 - B
∴ 80 x 35 = B x 30 + (80 - B) x 55
2800 = 30B + 4400 - 55B
1600 = 25 B
B = 64
∴ B = 64 = Number of students in Class VB.
Q186. In Arun's opinion, his weight is greater than 65 kg but least than 72 kg. His brother
does not agree with Arun and he thinks that Arun's weight is greater than 60 kg but
lessthan70kg.Hismother'sviewisthathisweightcannotbegreaterthan68kg.Ifall of them
are correct in their estimation, what is the average of different probable weights of
Arun?
(a)55.5kg (b)66.5kg (c)77.5kg (d) 88.5kg
Answer: B
Solution: Let Arun's weight be X kg. According to Arun, 65 < X < 72.
According to Arun's brother, 60 < X < 70.
According to Arun's mother, X < 68.
Thevaluessatisfyingalltheaboveconditionsare66and67.
Requiredaverage=(66+67)/2=66.5kg

Q187. If the average weight of a group of 30 people is 50 kg and another group of 40people
has an average weight of 60 kg, then the weighted average of two groups (i.e. , the
averageweightofthecombinedgroupof70people)is:
(a)55kg (b) 55.71kg (c)56kg (d) 53.5kg

MIXTURE AND ALLEGATIONS


Q188. 12 kg of sugar costing Rs 6 per kg and 6 kg of sugar costing Rs 15 per kg are mixed.
What is the price per kg of the resultant mixture?
(a)Rs10 (b)Rs11 (c)Rs8 (d) Rs9
Answer: D

GENERAL APTITUDE PAGE 46


Q189. In what ratio must 35% milk solution be mixed with pure milk to get a resultant
solution of 56% milk?
(a) 35: 44 (b) 44: 21 (c) 8:3 (d) 56 : 9
Answer: B

Q190. How many kilogram of wheat at rs 9.30 per kg must be mixed with 16 kg of wheat at
Rs 13.80 per kg such that the mixture when sold at Rs 12.43 per kg gives a profit of
10%?
(a)24 (b)15 (c)20 (d)25
Answer: C

Q191. The ratio of copper and zinc in two alloys is 2: 3 and 3: 5 respectively. What is the
proportion of copper and zinc in an alloy obtained by mixing equal quantities ofthese
twoalloys?
(a) 21: 59 (b) 41: 39 (c) 31:49 (d) 69 : 11
Answer: C
Q192. In what ratio must two solutions containing beer and water in the ratios of
3 : 2 and 7 : 3 respectively be mixed such that the resultant solution has beer and water
in the ratio 2 :1?
(a) 1: 2 (b) 2: 3 (c) 3:2 (d) 2 :1
Answer: A

Q193. Theratioofthevolumesofmilkandwaterinasolutionis4:3.Byadding28litresof
water,theratiogetsreversedandthecanbecomesfull.Whatisthecapacityofthecan, inlitres?
(a)120 (b)112 (c)84 (d)76
Answer: B

Q194. A vessel contains a mixture of spirit and water with 40% spirit in it. When 10 litres of
pure spirit is added, the vessel becomes full and contains 50% spirit. What is the
capacity of vessel (in litres)?

GENERAL APTITUDE PAGE 47


(a)70litres (b)60litres (c)50litres (d) 65 litres
Answer:B

Q195. Abarrelhas60litresofpurespirit.Sixlitresofspiritisremovedandreplacedwiththe same


amount of water. The process is repeated two more times. What is the amount of
spirit (in litres) in the resultantsolution?
Answer: 43.74

Q196. A large tank has 49 litres of pure milk. Seven litres of milk is removed and replaced
with water. How many more times should this process repeated such that the resultant
solution has 30 6litres of milk?
7

(a)1 (b)2 (c)3 (d)4


Answer: B
Q197. The milk and water in two vessels A and B are in the ratio 4: 3 and 2: 3respectively. In what
ratio, the liquids in both the vessels be mixed to obtain a new mixture in vessel C containing
half milk and half water?
(a) 5: 7 (b) 3: 4 (c) 4:3 (d) 7 :5
Answer: D

Q198. The cost of Type 1 rice is Rs. 15 per kg and Type 2 rice is Rs. 20 per kg If both Type 1 and
Type 2 are mixed in the ratio of 2 : 3.then the price per kg of the mixed variety rice is :
(a)Rs.18 (b)Rs.18.50 (c)Rs.19 (d) Rs. 19.50
Answer: A

Q199. A container contains 40 litres of milk From this container 4 litres of milk was taken out

and replaced by water. This process was repeated further two times. How much milk is

now contained by the container?

(a)26.34litres (b)27.36litres (c)28litres (d) 29.16litres


Solution:
Amount of milk left after 3 operations
4 9 9 9
=[40(1− )3)]𝑙𝑖𝑡𝑒𝑟𝑠=(40∗ ∗ ∗ )=29.16litres.
40 10 10 10
GENERAL APTITUDE PAGE 48
Q200. One quality of wheat at Rs. 9.30 per kg is mixed with another quality at a certain rate in the
ratio 8: 7. If the mixture so formed be worth Rs. 10 per kg, what is the rate per kg of the
second quality of wheat?

(a) Rs. 10.30 (b) Rs. 10.60 (c)Rs.10.80 (d) Rs.11


Solution:

Q201. Two vessels A and B contain milk and water mixed in the ratio of 8:5 and 5:2

respectively.Theratioinwhichthesetwomixturesbemixedtogetanewmixture
3
containing 69 % milk is :
13

(a)2:7 (b)3:5 (c)5:2 (d) 5:7

Q202. 4/7 of 42 litres of a mixture of phenol and water is water. When k litres of water is added to

this mixture, the ratio of phenol and water becomes 2 : 3. What is the value of x?

(a)12 (b)6 (c)3 (d)2

Answer: C

Q203. Tea worth of Rs. 135/kg & Rs. 126/kg are mixed with a third variety in theratio 1 : 1 : 2. If

the mixture is worth Rs. 153 per kg, the price of the third variety per kg will be?

(a) Rs. 169.50 (b) Rs.1700 (c) Rs. 175.50 (d) Rs.180

GENERAL APTITUDE PAGE 47


Answer: C

Q204. A merchant has 1000 kg of sugar, part of which he sells at 8% profit and the rest at 18%

profit. He gains 14% on the whole. The quantity sold at 18% profit is: (a)400kg

(b)560kg (c)600kg (d)640kg

Answer: C

Solution:

TIME AND WORK

Q205. Eight men can do a work in 12 days. If there were two men less, in how many days
can they complete one-fourth of the originalwork?
Answer: 4
Q206. Acandoapieceofworkin20daysandBcandoitin30days.Inhowmanydayswill they
complete the worktogether?
Answer: 12
Q207. Forty eight clerks can clear 360 files in 15 days. How many clerks are needed toclear
480filesin12days?
Answer: 80
Q208. TheratioofthecapacitytodoaworkofAandBis3:2.Iftheytogethercancomplete
a work in 18 days, then how long does A take to complete the work alone?

GENERAL APTITUDE PAGE 48


(a)45 (b)30 (c)24 (d)40
Answer: B
Q209. 12 men and 8 women can do a piece of work in 16 days, while 13 men and 4 women
can do it in 18 days. How long will 13 men and 12 women take to finish the work?
6 1 3 1
(a)12 days (b) 12 days (c)12 days (d) 13 days
13 2 5 11

Answer: D
Q210. A is four times as efficient as B while C is twice as efficient as B. if A can do a job in 6
days. How long will A, B and C together take to complete thework?
(a)31days (b) 33days (c) 33days (d)none ofthese
2 7 4

Q211. 4 men or 8 women or 12 boys can do a certain work in 98 days. How many days will
3 men, 5 women and 8 boys together take to do the work?
(a)40 (b)44 (c)46 (d) 48
Answer: D
Q212. Agroupof10boyscandoajobin12days.anothergroupof10girlscandothework in 6 days.
How many days are required to complete the work if both the group work together?
(a)8 (b)6 (c)4 (d) none of these

Q213. Two pipes A and B can fill a tank in 30 minutes and 40 minutes respectively. C can
empty the tank in 60 minutes. When the tank is empty all the three pipes are opened
for 12 minutes and then pipe B is closed. How many minutes, form the beginning,
does it take for the tank to be filled?
(a)42 (b)72 (c)24 (d)30

Q214. Xis50%moreefficientthanYindoingawork.ifthedifferenceoftheirearningsupon
completingtheworktogetherisRs160,howmuchdidthey receiveinall?
(a)Rs210 (b)Rs240 (c)Rs800 (d) Rs400

GENERAL APTITUDE PAGE 49


Q215. AnemptytankcanbefilledbypipeAifitisopenfrom 3:00pmto7:00pm.Butdue
toleakage,only3/4thofthetankisfilledby7:00pm.Iftheleakageisstopped,atwhat time will
the tank befull?
(a) 8 :00am (b) 8 :00pm (c) 7 :00am (d) 8 : 10am
Q216. P, Q and R together can complete a work in 18 days. They work together for 2 days
and then P leaves. If their rate of work is equal, in how many days will the remaining
work be completed?
(a)36 (b)30 (c)24 (d)21
Answer: C
Q217. A can do a piece of work in 16 days. He works for four days and then B joins, who
alone can complete the same work in 32 days. If A gets rs 1200, then how much does
B earn?
(a)Rs800 (b)Rs600 (c)Rs 500 (d) 400

Q218. SunilandPradeepcancompleteaworkin5daysand15daysrespectively.Theyboth work


for one day and then Sunil leaves. In how many days is the remaining work
completed byPradeep?
Answer:11
Q219. A can do a work in 9 days, if B is 50% more efficient than A, then in how many days
can B do the samework?
(a)3days (b)13.5days (c)4.5days (d) 6
daysAnswer:D
Solution:

Q220. PratibhaisthriceasefficientasSoniaandisthereforeabletofinishapieceofworkin 60 days


less than Sonia. Pratibha and Sonia can individually complete the work
respectivelyin
(a) 40 days, 120 days

GENERAL APTITUDE PAGE 50


(b) 30 days, 60days
(c) 60 days, 90days
(d) 30 days,
90daysAnswer: D
Solution:

Q221. A can build a wall measuring 20 ft × 3 ft × 15 ft in 6 days. B can build a wall measuring
30ft x 4 ft x 10 ft in 5 days. What is the ratio of work done by A and B in on day?
(a) 8: 5 (b) 5: 8 (c) 6:7 (d) 7 :5
Q222. Acandigapitworkingeighthoursaday,Bcanfillitworkingtenhoursaday.Ifboth
ofthemstarttheworkatthesametime,afterhowmanydays,willtherebeafullydug pit
available for B tofill?
(a)50 (b)25 (c)20 (d)40

Q223. Tenmencandiga25feetdeepwellineightdays.Aftertwodaysafifthoftheportion dug is


filled due to rain. How many more days will the same men take to dig it completely?
(a)1/5 (b)1/4 (c)2/5 (d)3/4

Q224. If 10 men and 4 boys working together can do twice as much work per hour as a man
and 4 boys together, then the ratio of the work done by a man to that of a boy for a
given time is
(a)2:1 (b) 1:2 (c) 3:4 (d) None ofthese

Q225. A is 50% more efficient than B and both are able to complete a work together in 15
days. How long would A, working alone, take to complete the work?
(a)20days (b)22days (c)24days (d) None ofthese

GENERAL APTITUDE PAGE 51


Q226. Acandoaworkin20daysandBalonecandoitin10days.Bothworkedtogetherand
werepaidRs1,800.TheshareofBis
(a)Rs1,200 (b)Rs1,500 (c)Rs900 (d) None ofthese

Q227. 16 men and 12 boys can complete a work in 26 days. 13 men can complete the same
work in 48 days. In how many days will 12 men and 6 boys complete half the work?
(a)30 (b)34 (c)39 (d)19.5

Q228. Two pipes can fill an empty tub in 5 minutes and 7 minutes respectively. An outlet
pipe can empty full tub in 3 minutes. If the tub is empty and all the pipes are opened
simultaneously when the tub will be full?
(a)11/4 hours (b)11/2 hours(c)2hours(d)13/4 hours
Q229. A cistern has a leak which would empty it in 3 hours. A tap is turned on which lets in
6litresofwaterperminute.Thetankisinitiallyfullwhenbothtapsareturnedonand
itisemptiedin6hours.Whatisthecapacityofthetank?
(a)2100litres (b)2160litres (c)2500litres (d) None ofthese

Q230. Two pipes can fill a tank in 4 hours and 8 hours respectively whereas a third pipe can
empty it in 6 hours .If the tank is initially empty and the three pipes are opened at
10:00 am, 11:00 am and 12:00 noon respectively, when will the tank be full?
(a)2:00P.M (b)1:30P.M (c)1:48P.M (d)None ofthese

TIME AND DISTANCE (AVERAGE SPEED, TRAIN, BOAT STREAM, RACES AND CIRCULAR TRACK)

GENERAL APTITUDE PAGE 52


Q231. A train covers a certain distance in one hour and 45 minutes and another train covers
twice the distance travelled by first train in 2 hours and 45 minutes. What is the ratio
of their speeds?
(a) 7: 11 (b) 11: 14 (c) 1:2 (d) 1 :7

Q232. A motorcyclist covered two-thirds of a total Journey at his usual speed. He covered
the remaining distance at three-fourth of his usual speed. As result, he arrived 30
minutes later than the time he would have taken at usual speed. If the total journey
was 180 km, then what was his usual speed?
(a)40kmph (b)36kmph (c)30kmph (d)32kmph

Q233. A man covers the distance between A and B in 30 minutes and returns from B to A

in 60 minutes the distance between A and B is 40 km, then what is average speed?
1 2
(a)26kmph (b)53 kmph (c)66 kmph (d) 70kmph
3 3

Q234. A person in a car moving at a speed of 64 km/h covers a certain distance in 9 hours.
How much time does another person travelling by a van at a speed of 72 km/h take
to cover the samedistance?
(a)6hours (b)12hours (c)8hours (d)7hours

Q235. A motorist covers a distance of 50 km in 45 minutes by moving at a speed of x kmph


for the first 10 minutes, x/2 kmph for the next 20 minutes and 2x kmph for the rest of
the journey. Then x is equal to
(a)50 (b)60 (c)75 (d)90

Q236. A bus covers four successive distances of 45 km each at speeds of 10 kmph, 20 kmph,
30 kmph and 60 kmph respectively. What is the average speed of the bus?

(a)20kmph (b)30kmph (c)40kmph (d)50kmph

Q237. An escalator is moving up at the rate of 12 ft/sec. If a man runs on it to reach the top
at the rate of 9 ft/sec. Given that when the escalator is not moving the man hasto

GENERAL APTITUDE PAGE 53


climb 42 feet to reach the top, what time does he take to reach the top?
(a)14seconds (b)9seconds (c)2seconds (d) 4seconds

Q238. Acyclisthastocover150kminthreehours.Ifhecoversone-thirdofthedistancein half of


the total time, what should his average speed for the rest of the journey be? (a
)50kmph (b)66.66kmph (c)75kmph (d) 30kmph

Q239. It takes 7 hours to cover a certain distance at a speed of 75 kmph by a car. At what
speed must one travel to cover the same distance in 2 hours less time?
(a)105kmph (b)534kmph (c)90kmph (d) 84 1kmph
7 7

Q240. In a journey, one –third of the distance is covered at 30 kmph and the remaining at
40 kmph. What is the average speed for the entire journey?
(a)35kmph (b)37.5kmph (c)36kmph (d)33.5kmph

Q241. A bus travels without any stoppages, form Ujjain to Indore. With an average speed of
60kmphwhilewithstoppagesitsaveragespeedforthejourneyisreducedto55kmph. Of
many minutes does the bus stop on an average perhour?
(a)20minutes (b)25 minutes (c)11minutes (d) none ofthese
Q242. A train moves a certain distance in 8 hours. If it had moved 5 kmph faster, it would
have covered the distance in 6 hours 40 minutes. What is the original speed of the
train?
(a)25kmph (b)40kmph (c)45kmph (d) 35kmph

Q243. The speed of a train is 15 m/s. The time it takes to cross a telephone pole is 10
seconds. What is the length of the train?
(a)100m (b)1.5km (c)150m (d)200m

Q244. A train 300 m long and moving at a speed of 25 m/s can cross a man standing on a
platform in

GENERAL APTITUDE PAGE 54


(a)60seconds (b)75seconds (c)24 seconds (d) 12seconds

Q245. Two trains start at the same time from two stations A and B towards each other. They
arrive at B and A respectively in 5 hours and 20 hours after they passed each other. If
the speed of the train that started from A is 56 kmph, then what is the speed of the
second train?

(a)26kmph (b)28kmph (c)24kmph (d)30kmph

Q246. A train leaves Hyderabad for Bangalore at 6:00 a.m. at a speed of 72 kmph. Another
trainleavesBangaloreforHyderabadat8:00a.m.ataspeedof54kmph.Howfarfrom
Hyderabad will the two trainsmeet?
(a) 144km
(b) 216km
(c) 314km
(d) Cannot bedetermined

Q247. Two trains are travelling in the same direction at 40 kmph and 60 kmph. The faster
train overtakes the slower train in 3 minutes. If the length of the faster train is 400 m,
then what is length of the slower train?
(a)200m (b)100m (c)150m (d) 600m
Q248. A train is 300 m long and moves at a speed of 50 km per hour. In what time does it
cross a person walking in the opposite direction at 4 km per hour?
(a)18sec (b)24sec (c)20sec (d) 15sec

Q249. A train which is 225 m long travels at 45 km/h. In what time does it cross a man
running at 18 km /hin the same direction?

(a)120seconds (b)30seconds (c)25seconds (d) 15second

Q250. Amanstandsonarailwaybridgewhichis300mlonghefindsthatatraincrosseshim
10 seconds and the bridge in 25 seconds. What is the length of the train ,

GENERAL APTITUDE PAGE 55


(a)100M (b)200M (c)240M (d)360M

Q251. Two trains started from a station, at the same time in the same direction, at speeds
of 72 kmph and 60 kmph. What will be the distance between the two trains after 7
hours?
(a)42km (b)50.4km (c)63km (d)84km

Q252. In what time do two trains of lengths 380 m and 470 m, travelling in opposite
directions at respective speeds 55 km/h, and 35 km/h, cross each other?

(a)34Seconds (b)5minutes (c)36seconds (d)6minutes

Q253. Two trains are running on parallel lines in the same direction at a speed of 50 kmph
and 30 kmph respectively. The faster train crosses a man in the slower train in 18
seconds. The length of the faster train is train is.
(a)170m (b)100m (c)98m (d)85m

Q254. A man can row at 12 kmph in still water. He finds that it takes him thrice as much
time to row up the river as it takes to row down the river. What is the speed of the
current?
(a)6kmph (b)24kmph (c)48kmph (d)18kmph

Q255. A man can row a certain distance against the stream in six hours. However, he would
take two hours less to cover the same distance with the current. If the speed of the
current is 2 kmph, then what is the speed of the man in still water?
(a)10kmph (b)12kmph (c)14kmph (d) 8kmph

Q256. A man can row three quarters of a kilometer against the current in 11 ¼ minutes
and return in 7 ½ minutes. Find the speed of the man in still water.
(a)8kmph (b)7kmph (c)6kmph, (d) 5kmph
Q257. A man can row downstream at 12 kmph and upstream at 8 kmph. Find the ratio of
the speed of the current to the speed of the man in still water.

GENERAL APTITUDE PAGE 56


(a)1 : 5 (b) 5: 4 (c) 25:16 (d) 16 : 25

Q258. Apersoncanrowat9kmperhourinstillwater.Hefindsthatittakeshimtwiceas
muchtimetorowupstreamascomparedtodownstream.Whatisthespeedofthe current?
(a)3kmph (b) 4½ kmph (c)6kmph (d) 3 ½ kmph

Q259. Amancanrowat6kmphinstillwaterandariverisflowingat4kmph.Howlong
willthemantaketogotoaplace1kmdownstreamandreturn?
(a)36minutes (b)24minutes (c)12minutes (d) 18minutes

Q260. A person can row a distance of one km against the - stream in ten minutes and
along the stream in four minutes. What is the speed of the stream?
(a)3km/h (b)9km/h (c)5.6km/h (d)4.5km/h

Q261. Apersoncanrow18kmdownstreamand27kmupstreaminthreehoursand18
kmupstreamand27kmdownstreamintwohours42minutes.Whatisthetime
takenbythepersontorow25kmupstream?
(a) 17hours (b) 11hours (c)2hours (d)2 3hours
10 2 10

Q262. It takes eight hours for a 600 km journey, if 120 km is done by train and the rest by
car.Ittakes20minutesmore,if200kmisdonebytrainandtherestbycar.Theratio
ofthespeedofthetraintothatofthecarsis:
(a) 3: 4 (b) 3: 2 (c) 2:3 (d) 4 :3
Q263. Apersoncanrowataspeedof18kmphinstillwater.Forthesamedistance,thetime taken
by him to row upstream is thrice the time taken by him to row downstream.
Whatisthespeedofthestream?
(a)9km/h (b)6km/h (c)8½km/h (d)3km/h
Q264. Speed of sound is 330 m/s in air while it is 440 m/s in a liquid. the total time

GENERAL APTITUDE PAGE 57


takenbysoundtocoveradistanceof6.6kminairand4.4kminthatliquidis (a)20
seconds (b)2½seconds (c)30seconds (d)50seconds

Q265. Inakilometerrace,AbeatsBby20meter.Ina2kilometersrace,AbeatsBby (a)10m


(b)20m (c)40m (d) 25m

Q266. InaKmraceAbeatsBby50m.Ina3kmraceAbeatsBby
(a)150m (b)300m (c)400m (d)600m

Q267. InAkilometerrace,ifAgivesBastartof100meters,bothofthemreachthefinish
lineatthesametime,ifA`sspeedis20m/s,themwhatisB`sspeed?
(a)16m/s (b)17m/s (c)18m/s (d)19m/s

Q268. Inakilometerrace.AcangiveBastartof40mandBcangiveCastartof25m(In
thesamerace)howmanymetresstartcanAgiveC.
(a)46m (b)56m (c)66m (d)64m

Q269. Inakilometerrace,AbeatsBby50metresandBbeatsCby75metres.Byhowmany
metresdoesAbeatCinthesamerace?

(a)135m (b)142.5m (c)121.25m (d) 125m


Q270. A policeman chases a thief at a speed of 27 km per hour and catches him after one
minute36seconds.Ifthedistancebetweenthepolicemanandthethiefwas240meter
initially. What was the speed of thethief?
(a)12km/h (b)9km/h (c)18km/h (d) 21km/h

Data for next two question:


A, B, and C start running simultaneously on a circular track of length 120 meters from the
same point with the speeds 2m/s, 3m/s and 6m/s respectively. A and C are running in anti-
clockwise direction and B is running in a clockwise direction.

Q271. After how much time from the start all three will meet again at the starting point?
(a)24seconds (b)30seconds (c) 60 seconds (d) 120 seconds
Answer:D
Solution: For meeting at the starting point:

GENERAL APTITUDE PAGE 58


Time taken to complete one round by A = 120/2 = 60 seconds.
Similarly,forBandC,itwouldbe40and20secondsrespectively. Time
taken by all three to meet at the startingpoint
= LCM (60, 40, 20) = 120 seconds.
Q272. After how much time from the start all three will meet for the first time anywhere on
the track?
(a)24seconds (b)30seconds (c) 60 seconds (d) 120 seconds
Answer:D
Solution: For the first meeting:

TimetakentomeetforthefirsttimebyAandB= 𝐿𝑒𝑛𝑔𝑡𝑕𝑜𝑓𝑡𝑕𝑒𝑡𝑟𝑎𝑐𝑘
𝑅𝑒𝑙𝑎𝑡𝑖𝑣𝑒 𝑠𝑝𝑒𝑒𝑑 𝑜𝑓 𝐴 𝑎𝑛𝑑 𝐵

= 120/5 = 24 seconds.

TimetakentomeetforthefirsttimebyAandC= 𝐿𝑒𝑛𝑔𝑡𝑕𝑜𝑓𝑡𝑕𝑒𝑡𝑟𝑎𝑐𝑘 =120/4=30


𝑅𝑒𝑙𝑎𝑡𝑖𝑣𝑒 𝑠𝑝𝑒𝑒𝑑 𝑜𝑓 𝐴 𝑎𝑛𝑑 𝐶
seconds.
A and B will meet in every integral multiple of 24 and A and C will meet in every
integralmultipleof30.Allthreetomeettogetherforthefirsttime,thetimeshouldbe
multipleofboth24and30,i.e.,LCM(24,30)=120seconds.
Hence they will meet for the first time at the starting point only.
Q273. In a circular race of 2400m, A and B start from the same point and at the same time
withspeedsof27km/hr.and45km/hr.Findwhenwilltheymeetagainforthe first time on
the track when they are running in the same direction and Opposite direction,
respectively?
(a) 120 sec, 480sec
(b) 480 sec, 120sec
(c) 133.33 sec, 33.33sec
(d) 33.33 sec, 133.33 sec

Answer: B
Solution:

GENERAL APTITUDE PAGE 59


Q274. In a circular race of 1200m, A and B start from the same point and at the same time
with speeds of 27kmph and 45 kmph. Find when will they meet again for the first
time on the trace when they are running in the same direction and Opposite
direction?
(a) 240 sec, 60Sec
(b) 200 sec, 60Sec
(c) 200 sec, 80Sec
(d) None of
theseAnswer:A
Solution:

GENERAL APTITUDE PAGE 60


Q275. Inacircularraceof1200mlength,AandBstartwithspeedsof18kmphand27kmph starting
at the same time from the same point. When will they meet for the first time at the
starting point when running in the same direction and oppositedirection?

(a) 240 sec, 160sec

(b) 480 sec, 480sec

(c) 240 sec, 240sec

(d) 160 sec, 480sec

Answer: A
Solution:

GENERAL APTITUDE PAGE 61


Q276. A, B and C run around a circular track of length 1200 m with respective speeds 9, 18,
27 kmph. If they started at the same time from the same point and run in the same
direction when will they meet for the firsttime?

(a)360sec (b)480sec (c)240sec (d)None

Answer: B
Solution:

Q277. P, Q, R run around a circular track 1200 m long with speed of 9, 18, 27 kmph.If they

GENERAL APTITUDE PAGE 62


start at the same point and at the same time in the same direction, when will they meet

again at the starting point?

(a)360sec (b)480sec (c)240sec (d)None

Answer: B
Solution: L = 1200 m
Speed of P(p) = 9 x 5/18 = 2.5 m/sec

Speed of Q (q) = 18 x 5/18 = 5 m/sec


SpeedofR(r)=27x5/18=7.5m/sec
TheywillmeetforthefirsttimeatatimewhichistheLCMofL/p,L/a,L/r L/p =
1200 /2.5 = 480
L/q = 1200 /5 = 240
L/r = 1200 /7.5 = 160
LCM of 480, 240, 160 is 480 Sec. So they meet after 8 min.

GEOMETRY AND MENSURATION

Q278. Consider the following shape:

Whatistheperimeterof∆ABCincm?
Answer: 29

Q279. In the given figure ABCD is a Parallelogram then the find out of the value of x is?

GENERAL APTITUDE PAGE 63


(a)25 (b)60 (c)75 (d)45

Q280. In the given Figure ∠PQA = 20° and ∠APQ = 120° then what is the value of
∠PAQ?

Answer: 40°

Q281. For a triangle ABC, D and E are two points on AB and AC such that AD = 1/4 AB,
AE =1/4 AC. If BC =12 CM, then DE is
(a)5cm (b)4cm (c)3cm (d)6cm
Answer:C
Solution:

GENERAL APTITUDE PAGE 64


Q282. In a triangle , if orthocenter , circumcenter, incenter and centroid coincide ,then the
triangle must be
(a)obtuseangled (b)isosceles (c)equilateral (d)right-angled
Answer: C
Solution:

Q283. In triangle ABC, a line is drawn from the vertex A to a point D on BC. If BC =9 cm
and DC = 3 cm, then what is the ratio of the areas of triangle ABD and triangle ADC
respectively?
(a)1:1 (b)2:1 (c)3:1 (d)4:1
Answer: B
Solution:

GENERAL APTITUDE PAGE 65


Q284. Consider the following figure:

Given that BAC = ADC. If AB = 15 cm, AD = 20 cm, then what is CD?


(a)12cm (b)14cm (c)16cm (d)18
Answer:C

Q285. What is the length of the inradius of a triangle of sides 3 cm, 4cm and 5 cm?
(a)2cm (b)2.5cm (c)1.8cm (d) 1 cm
Answer:D
Solution: Area = r * (a + b + c)/2
 6 = r (6).
So, r = 1 cm.

GENERAL APTITUDE PAGE 66


Q286. What is the number of sides of a regular polygon if the sum of the internal angles in
degrees, is numerically equal to 60 times the number of sides of thepolygon?
Answer: 3
Solution: 180 (n – 2)=60n  n =3.
Q287. Consider the following figure:

Inthefigureabove,ABCisanequilateraltriangle.Thebisectorsof∠ABCand∠ACB
meetatD.WhatisthevalueofBDCindegree?
Answer: 120

Q288. Averticalstick20cmlongcastsashadow15cmontheground.Whatisthelengthof
alamppost,ifitcastsashadow60cmlongtheground?
(a)20cm (b)40cm (c)60cm (d) 80cm
Answer: D
Solution:

∆ABE and ∆CDE are similar.


CD/DE=AB/BE
20/15 = x/60
x = 80 cm.

GENERAL APTITUDE PAGE 67


Q289.

In the above figure, l1ans l2 are parallel lines. Find x.


(a)20° (b)30° (c)40° (d)50°

Q290.

In the above figure 𝑙1 and 𝑙2 are parallel lines. ∠𝐴𝐵𝐶= 80°. If the lines BD and CF are parallel
and. ∠𝐷𝐵𝐶 = 30°, then what is ∠𝐹𝐶𝐸?

(a)30° (b)40° (c)50° (d)60°

Q291.

In the above figure, AB and CD are parallel.


If BAD = ADC+10° and ABC = BAD – 40°, then what is BCE?
(a)55° (b)65° (c)75° (d)85°

GENERAL APTITUDE PAGE 68


Q292.

In the above figure, l1 and l2 are parallel lines. What is the value of y?
(a)15° (b)30° (c)45° (d)60°

Q293. In the above figure, the point C divides AE and BD in the ratio 2:1. What is ∠𝐴𝐶𝐵
given ∠𝐶𝐷𝐸 = 20°, ∠𝐵𝐴𝐶=40°?

(a)40° (b)80° (c)100° (d)120°

Q294.

IntheabovefigurethelinePSbisects∠𝑄𝑃𝑅,𝑎𝑛𝑑∠𝑃𝑆𝑄=∠𝑃𝑆𝑅,and

SPR = PRS + 10°. What is the ∠𝑄𝑃𝑅?

(a)70° (b)80° (c)100° (d)110°

GENERAL APTITUDE PAGE 69


Q295.

In the above figure, PS is the altitude drawn to side QR of triangle PQR.


∠𝑃𝑅𝑆 = ∠𝑄𝑃𝑆 −20°. What is the value of ∠𝑄𝑃𝑅 , if ∠𝑃𝑄𝑆 = 20°?
(a)100° (b)110° (c)120° (d)130°

Q296.

Intheabovefigure,∠𝑄𝑃𝑆=2∠𝑆𝑃𝑅=∠𝑃𝑅𝑆,Find∠𝑄𝑃𝑅.
(a)70° (b)80° (c)90° (d)100°
Q297. The sum of the interior angles of a regular polygon is 24 times the exterior angle.
What is the number of sides of the polygon?
(a)4 (b)6 (c)8 (d)12

Q298. Ifeachinteriorangleofaregularpolygonis150°,thenwhatarethenumberdiagonals?
(a)54 (b)48 (c)66 (d)60
Q299. In a triangle ABC, the in center is O. If ∠𝐵𝐶𝐸 =100°, ∠𝐵𝐴𝐶 is equal to

(a)70° (b)60° (c)20° (d)40°

GENERAL APTITUDE PAGE 70


Q300.

Intheabovefigure,if2∠𝑃𝑂𝑅=3∠𝑅𝑂𝑄and∠𝑃𝑂𝑄=100°.Thenwhatis∠𝑃𝑂𝑅? (a)30°

(b)40° (c)50° (d)60°


Q301.

Intheabovefigure,QPandQRaretangentstothecircle.If∠𝑃𝑄𝑅=50°,thenwhatis
∠𝑃𝑂𝑅?
(a)130° (b)120° (c)110° (d)100°

Q302.

In the above figure, AB is a tangent to the circle O. If CD bisects ∠𝐴𝐶𝑂 then what is
∠𝐶𝑂𝐷?

(a)80° (b)90° (c)100° (d)110°

GENERAL APTITUDE PAGE 71


Q303. Intheabovefigure,PQRSISACyclicquadrilateral.ifPQRisanequilateraltriangle find
∠RSP.

(a)120° (b)60° (c)30° (d)150°

Q304.

Intheabovefigure,Oisthecenterofthecircle.∠OQP+∠ORP=70. What is
theORQ?
(a)20° (b)30° (c)40° (d)50°

Q305.

In the above figure, AB is the diameter of the circle AC and BD are tangents drawn to
thecircleatAandB.Whatisthe∠𝐴𝐶𝐷,if∠CDB=70°?

(a)90° (b)110° (c)120° (d)130°

GENERAL APTITUDE PAGE 72


Q306. When a diagonal of a square is 24 cm then what is its perimeter?
(a)28cm (b)48√2cm (c) 36√2cm (d) 46 √2 cm
Answer:B

Q307.

Inthefigureabovel1andl2areparallellines.Thebisectorsof∠DACand∠ACE
MeetatB.IfAB=40cmandBC=9cm,thenwhatisthelengthofAC?

(a)41cm (b)42cm (c)44cm (d)45cm

Q308.

In the above figure, ∠PSR = ∠PQT, ∠TPQ =60°, ∠PQT = ∠PRS + 40°. What is ∠PRS?
(a)30° (b)40° (c)80° (d) 60°
Q309.

Inthefigureabove,CFandDGarethebisectorsof∠BCDand∠BDCrespectively. What is
the value of angle∠GFC?

GENERAL APTITUDE PAGE 73


(a)50° (b)60° (c)65° (d)75°
Q310. ABC is a right –angle triangle right angled at B the perpendicular drawn from B
meets AC at D, if AD =4 cm, DC=16 cm, then find BD.

(a)16cm (b)12cm (c)8cm (d)50cm

Q311.

In the above figure, O is the center of the circle, AB is a tangent to the circle and

OBE = 50. What is BOE?

(a)50° (b)40° (c)80° (d) 45°


Q312. Howmanycubeseachofedge6cmcanbecutfromacuboidwoodenblockof
44cm×38cm×27cm?
Answer: 168

Q313. The ratio of the length, breadth and height of cuboid is 5 : 4 : 2. What is the lateral
surface area (in m2) of the cuboid if the length of the longest diagonal that fits in the
cuboid is 15√5m?
Answer: 900

Q314. The ratio of the lengths of two cuboids is 30 : 11, that of their breadths is 11: 13, and
that of their heights is 26 : 15. What is the ratio of their volumes?
(a)1 : 4 (b) 1: 2 (c) 2:1 (d) 4 : 1

Q315. The length of the longest rod which can fit into a cubical room of volume 2197 m 3, is

(a)13√2𝑚 (b)13√3m (c)13m (d) none ofthese

GENERAL APTITUDE PAGE 74


Q316. Thebreadthofagardenishalfitslength.Aplaygroundmeasuring4.5squaremetres
occupiesonefourthofthetotalareaofthegarden.
The length (in metres)of thegardenis

Answer:6
Q317. A solid metallic spherical ball of diameter 6 cm is melted and recast into a cone with
diameter of the base as 12 cm. The height of the cone is .

(a)12cm (b)3 cm (c)24cm (d) 6 cm.

Q318. If the volume of a sphere is divided by its surface area, the result is 27 cm. The radius of
the sphere is:
(a)9cm (b) 36cm (c)54cm (d) 81cm

Q319. A right circular cone and a right circular cylinder have equal base and equal height. If the
radius of the base and the height are intheratio 5:12, then the
ratio of the total surface area of the cylinder to that of the coneis
(a)29:18 (b)13:9 (c)17:9 (d) 34:9

Q320. A hollow spherical metallic ball has an external diameter 6 cm and is 1cm thick.
2

The volume of metal used in the ball is :

(a)372𝑐𝑚3 (b)402𝑐𝑚3 (c)412𝑐𝑚3 (d)472𝑐𝑚3


3 3 2 3

Q321. The number of coins of radius 0 .75 cm and thickness 0.2 cm to be melted to make right
circular cylinder of height 8 cm and base radius 3 cm is:

(a)460 (b)500 (c)600 (d)640

Q322. Thecurvedsurfaceofarightcircularconeofheight15cmandbasediameter16cm. (a)60𝜋𝑐𝑚2

(b)68𝜋𝑐𝑚2 (c)120𝜋𝑐𝑚2 (d)136𝜋𝑐𝑚2

GENERAL APTITUDE PAGE 75


Q323. If a right circular cone of height 24 cm has vloume of 1232 cm3. Then the area of its curved
surface is :

(a)154cm2 (b)550cm2 (c)704cm2 (d) 1254 cm2


Q324. A circular well with a diameter of 2 metres, is dug to a depth of 14 metres. What is the
volume of the earth dug out?
(a)32m3 (b)36m3 (c)40m3 (d) 44m3

Q325. Ifthenumbersrepresentingvolumeandsurfaceareaofacubeareequal,thenthe length of the edge


of the cube in terms of the unit of measurement willbe:
(a)3 (b)4 (c)5 (d)6

Q326. The radius of the cylinder is half its height and surface area of the cylinder is616 sq. cms.
What is volume of the cylinder incm3?
Answer: 2156

Q327. The distance between the centers of two circles touching each other internally is 7 cm.
What is the area common to both the circles, if ratio of their radii is2 : 1? (in cm 2)
(a)154 (b)144 (c)6164 (d)1386
Answer: A

Q328. Twocircleswhichtoucheachotherexternallyhaveratiooftheirareasas4:9.Whatis the


radius of bigger circle, if the distance between the centers of the two circles is 75
cm?
(a)30cm (b)45cm (c)60cm (d) 40cm

Q329. What is the area of the shaded region, if O is the center of the circle and OA = 12
cm and AOB = 120?

GENERAL APTITUDE PAGE 76


(a) (48𝜋 − 36√3) 𝑐𝑚2(b) (12𝜋

− 3√3) 𝑐𝑚2(c) (24𝜋 − 36√3)

𝑐𝑚2

(d) (4𝜋 − 3√3) 𝑐𝑚2


CLOCK AND CALENDAR
Q330. In 16 minutes, the minute hand gains over the hour hand by
(a)16° (b)80° (c)88° (d)96°
Answer: C

Q331. At what time between 7 and 8 o'clock will the hands of a clock be in the same
straight line but, not together?
(a) 5 min.past
2
(b) 5 minutes past7
11
3
(c) 5 minutes past7
11
5
(d) 5 minutes past7
11

Answer: D
Q332. At what time between 2 and 3 o'clock will the hands of a clock be together?
109
(a) min past2
11
120
(b) min past2
11
131
(c) min past2
11
142
(d) min past2
11

Answer: B
Q333. An accurate clock shows 7 a.m. Through how many degrees will the hour hand
rotate when the clock shows 1 p.m.?
(a)154° (b)180° (c)170° (d)160°
Answer: B
Solution:Weknowthatangletracedbyhourhandin12hrs.=360° From 7
to 1, there are 6hours.
Angle traced by the hour hand in 6 hours =6*(360/12)=180°
Option B is the correct answer.

GENERAL APTITUDE PAGE 77


Q334. When the time is 5:40, then what is the angle between the hour hand and the minute
hand of a clock?
(a)70° (b)60° (c)74° (d)80°

Q335. What is the time between 3 and 4 will the hands of a watch point in the opposite
direction?
(a) 49(1/11) min past3
(b) 49(3/11) min past3
(c) 49(2/11) min past3
(d) 49(4/11) min past
3Answer:A

Q336. A clock is started at noon. By 10 minutes past 5, the hour hand has turned through:
(a)145° (b)150° (c)155° (d)160°
Answer: C
Solution:

Q337. How many times are the hands of a clock at right angle ina day?

Answer:44
Q338. How many times do the hands of a clock coincide inaday?

Answer:22

GENERAL APTITUDE PAGE 78


Q339. Aclockissetrightat8a.m.Theclockgains10minutesin24hours.Whatwillbethe
truetimewhentheclockindicates1p.m.onthefollowingday?
(a) 48min.past12.
(b) 46min.past12.
(c) 45 min. past12.
(d) 47 min. past12.
Answer: A
Solution:

GENERAL APTITUDE PAGE 79


Q340. Aclockgains5secondsforevery3minutes.Iftheclockstartedworkingat7a.m.in the
morning, then what will be the time in the wrong clock at 4 p.m. on the same day?
(a) 3 :45pm (b) 4 :15am (c) 5 :30pm (d) 4 :15pm
Solution:
A clock gains 5 seconds for every 3 minutes, then it will gain 50 seconds in 30
minutes,oritwillacquire100secondsin60minutes.i.e.itwillgain100secondsin1 hour.
Since the clock was started at 7 a.m. in the morning and right now the correct time is
4 p.m. the total time the clock has worked is 9 hours. We know that in 1 hour it gains
100 seconds then in 9 hours it increases 900 seconds. The conversion of 900
secondstominuteswillbe15minutes.Thisincreaseindicatesthataclockisfasterby
15 minutes as the clock is gaining. Hence, the time in the watch would be 4:15 p.m.

Q341. If today is a Monday then what will be the day after 68 days?
(a)Tuesday (b)Wednesday (c)Thursday (d)Saturday

Q342. [MSQ]
Which of the following years are leap years?
(a)700 (b)800 (c) 2000 (d)2100
Answer: b, c
Q343. 17th June, 1998 wasa
(a)Monday (b)Tuesday (c)Wednesday (d)Thursday

Q344. If it was Thursday on Aug 15, 2012, then what was the day on June 11, 2013?
(a)Wednesday (b)Monday (c)Saturday (d)Tuesday

GENERAL APTITUDE PAGE 80


Solution:
First, we count the number of odd days for the left over days in the given period.
Here, given period is from 15.8.2012 to 11.6.2013
Aug Sept Oct Nov Dec Jan Feb Mar Apr May Jun
1630313031312831303111(leftdays)
Therefore,2+2+3+2+3+3+0+3+2+3+4(odddays)=6odddays
So,thegivendayThursday+6=Wednesday.

Q345. What was the day of the week on 7th September, 1990?
(a)Wednesday (b)Thursday (c)Friday (d)Saturday

Q346. Tuesday fell on which of the following dates of April, 2002?


(a) 3rd, 10th,17th,24th (b) 1st, 8th, 15th, 22nd,29th
(c) 4th, 11th,18th,25th
(d)2nd,9th,16th,23rd,30thA
nswer:D
Solution:
We have to find the day on 1st April, 2002.
1st April, 2002 = (2000 years + time period from 1.1.2002 to 1.4.2002)
1600 years » 0 odd days
400 years » 0 odd days
Jan. Feb. March April
(31+28+31+1) = 91 days ≡ 0 odddays.
Totalnumberofodddays=(0+0+0)=0 On
April 1, 2002 it wasMonday.
So in April 2002, Tuesday falls on 2nd, 9th, 16th and 23rd & 30th
Q347. Which of the calendars would be same as that of the year 2007?
(a)2014 (b)2016 (c) 2017 (d)2018

Q348. If6thMarch,2005isMonday,whatwasthedayoftheweekon6thMarch,2004?
(a)Sunday (b)Saturday (c)Tuesday (d)Wednesday

Q349. Which two months in a year have the same calendar?


(a) October,December
(b) April,November
(c) June,October

GENERAL APTITUDE PAGE 81


(d) April, July

Q350. If the seventh day of a month is three days earlier than Friday, What day will it be on
the nineteenth day of the month?
(a) Sunday
(b) Tuesday
(c)Wednesday
(d)Monday
Answer: A
Solution:
The seventh day of the month is three days earlier than Friday, which is Tuesday.
So, the fourteenth day is also Tuesday and thus, the nineteenth day is Sunday.
Q351. XwasbornonMarch6,1993.ThesameyearIndependenceDaywascelebratedon Friday.
On which day was Xborn?
(a)Monday (b)Wednesday (c)Thursday (d)Friday
Answer: C
Solution:
NumberofdaysfromMarch6,1993toAugust15,1993. Mar
Apr May June JulyAugust
= 25 + 30 + 31 + 30 + 31 +15
= 162 days = 23 weeks + 1 day.
Clearly, the day on March 6 will be the same as on August 14, i.e., Thursday.

Q352. How many leap years does 100 years have?


(a)25 (b)24 (c)4 (d)26

Q353. The last day of a century cannot be


(a)Monday (b)Wednesday (c)Tuesday (d)Friday

Q354. The calendar of the year 2024 can be used again in the year?
(a)2030 (b)2052 (c) 2048 (d)2036

GENERAL APTITUDE PAGE 82


Q355. The maximum gap (in years) between two successive leap year is?
(a)4 (b)8 (c)2 (d)1

QUADRATIC EQUATION
Q356. The roots of the equation 3x2 - 12x + 10 = 0 are?
(a) rationalandunequal (b)real andequal
(c) irrationalandunequal (d) rational andequal

Q357. If the roots of a quadratic equation are 20 and -7, then find the equation?
(a) x2 + 13x - 140 = 0
(b)x2 - 13x + 140 = 0
(c) x2 - 13x - 140 = 0
(d) x2 + 13x + 140 = 0
Answer: C
Solution:
Any quadratic equation is of the form
x2-(sumoftheroots)x+(productoftheroots)=0 ------------ (1)
wherexisarealvariable.Assumoftherootsis13andproductoftherootsis-140,
thequadraticequationwithrootsas20and-7is:x2-13x-140=0.

Q358. If the sum and the product of the roots of the quadratic equation x 2 + 20x + 3 = 0 are
𝛼and𝛽,thenwhatthevalueof(𝛼+𝛽)−𝛼𝛽?

Answer: 43
Q359. The sum of the squares of two consecutive positive integers exceeds their product by
91. Find the integers?
(a)9,10 (b)10,11 (c) 11,12 (d) 12,13
Answer: A
Solution: Let the two consecutive positive integers be x and x + 1
x2 + (x + 1)2 - x(x + 1) = 91
x2 + x - 90 = 0
(x + 10)(x - 9) = 0 => x = -10 or 9.
As x is positive x = 9
Hence the two consecutive positive integers are 9 and 10.
GENERAL APTITUDE PAGE 83
Q360. Two equations (I) and (II) are given. You have to solve both the equations and find
relation between x and y.
I. x2 – 0.5x – 39 = 0
II. y2 – 15.5y + 60 = 0
(a)𝑥=𝑦 (b)𝑥<𝑦 (c)𝑥>𝑦 (d) can‟t say
Solution:
I. x2 – 0.5x – 39 = 0
x2 + 6x – 6.5x – 39 = 0
x(x + 6) – 6.5(x + 6) = 0
(x – 6.5)(x + 6) = 0
x = 6.5, – 6

II. y2 – 15.5y + 60 = 0 y2
– 8y – 7.5y + 60 = 0 y(y
– 8) – 7.5(y – 8) =0
(y – 8)(y – 7.5) = 0
y = 7.5, 8
Therefore, for any value of x and any value of y
x < y.

Q361. Two equations (I) and (II) are given. You have to solve both the equations and find
relation between x and y.
I. x3 – 14 – 1714 =0
II. 3y2 – 63 – 300 =0
(a)𝑥=𝑦 (b) 𝑥<𝑦 (c)𝑥 >𝑦 (d) can‟tsay

Q362. For what values of k, the roots of the quadratic equation:


(k + 4)x2 + (k + 1)x + 1 = 0 are equal?
(a)k =-5,-3 (b) k =-5,3 (c) k = 5,-3 (d) k = 5,3
Solution:
Wehave,(k+4)x2+(k+1)x+1=0
Here,a=k+4,b=k+1,c=1
D =0 …[∵ Roots are equal
b2 – 4ac = 0
∴ (k + 1)2 – 4(k + 4)(1) = 0
k2 + 2k + 1 – 4k – 16 = 0
k2 – 2k – 15 = 0

GENERAL APTITUDE PAGE 84


k2 – 5k + 3k – 15 = 0
k(k – 5) + 3(k – 5) = 0
(k – 5)(k + 3) = 0
k–5=0ork+3=0
k=5ork=-3
∴ k = 5 and-3

Q363. If the speed of a car is increased by 15 kmph, it takes one hour less to cover a
distance of 300 km. What is the original speed of the car?
(a)40kmph (b)50kmph (c)60kmph (d) 75kmph

Q364. What are the value of x that satisfy the inequality 3x2 + 8x + 5 >0?
(a)5> 𝑥 > −1
3

(b)−5<𝑥<−1
3

(c)−5>𝑥<−1
3
5
(d)− > 𝑥 and 𝑥 > −1
3

Q365. The two sides of a right –angled triangle containing the right angle are 3x+4 and
2x+3. If the area of the triangle is 20 m2 then what is the hypotenuse of the triangle?

3049 225 256 √3049


(a)√ m (b)√ m (c)√ (d) m6
4 9 6

Q366. Whatissumoftherootsoftheequation10x4-63x3+52x2+63x+10=0?

Answer:6.3
Q367. A person on a tour has Rs 9,600 for his expenses. If his tour is extended by sixteen
days, he has to cut done his daily expenses by Rs 20. What is the original durationof
the tour indays?
Answer: 80

Q368. If one root of the quadratic equation x2 + 11x + k = 0 is 4, find the other root and the
value of k (respectively).

GENERAL APTITUDE PAGE 85


(a)-15,-60 (b)-4,15 (c)2,30 (d)8,1

11
Q369. If x < 0, what the maximum value of 𝑥 + ?
11 𝑥

(a)1 (b)-1 (c)-2 (d)0

Q370. Which of the following is a root of 52x – (24) × (5x) – 25 = 0?


(a) 2
(b)-2
(c)-1
(d)none of these

𝑥 𝑥
Q371. The roots of the equation (2 + √3) +(2 − √3) = 14 are

(a)2,-1
(b)1,2
(c)2,-2
(d) none of these
2 2
Q372. If P is rational number and 12 (𝑃 + 1) = 120 + 5 (𝑃 − 1) , then what is P?
𝑃 𝑃

(a)2,1 (b) 1,3 (c)3,-1 (d) 5,1


2 3 3 5

Q373. What is the sum of all possible value of x, for which the following equation satisfies?
32𝑥−12.3𝑥+27=0
Answer: 3

Q374. What value of x satisfies the equation

√(4 𝑥 + 5) + √(8 𝑥 − 4) = 11?


(a)181 (b)5 (c)11 (d)none ofthese

GENERAL APTITUDE PAGE 86


Q375. If (3𝑥)2+(27.31/𝑝−15)𝑥+4=0hasrealandequalroots,thenwhatisthevalueof P?
(a)2 (b)(-2) (c)(-1/2) (d)3

Q376. For what value of x is √7𝑥 + 7 − √4𝑥 + 1 = 2?

(a) 3,-3/9
(b) 6,-2/9
(c) 8, 2/5
(d) 16, 1/5

1 10
Q377. If√2𝑥−3+ = then 𝑥 is equalto
√2𝑥−3 3

(a)6,14 (b)3,1 (c)6,1 (d)3,14


9 3 3 9

Q378. The number of real solution of the equation x2 – 3|x|+ 2 = 0 is


(a)4 (b)1 (c)3 (d)2
Answer: A
Solution:

GENERAL APTITUDE PAGE 87


Q379. Let two numbers have arithmetic mean 9 and geometric mean 4 . Then these
numbers are the roots of the quadratic equation
(a)x2 + 18 x - 16=0
(b) x2 – 18 x + 16 = 0
(c) x2 + 18 x + 16 =0
(d) x2 – 18 x – 16 = 0
Answer: B
Solution:

GENERAL APTITUDE PAGE 88


Q380. If (1-p) is a root of quadratic equation x2 + px + (1 – p) = 0, then its roots are .
(a)0,-1 (b)-1,1 (c)0,1 (d) -1,2
Answer: A
Solution:

LOGARITHMS
Q381. [MSQ]
If 𝑥=1+log 𝑎𝑏𝑐,𝑦=1+log 𝑏𝑐𝑎,𝑧=1+log 𝑐𝑎𝑏,thenwhichofthefollowing
statement is are true?
(a) 𝑥𝑦𝑧 + 𝑥𝑦 + 𝑦𝑧 + 𝑧𝑥 = 0
(b) 𝑥𝑦 + 𝑦𝑧 + 𝑧𝑥 − 𝑥𝑦𝑧 = 0
1 1
(c) 1 + + = 0
𝑥 𝑦 𝑧
1 1
(d) 1+ + = 1
𝑥 𝑦 𝑧

Answer: b, d

GENERAL APTITUDE PAGE 89


Q382. If 𝑎 𝑥=𝑏,𝑏 𝑦=𝑐,𝑐 𝑧=𝑎,𝑥=log 𝑏𝑎 𝑝,𝑦=log 𝑐𝑏 𝑞,𝑧=log 𝑎𝑐 𝑟,𝑡𝑕𝑒𝑛whatisthevalue of p × q × r?

Answer: 1

Q383. Forwhichvalueofxtheequation:3𝑥log52+2𝑙𝑜𝑔5𝑥=64,satisfies?

Answer:625
Q384. The number of the solutions of log4(𝑥 − 1) = log2(𝑥 − 3) is
(a)3 (b)1 (b)2 (d)0

Q385. 𝐼𝑓4𝑙𝑜𝑔16 4+9𝑙𝑜𝑔3 9=10log𝑥83 ,thenwhatisthevalueofx?

Answer:10

PROGRESSIONS (SERIES)
Q386. The interior angles of a polygon are in arithmetic progression. The smallest angleis
120 and the common difference is 5. What is the number of sides of the polygon?
(a)16 (b)9 (c)7 (d) Both a &b
Answer: B
Q387. IfthesumofthreenumbersinH.Pis37andthesumoftheirreciprocals0.25,then the
product of these numbers willbe
Answer: 1800

Q388. If the sum to n terms of these series, 5 + 55 + 555 + …, is


5𝑛
𝑝(10𝑛−1)− ,thenwhatisthevalueofp+q?
𝑞 9

Answer: 131

Q389. Whatisthesumoftheseries(33−23 )+(53−43 )+(73−63 )+⋯𝑡𝑜10𝑡𝑒𝑟𝑚𝑠?

Answer: 4960

GENERAL APTITUDE PAGE 90


Q390. WhatisthearithmeticmeanofaseriesinAP,with33termswhosetermis2and
commondifferenceis2.25?
Answer: 38

Q391. Consider three terms in arithmetic progression such that their sum is 33 and product
is1155.Whatiscommondifference?
Answer: 4

Q392. The sum to n terms of a series in AP is 6𝑛2+ 6𝑛. What is the 4thterm of the series?

Answer: 48
Q393. Three terms are in geometric series such that their sum is 49 and product 2744.What
is the sum of first term and commonratio?
Answer: 9

Q394. The third term of a GP is 4. The product of first five terms is


(a)64 (b)1024 (c)256 (d) none ofthese

(𝑐+𝑏)
Q395. If (𝑎+𝑏),𝑏, areinAP,thena,1,carein
1−𝑎𝑏 1−𝑐𝑏 𝑏

(a)AP (b)GP (c)HP (d) none ofthese

Q396. If a, b, c are in AP as well as in GP, then


(a)𝑎=𝑏≠𝑐 (b) 𝑎 ≠𝑏 ≠𝑐 (c)𝑎≠𝑏=𝑐 (d) 𝑎 = 𝑏 =𝑐
Q397. Ifthesumtoinfinityoftheseries1+4x+7x2+10x3+…is35,thenwhatisthe
16

value of x?
(a)0.2 (b) 19/7 (c) Both a&b (d)can‟t
determined

Q398. If 𝑆∞= 1 + 22x + 32x2 + 42x3 + …….∞; where |x|<1, then 𝑆∞ is equivalent to

GENERAL APTITUDE PAGE 91


(1+𝑥) (1+𝑥) (1+𝑥)
(a) (b) (c) (d) none ofthese
(1−𝑥) (1−𝑥)2 (1−𝑥)3

Q399. Whatissumoftheseries1+3+7+15+31+…..upto16 thterm?

Answer:131054
Q400. Whatis60thtermoftheseries1+2+5+12+25+46+…?
Answer: 68500

Q401. What is the nth term of the series 2 + 5 + 12 + 31 + 86 +…..?


(a)3𝑛−1 (b)3𝑛−1−𝑛 (c)3𝑛−1+𝑛 (d) None ofthese

Q402. [MSQ]
Howmanytermsseries54+51+48+45+…mustbetakentomake513? (a)16
(b)17 (c)18 (d)19
Answer: c, d

STATISTICS AND INEQUALITY


Q403. For the individual series compute the mean and median.
13,15,20,22,30,35,38,40,42,53
(a)Mean = Median=30.8 (b)Mean=32.5,Median=30.8
(c)Mean=30.8,Median=32.5 (d) None ofthese

Q404. Whatisthevarianceof1,2,3,4,5?
Answer: 2

Q405. Whatisthestandarddeviationof1,2,3,4,5,6?
Answer: 1.71

Q406. What is the range and quartile deviation from the following marks of 13 students in
Mathematics?
60, 40, 10, 72, 85, 96, 38, 41, 52, 28, 9, 35, 25.

GENERAL APTITUDE PAGE 92


(a) Range = 45 and Quartile deviation =19.75
(b) Range = 87 and Quartile deviation =26.5
(c) Range = 87 and Quartile deviation =66
(d) Range = 87 and Quartile deviation =19.75
Q407. Quartiledeviationof10,13,18,21,24,36,50,84,63,90,75is
Answer: 28.5

Q408. What is the mean deviation about the mean for the following data:
12, 18, 25, 30, 35
(a)3.5 (b)7.2 (c)7.5 (d)6.8
Answer: B
Solution: Mean = 1/5 (12 + 18 + 25 + 30 + 35) = 120/5 = 24
Mean Deviation = 1{|12 – 24|+|18 – 24|+|25 – 24|+|30 – 24|+ |35 – 24|}
5
= 36/5 = 7.2

Q409. The standard deviation of 8, 8, 8, 8, 8, 8, 8, 8, 8, 13 is


(a)2.25 (b)6.25 (c)2.35 (d)1.5
Answer: D

Q410. The variance of the numbers -35, -30, -5, -20, -15 is
(a)20 (b)50 (c)25 (d)114

Q411. If the sum of the squares of 8 observations is 144 and their mean is 3, then the
standard deviation is
(a)9 (b)3 (c)1.73 (d)81
Q412. How many integers satisfying the inequality
(𝑥2−14𝑥+45)(𝑥2−4𝑥−21)(𝑥2−11𝑥+10)<0?

Answer:4
Q413. Whatistherangeofallrealvalueofxif||x–3|+5|<4?
(a)(3,12)
(b) (-6,4)
(c) (0, 5)

GENERAL APTITUDE PAGE 93


(d) None of these

Q414. If |x – 3|<2, then the solution set of x is


(a)(1, 3) only
(b) (3, 5) only
(c) (1, 3) ∪ (3, 5)
(d) (1, 5)
Q415. If(𝑥+3)(𝑥+2)(𝑥+1)2≤0,thenthesolutionsetofxfortheinequalityis (a) [-3,-1]
(b) [-3, -2] U {-1}
(c) [-3, -2]
(d) (-3, -2)

Q416. If (𝑥−3)>5,thenthesolutionsetofxfortheinequalityis
(𝑥+2)

(a)(2, 13/4)
(b)(-2,13/4)
(c) (-13/4,2)
(d) (-13/4, -2)
DATA INTERPRETATION (TABLE, PIE CHART, BAR DIAGRAM AND LINE GRAPH)
Directionfornextfourquestions:Studythefollowingtablechartcarefullyandanswerthe
questions given beside.
The table given below shows the percentage of appeared and qualified candidates in a
competitive examination from different institutes.

GENERAL APTITUDE PAGE 94


Q417. What is the ratio of the qualified candidates from institutes D, E and F together to the
appeared candidates from institutes A, B and C together?
(a) 11 :17
(b) 12 :67
(c) 19 :75
(d) 13 : 75
Answer: D
Solution:
Qualified candidates from D, E and F = (16 + 20 + 16) % of 4000
Appeared candidates from A, B and C = (25 + 10 + 15) % of 24000

52 × 4
Reqd. ratio = = 13 : 75
50 × 24
Hence, option D is correct.

Q418. What percentage of the candidates from institute C has been declared qualified out
of the total candidates appeared from this institute?
(a)30% (b)25% (c)40% (d)20%
Answer: D
Solution:
Qualified candidates from institute C = 18% × 4000
Appeared candidates from institute C = 15% × 24000

18 × 4 × 100
Reqd. % = = 20%
15 × 24
Hence, option D is correct.

Q419. What is the percentage of students who qualified from the institute C and D together
with respect to those who appeared from the institute C and D together?
(a)24.98% (b)30.98% (c)20.98% (d)31.98%
Answer: C
Solution:
Qualified candidates from C and D together = {18 + 16} % × 4000
Appeared candidates from C and D together = {15 + 12} % × 24000

34 × 4 × 100
Reqd. % = = 20.98%
27 × 24
Hence, option C is correct.

GENERAL APTITUDE PAGE 95


Q420. Which institute has the highest percentage of candidates qualified with respect to
those appeared?
(a)A (b)D (c)E (d)C
Answer: B
Solution: Percentage of candidates qualified with respect to those appeared

18 × 4
A=
24 × 25

12 × 4
B=
10 × 24

18 × 4
C=
15 × 24

16 × 4
D=
12 × 24

20 × 4
E=
18 × 24

16 × 4
F=
20 × 24
In these values 4/24 can be discarded because it is common in comparison.
So, here, A and F are less than 1, so they are removed without solving.
B = 1.2, C = 1.2, D = 1.33, E = 1.1
D has the highest percentage of qualified candidates with respect to those appeared.
Hence, option B is correct.

Directions for next four questions: Following table shows the mobile addicted population
ofsixdifferentcitiesMumbai,Delhi,Jaipur,Kochi,PuneandAgraandtheratioofMalesto Females
among them and the ratio of Adults to Children in the total mobileaddicted
population. Answer the given questions based on this table

GENERAL APTITUDE PAGE 96


Q421. What is the total number of addicted females in Agra?
(a)1.432lakh (b) 1.578lakh
(c)1.708lakh (d) 1.981lakh
Answer: D
Solution: Total addicted female population of Agra
7
= 424500 × = 1.981 lakh
15
Hence, option D is correct

Q422. Whatisthedifferencebetweenthetotalnumberofadultandthetotalnumber
childrenmobileaddictedpopulationofMumbai?
(a)0.918lakh (b) 1.026lakh
(c)1.124lakh (d) 1.107lakh
Answer: B
Solution: ∴ Required Difference
(5 – 3)
= 410400 ×
8
= 1.026 lakh
Hence, option B is correct.

Q423. Which of the following cities has the maximum addicted adult population?
(a) Pune
(b) Kochi
(c) Jaipur
(d) Delhi
Answer:D
Solution:AdultpopulationofMumbai-2.565lakh,Delhi-2.877lakh,
Jaipur-2.583lakh,Kochi-2.66lakh,Pune-2.775lakh
Hence, Delhi has the maximum number of mobile addicted adults.
Hence, option D is correct.

GENERAL APTITUDE PAGE 97


Q424. The female addicted population of Delhi is approximately what per cent of its male
addicted population?
(a)40%
(b) 60%
(c) 80%
(d)120%
Answer: C
Solution: The ratio of addicted males to females in Delhi = 5 : 4
4
∴ Required% = × 100 = 80%
5
Hence, option C is correct.

Data for next three questions:


The pie chart below shows the percentages of blood types for a group of 200 people.

Q425. How many people, in this group, have blood typeAB?


Answer: 38
Solution: 19% × 200 = 19 × 200 /100 = 38 people
Q426. How many people, in this group, do not have blood typeO?
Answer: 120
Solution: (100% - 40%) × 200 = 60 × 200 /100 = 120 people

Q427. How many people, in this group, have blood types A orB?
Answer: 82
Solution: (16% + 25%) × 200 = 41 × 200 /100 = 82

Data for next two questions:

GENERAL APTITUDE PAGE 98


The following pie-chart shows the number of students appearing GATE Examination From
different states.

Q428. Number of students appearing from B state in 2016 was, what percent of that from
state F in 2015?
(a)94 (b)95 (c)96 (d)97.
Answer: C
Solution:
Number of students appearing from B state in 2016 = 20*240000/100 = 48000
Number of students appearing from F state in 2015 = 20*250000/100 = 50000
Percentage = 48000*100/50000 = 96%

Q429. The total number of students appearing from B & C together in 2015 is
approximately equal to that from which of the following pairs of states in 2016?
(a) A&F (b) C&D (c) A&C (d) D &F
Answer: C
Solution:
StudentsappearingfromBandCin2015=(18+12)*250000/100=75000 Go
through options verification check one byone
You will find option C is approximately equal to 75000

Data for next four questions:


Study the pie chart and table carefully based on that answer the following questions.
The pie chart shows the percentage of Employees working in different government
department
Total number of Employees=1200

GENERAL APTITUDE PAGE 99


The table shows the number of females in each department:

Q430. What is total number of males working in department Manager, HR and Clerk?
(a)400 (b)380 (c)394 (d)396

Q431. What is the ratio of females working in department Manager and clerk and males in
department HR and marketing?
(a)56:37 (b)56:35 (c)55:37 (d)55:30

Q432. Number of Females working in Manager department is what percentage


(approximately) of total number of employees working in all the department?
(a)17% (b)20% (c)9% (d)15%
Q433. What is the central angle corresponding to the total number of clerk?
(a)52.6° (b)55.2° (c)61.1° (d)57.6°

Q434. What is the ratio between total number of female employees working in all the
department together and males working in all the department?

GENERAL APTITUDE PAGE 100


(a)187:200 (b)199:201 (c)199:221 (d)201:199

Data for next five questions: Study the graph carefully and answer the following question.
Data related to Income (in Rs thousands) and Expenditure (in Rs thousands) of company
during six months.

Profit =(Income –Expenditure )


Percent profit =(Profit /Expenditure *100)
Loss =(Expenditure-Income )
Loss percent =(Loss/Expenditure *100)
Q435. What is average profit earned by company In January, February and May?
(a)280 (b)220 (c)240 (d)200

Q436. What is total loss percentage incurred by company in March and April?
(a)40 (b)50 (c)30 (d)45
Q437. Profit earned by company in Feb is by what percent more than profit earned by
company in May.
(a)443 (b)488 (c)384 (d)444
9 9 9 9

Q438. In Which month company earned the maximum profit?

GENERAL APTITUDE PAGE 101


(a)January (b)February (c)March (d)April

Q439. If company income increased by 20% from June to July and Expenditure decrease by
10% .What was his profit percent in month of July?
(a)4.76 (b)2.56 (c)5.88 (d)3.17

Data For next five questions:


Study the following line graph and answer the questions.

Q440. For which of the following pairs of years the total exports from the three Companies
together are equal?
(a) 1995and1998 (b) 1996 and1998
(c) 1997and1998 (d) 1995 and1996
Answer: D
Solution:

GENERAL APTITUDE PAGE 102


Q441. Average annual exports during the given period for Company Y is approximately
what percent of the average annual exports for Company Z?
(a)87.12% (b)89.64% (c)91.21% (d)93.33%
Answer: D
Solution:

Q442. In which year was the difference between the exports from Companies X and Y the
minimum?
(a)1994 (b)1995 (c)1996 (d)1997
Answer: C
Solution:

GENERAL APTITUDE PAGE 103


Q443. What was the difference between the average exports of the three Companies in 1993
and the average exports in 1998?
(a) Rs.15.33crores (b) Rs. 18.67crores
(c) Rs.20crores (d) Rs. 22.17
croresAnswer:C
Solution:

Q444. In how many of the given years, were the exports from Company Z more than the
average annual exports over the givenyears?
Answer: 4
Solution:
Average annual exports of Company Z during the given period
1
= x (60 + 90 + 120 + 90 + 60 + 80 + 100)
7
600
=Rs. crores
7
= Rs. 85.71crores.

GENERAL APTITUDE PAGE 104


From the analysis of graph the exports of Company Z are more than the average
annualexportsofCompanyZ(i.e.,Rs.85.71crores)duringtheyears1994,1995,1996
and1999,i.e.,during4ofthegivenyears.
Data for next five questions:
The bar graph given below shows the sales of books (in thousand numbers) from six
branches of a publishing company during two consecutive years 2000 and 2001.

Q445. WhatistheratioofthetotalsalesofbranchB2forbothyearstothetotalsalesof branch B4 for


bothyears?
(a)2:3 (b)3:5 (c)4:5 (d)7:9
Answer: D
Solution:

Q446. TotalsalesofbranchB6forboththeyears,iswhatpercentofthetotalsalesof branches B3 for


both theyears?
(a)68.54% (b)71.11% (c) 73.17% (d)75.55%
Answer: C
Solution:

GENERAL APTITUDE PAGE 105


Q447. WhatpercentoftheaveragesalesofbranchesB1,B2andB3in2001istheaverage
salesofbranchesB1,B3andB6in2000?
(a)75% (b)77.5% (c) 82.5% (d)87.5%
Answer: D
Solution:

Q448. What is the average sales of all the branches (in thousand numbers) for the year
2000?
(a)73 (b)80 (c) 83 (d)88
Answer: B
Solution:
Average sales of all the six branches (in thousand numbers) for the year 2000
1
= x [80 + 75 + 95 + 85 + 75 + 70]
6
= 80.

Q449. TotalsalesofbranchesB1,B3andB5togetherforboththeyears(inthousand numbers),is?


(a)250 (b)310 (c) 435 (d)560
Answer: D
Solution: Total sales of branches B1, B3 and B5 for both the years
(in thousand numbers) = (80 + 105) + (95 + 110) + (75 + 95) = 560.

GENERAL APTITUDE PAGE 106


Directions for next four questions: Study the following line graph and table carefully and
answer the questions given below.
Numbers of employees working in five different banks A, B, C, D and E.

Ratio of males to females employees

Q450. What is the total number of male employees taking all the banks together?
(a)4060 (b)4120 (c)4180 (d)4280
Answer: C
Solution:

Required answer = 4180

GENERAL APTITUDE PAGE 107


Q451. What is the average number of female employees taking all the banks together?
(a)656 (b)686 (c)668 (d)646
Answer: A
Solution: Required Average = 1(480 + 720 + 803 + 780 + 497) = 3280/5 = 656.
5

Q452. Approximately by what percent is the number of male employees working in banks
A and C together more than that of the total number of female employees working in
bank B andD?
(a)9% (b)15% (c)11% (d)13%
Answer: D
Solution:

Q453. What is the ratio of female employees working in bank D to that in E?


(a) 7: 4 (b) 8: 5 (c) 7:3 (d) None of these
Answer:D
Solution: Required ratio = 780 : 497

Q454. Approximately by what per cent is the number of total employees in bank C more
than that of bank D?
(a)8% (b)6% (c)4.4% (d)10%
Answer: B
Solution:

Q455. The following line graph gives the percentage of the number of candidates who
qualified an examination out of the total number of candidates who appeared for the
examination over a period of seven years from 1994 to 2000.

GENERAL APTITUDE PAGE 108


Thetotalnumberofcandidatesqualifiedin1999and2000togetherwas33500and the
number of candidates appeared in 1999 was 26500. What was the number of
candidates in2000?
(a)24500 (b)22000 (c)20500 (d)19000
Answer: C
Solution:

ADVANCED LEVEL QUESTIONS


Q456. Which of the following numbers will always divide a six-digit number of the form
xyxyxy(where 1 <x < 9, 1 <y < 9)?
(a)1010 (b)10101 (c) 11011 (d)11010

Q457. A student was asked to multiply a given number by 8/17 . Instead, he divided the
given number by 8/17 . His answer was 225 more than the correct answer. The given
number was
(a)64 (b)289 (c)136 (d)225

GENERAL APTITUDE PAGE 109


Answer: C

Q458. On dividing 2272 as well as 875 by 3-digit number N, we get the same remainder.
The sum of the digits ofNis _
(a)13 (b)12 (c)11 (d)10
Answer: D
Solution:
(2272-875) = 1397, is exactly divisible by N.
Now , 1397 = 11 x 127
The required 3-digit number is 127,the sum of digits is 10.

Q459. A number when divided by 7 leave a reminder x. When divided by 19 it leave a


reminder 2x. Also when divided by 39 it leave a remindre 3x. If x = 4, then find the
least possible such number.
(a)4305 (b)4302 (c)4263 (d)9489
Answer: B
Solution:

GENERAL APTITUDE PAGE 110


Q460. What is the number which when successively divided by 8, 9 and 10, leaves a
remainders of 6, 7 and 8 respectively and last quotient is five?
(a)3182 (b)529 (c)4238 (d)591

Q461. It is being that (232+ 1 )is completely divisible by whole number. Which of the
following numbers is completely divisible by this number?
(a)(216+1) (b)(216−1) (c)(7×233) (d) 296 +1
Answer: D
Solution:Let𝑥=232 ,𝑡𝑕𝑒𝑛(232+1)=𝑥+1
296 + 1 = (232)3+ 1 = (𝑥3 + 1) = (𝑥 + 1)(1 + 𝑥 + 𝑥2);
This is divisible by x +1. Therefore, (296+ 1 )is completely divisible by (232+ 1 ) .

Q462. What is the remainder when N = (1! + 2! + 3! + … 10000!) 50 is divided by 10?

Answer: 9

GENERAL APTITUDE PAGE 111


…∞
Q463. 7
Whatistheremainderwhen77 is divided by5?
Answer: 3

Q464. [MSQ]
LetN=(n–2)(n–1)n(n+1)(n+2);wherenisanaturalnumber.Whichofthe following
statement(s) is/aretrue?
(a) The unit digit of N is0.
(b) N is perfectly divisible by24.
(c) N is perfect square.
(d)(N+1)2mod10=1.
Answer: a, b, d

Q465. LetN=n2 +n+41;wherenisanaturalnumber.Forwhichsmallestvalueofn,Nisa


perfectsquare?
Answer: 40

Q466. A number when divided successively by 4 and 5 leaves remainder 1 and 4


respectively. When it is successively divided by 5 and 4, then the respective
remainders will be
(a)1,2 (b)2,3 (c)3,2 (d) 4,1
Answer: b

Data for next two questions: A trader gives a discount of 16% and earns a profit of 40%.If
the difference between the marked price and the cost price is Rs 400, then
Q467. Whatwillbethesellingprice(inRs)?
Answer: 840
100
Solution: Let MP = Rs 100, then CP = 84 × = 𝑅𝑠 60
140
MP – CP = 100 – 60 = 40
Since, the difference between the marked price and the cost price is Rs 400, hence
40% of MP=400 => 𝑀𝑃 = 𝑅𝑠1000
Therefore SP = 1000 × 0.84 = Rs 840.

GENERAL APTITUDE PAGE 112


Q468. WhatisthecostpriceinRs?
Answer: 600
840
Solution: CP = 𝑆𝑃= = 600 𝑅𝑠
1.4 1.4

Q469. A shopkeeper marked a product, 20% above the cost price and sold the product for
Rs. 3888 by giving two successive discounts of 10% each. Find the cost price of the
product and the loss percentage of the shopkeeper.
(a)4200,7.4% (b)4000,2.8% (c)4100,5.1% (d) 4050,4%
Answer: B
Solution:

Q470. The marked price of a John Players Denim is Rs. 1490. On the occasion of
Janmashtami, the showroom offered two successive discounts of 11.25% and D%
respectively. If an individual purchased the Denim for Rs. 1193.44, then find the
value of value of „D‟?
(a)9.75 (b)8.75 (c)5.5 (d)10.75
Answer: A
Solution:

GENERAL APTITUDE PAGE 113


Q471. The C.P of a Denim is 10% more than C.P of a T-shirt. The shopkeeper marked up
Denim at 20% above the C.P and T-shirt at 30% above the C.P. Find the sum of
marked price of 4 Denims and 5 T-shirts, if shopkeeper allowed discount of 20% on
T-shirtand25%onDenimandthedifferencebetweensellingpricesofbothisRs.50.

Answer: 11780
Solution:

GENERAL APTITUDE PAGE 114


Q472. An Article costs Rs. 5000 and it is marked up 40% by the shopkeeper. A customer
walks into the shop and seems really interested in the article. Sensing this, the
shopkeeper gets greedy and he raises the markup % to 80% and gives a discount of
20% to the customer. How much more/less money would he had made, had he not
gotten greedy?
(a) Rs.200more (b) Rs.200less (c)Rs.400more (d)Rs. 400less
Answer: B
Solution:

GENERAL APTITUDE PAGE 115


Q473. ThemarkedpriceofanarticleisRs.3,000.Iftwosuccessivediscounts,eachofx%,on the
marked price is equal to a single discount of Rs. 1,174.80, then what will be the
sellingpriceofthearticleifasinglediscountofx%isgivenonthemarkedprice?

Answer: 2340
Solution:

Q474. There four articles are sold at the same price (i.e., the selling prices are equal).
 Firstoneissoldataprofitof20%,
 Secondoneissoldataprofitof20%,
 Thirdoneissoldatalossof20%,and
 Fourth is sold at a loss of20%.
What is the overall percentage of profit/loss?
(a)0%loss (b) 4%profit (c)4%loss (d) Insufficient
dataAnswer:C
Solution:
1 1 1 1
(+ − − )
Required %ofloss==1.21.20.80.8×0.2×100=4%
1 1 1 1
1.2+1.20.+80.8+

GENERAL APTITUDE PAGE 116


Q475. There four articles are sold at the same price (i.e., the selling prices are equal).
 Firstoneissoldataprofitof25%,
 Secondoneissoldataprofitof50%,
 Thirdoneissoldatalossof25%,and
 Fourth is sold at a loss of20%.
What is the overall percentage of profit/loss?
(a)1.2345%loss (b) 1.2345 %profit
(c)25%profit (d) None ofthese
Answer: A
Solution:
0.25 0.5 0.25 0.2
( + − − )
Required %ofloss=1.251.50.750.8×100=1.234567%
1 1 1 1
+ − −
1.251.50.750.8

Q476. Rs.39030isdividedbetween„a‟and„b‟insuchawaythatamountgivento„a‟onC.I.
in7yearsisequaltotheamountgivento„b‟onC.I.in9years.Findthepartof„a‟.If the rate of
interest is4%.
(a)20200 (b)20900 (c)20280 (d)20100
Answer: C
Solution:

Q477. What will be the difference between simple and compound interest on a sum of Rs.
4500 put for 2 years at 5% per annum?
(a)12.45 (b)12.95 (c)11.25 (d)10.25
Answer: C
Solution:

GENERAL APTITUDE PAGE 117


Q478. What will be the amount if a sum of Rs. 5000 is placed at compound interest for 3
years while the rate of interest for the first, second and third years is 2%, 3% and 4%
respectively?
(a)5473.12 (b)5463.12 (c)5163.12 (d)5353.12
Answer: B
Solution:

Q479. The compound interest on Rs. 10,000 in 2 years at 4% per annum the interest being
compounded half-yearly, is :
(a)824.3216Rs (b)804.3216Rs (c)814.3216 Rs (d) 834.3216Rs
Answer: A
Solution:

Q480. Thecompoundinterestonacertainsumofmoneyinvestedfor2yearsat5%per
annumisRs.328.Thesimpleinterestonthesum,atthesamerateandforthesame period
willbe?
(a)340 (b)320 (c)330 (d)390

GENERAL APTITUDE PAGE 118


Answer: B
Solution:

Q481. AsumofRs.2000amountstoRs.4000intwoyearsatcompoundinterest.Inhow
manyyearsdoesthesameamountbecomesRs.8000?
(a)2years (b)4years (c)6years (d) 8years
Answer: B
Solution:

Q482. A, B and C are partners. A got the (2/3)rd of the total profit. B and C got the rest
amount in equal ratio. When the rate of profit is increased from 5 to 7%, there is a
increase of Rs. 400 in profit of A. What will be the profit of B?
(a)Rs.500 (b)Rs.250 (c)Rs. 480 (d) Rs.350
Answer: D
Solution: Let the cost = 100x, i.e. Profit = 5% of 100x =5x
A's profit =(2/3)*5x
B'sprofit=(1/2)*(1/3)*5x=(1/6)*5x
C'sprofit=(1/2)*(1/3)*5x=(1/6)*5x
NewProfit=7%of100x=7x
i.e.NowA'sprofit=(2/3)*7x
ChangeinA'sprofit=(2/3)*7x-(2/3)*5x=(4/3)x
Given, (4/3)*x =Rs.400

GENERAL APTITUDE PAGE 119


i.e. x = Rs.300
B's profit = (1/6)*7x = 7*300/6 = Rs.350

Q483. P,QandRsignedforaprojectofRs.23600.PinvestedRs.5000morethanQandQ
investedRs.3000morethanR.IftheprofitisRs.35400,findtheshareofQ.
(a)Rs.21800 (b)Rs.18000 (c)Rs.10800 (d) Rs.11400

Q484. In a mixture of milk and water the proportion of water by weight was 75%. If in 60
gm of mixture 15 gm water was added, what would be the percentage of water?
(Weight in gm)
(a)80% (b)70% (c) 75% (d)62%
Answer: A
Solution:
Water in 60 gm mixture=60 x 75/100 = 45 gm. and Milk = 15 gm.
Afteradding15gm.ofwaterinmixture,totalwater=45+15=60gmandweightof
amixture=60+15=75gm.
So % of water = 100 x 60/75 = 80%.

Q485. The average weight of boys in a class is 30 kg and the average weight of girls in the
sameclassis20kg.Iftheaverageweightofthewholeclassis23.25kg,whatcouldbe
thepossiblestrengthofboysandgirlsrespectivelyinthesameclass?
(a) 14and16 (b) 13and 27 (c) 17and27 (d) None ofthese
Answer: B
Solution:

Q486. How many litres of pure water must be mixed with 175 litres of pure spirit in order
to gain (100/7) % by selling the spirit it at its cost price?
(a)24litres (b)25litres (c)50litres (d) 75litres
Answer: B

GENERAL APTITUDE PAGE 120


Q487. Apersonemployed15mentodoaworkin76days.After20daysherealizedthatthe work
cannot be finished the scheduled time with the present work force. So he employed
five more men and the work is finished one day earlier. How many days would he
have been behind the schedule if he had not admitted the newmen?
(a)73.33days (b) 93.33 days (c)95.33days (d)17.33days
Answer: D
Solution:

Q488. ThreecarsleaveAforBinequaltimeintervals.TheyreachBsimultaneouslyandthen
leaveforPointCwhichis240kmawayfromB.ThefirstcararrivesatCanhourafter the
second car. The third car, having reached C, immediately turns back and heads
towardsB.Thefirstandthethirdcarmeetapointthatis80kmawayfromC.Whatis
thedifferencebetweenthespeedofthefirstandthethirdcar?
(a)60kmph (b)20kmph (c)40kmph (d) 80kmph
Answer: A
Solution: Create a visual of where the three cars are and how they travel between the
points.

GENERAL APTITUDE PAGE 121


Condition I states that the cars leave in equal intervals of time and arrive at the same
time. Or, the difference in the time taken between cars 1 and 2 should be equal to the
time taken between cars 2 and 3.

GENERAL APTITUDE PAGE 122


So, Required difference = the difference between the speed of the first and the third
car = 120 – 60 = 60 kmph

Q489. Ankit when going slower by 15 Km/hr, reaches late by 45 hours. If he goes faster by
10 Km/hr from his original speed, he reaches early by 20 hours than the originaltime.
What is the distance he covered byAnkit?
(a)8750Km (b)9750Km (c)1000Km (d) 3750Km

Q490. Three friends A, B and C decide to run around a circular track. They start at the same
time and run in the same direction. A is the quickest and when A finishes a lap, it is
seen that C is as much behind B as B is behind A. When A completes 3 laps, C is the
exact same position on the circular track as B was when A finished 1 lap. What is the
ratio of the speeds of A, B and C?
(a) 5 : 4:2 (b) 4 : 3:2 (c) 5 : 4:3 (d) 3 : 2 :1

GENERAL APTITUDE PAGE 123


Answer: C
Solution:LettracklengthbeequaltoT.WhenAcompletesalap,letusassumeBhas run a
distance of (t - d). At this time, C should have run a distance of (t - 2d). After 3
lapsCisinthesamepositionasBwasattheendofonelap.
So, the position after 3t - 6d should be the same as t - d. Or,
C should be at a distance of d from the end of the lap.
C will have completed less than 3 laps (as he is slower than A), so he could have
traveled a distance of either t - d or 2t - d.
=> 3t - 6d = t - d
=> 2t = 5d
=> d = 0.4t
The distances covered by A, B and C when A completes a lap will be t, 0.6t and 0.2t
respectively. Or, the ratio of their speeds is 5 : 3 : 1.
In the second scenario, 3t - 6d = 2t - d => t = 5d => d = 0.2t.
The distances covered by A, B and C when A completes a lap will be t, 0.8t and 0.6t
respectively. Or, the ratio of their speeds is 5 : 4 : 3.
The ratio of the speeds of A, B and C is either 5 : 3 : 1 or 5 : 4 : 3.

Q491. A,B,C,andDaremovingonacirculartrackinthesamedirection.Allstartedrunning
simultaneously and from the same point such that the ratio of their speeds is 1 : 2 : 3:
4. At how many distinct points any two of themwillmeet?

Answer:4
Solution:
Therewouldbe4C2=6differentcombinationtakingtwoatatime.Let‟stakethemone byone.
A andB:
Thespeedsareinthesimplestformandnumberofdistinctpointstheymeet=2-1=1. A andC:
The speeds are in the simplest form and number of distinct points they meet = 3-1 = 2.
A and D:

GENERAL APTITUDE PAGE 124


Thespeedsareinthesimplestformandnumberofdistinctpointstheymeet=4-1=3. B andC:
Thespeedsareinthesimplestformandnumberofdistinctpointstheymeet=3-2=1. B andD:
The speeds are NOT in the simplest form. To reduce the same into simplest form, we
will take their simplest ratio. i.e. 2 : 4 = 1:2. Therefore, number of distinct points they
meet = 2-1 = 1.
And finally C and D:
Thespeedsareinthesimplestformandnumberofdistinctpointstheymeet=4-3=1. We
know that the last meeting point will always be at the starting point. So when any
twoofthemmeetonlyforonetimethatwillbeatthestartingpointonly.
Whentheymeetforthetwotimes,onewillbeatthestartingpointandotheratapoint
diametricallyoppositetostartingpoint.Similarly,whentheymeetforthreetimes,one
willbeatthestartingpointandresttwopointswillbeat120degreeswiththestarting point.
Hence in total, they will meet at FOUR distinct points.

Q492. Two persons A and B are racing along a circular track. The speed of A is twice the
speedofB.Thelengthofthecirculartrackis800mandthelengthoftheraceis7200
m. After the start of the race, A meets B for the first time at the end of the third
minute. If A and B start the race from the same point, What is the time taken by A to
finish the race?
(a)19minute (b)9.5minute (c)27minute (d)13.5minute
Answer:D
Solution:

GENERAL APTITUDE PAGE 125


Q493. Aboyagreestoworkattherateofonerupeeonthefirstday,tworupeesonthesecond day, and
four rupees on third day and so on. How much will the boy get if he started
workingonthe1stofFebruaryandfinishesonthe20thofFebruary?
(a)220 (b)220-1 (c)219-1 (d)219

Q494. A house has two wall clocks, one in kitchen and one more in the bedroom. The time
displayed on both the watches is 12.A.M right now. The clock in the bedroom gains
fiveminuteseveryhour,whereastheoneinthekitchenisslowerbyfiveminutesevery hour.
After how many hours the watches show the same timeagain?
(a)24hours (b)48hours (c)72hours (d) 96
hoursAnswer:C
Solution: The faster clock runs 5 minutes faster in 1 hr.
The slower clock runs 5 minutes slower in 1 hr.
Therefore, in 1 hour, the faster clock will trace 5+5=10 min more when compared to
the slower clock.
The table given below depicts the time difference between the slower and faster
clock:

GENERAL APTITUDE PAGE 126


In 6 hours, the faster clock will trace 10×6=60 minutes (an hour) more when
compared to the slower clock.
In 6 × 12 = 72 hours, the faster clock will trace an hour more when compared to the
slower clock since the quicker clock determines 12 hours more than, the slower clock.
At this point, both the clocks will show the same time. i.e., both the clocks will show
the same time after exactly 72 hours (or 3 days).
Q495. Aparna was born on 29th, Feb 2016 which happened to be a Monday. If she lives to
be till 2099, how many birthdays would she celebrate on a Monday?
(a)1 (b)2 (c)3 (d)5
Answer: B
Solution:
29th Feb, 2016 = Monday => 28th Feb, 2012 = Sunday
28thFeb,2017=Tuesday(because2016isaleapyear,therewillbe2odddays)
Therefore»Feb28th2018(Wednesday),Feb28th2019(Thursday),Feb28th2020
(Friday), Feb 29th 2020(Saturday)
Or, Feb 29th to Feb 29th after 4 years, we have 5 odd days.
So, every subsequent birthday, would come after 5 odd days.
2020 birthday – 5 odddays
2024 birthday – 10 odd days = 3 odddays
2028 birthday – 8 odd days = 1 odd day
2032 birthday – 6 odddays
2036 birthday – 11 odd days = 4 odd days
2040 birthday – 9 odd days = 2 odd days
2044 birthday – 7 odd days = 0 odd days. So, after 28 years, his birthday would fall
on Monday.
The next birthday on Monday would be in year 2072 (further 28 years later), the one
after that would be in year 2100. But we are told that she lives upto year 2099.
So, there are 2 occurrences of his birthday falling on Monday – 2044 & 2072.

GENERAL APTITUDE PAGE 127


Q496. If a𝑎≠𝛽but𝛼2=5𝛼−3and2 =5-3,thentheequationwhoserootsarea/and /a, is
(a) 3x2 - 25x + 3 = 0
(b) x2 + 5x – 3 = 0
(c) x2 - 5x + 3 = 0
(d) 3x2 - 19x + 3 =0
Answer: D
Solution:

Q497. The vlaue of 𝛼 for which one root of the quadratic equation
(𝛼2−5𝛼+3)𝑥2+(3𝛼−1)𝑥+2=0,istwiceaslargeastheotheris (a)-2/3
(b)1/3
(c)-1/3

GENERAL APTITUDE PAGE 128


(d) 2/3
Answer: D
Solution:

Q498. If p and q are roots of the equation x2+px+q=0 , then


(a)p=1, q=−2
(b)p=0, q=1
(c)p=−2, q=0
(d) p=−2, q=1
Answer: A
Solution:

GENERAL APTITUDE PAGE 129


Q499. If the sum of the roots of the quadratic equation ax2+bx+c=0 is equal to the sum of
𝑏
thesquaresoftheirreciprocalsthen𝑎, ,and𝑐 ,arein.
𝑐𝑎 𝑏

(a)Geometric progression
(b)Harmonic progression
(c)Arithmetic - geometric progression
(d)Arithmetic progression
Answer: B
Solution:

GENERAL APTITUDE PAGE 130


Q500. [MSQ]

If a1,a2,a3 ............... anareinAP,whereai>0foralli,whichofthefollowingstatement

Is/are true?
1 1 1 (𝑛+1)
(a) + +….,+ =
√𝑎1+√𝑎2 √𝑎2+√𝑎3 √𝑎𝑛−1+√𝑎𝑛 √𝑎1+√𝑎𝑛
1 1 1 (𝑛−1)
(b) + +….,+ =
√𝑎1+√𝑎2 √𝑎2+√𝑎3 √𝑎𝑛−1+√𝑎𝑛 √𝑎1+√𝑎𝑛

(c) 𝑎𝑛 + 𝑎1 = 𝑎3 + 𝑎𝑛−3 (d) 𝑎𝑛−5 −


𝑎𝑛−2 = 𝑎3 − 𝑎6 Answer: b, d

Q501. [MSQ]
Which of the following statement is/are true?
𝑦
(a) If𝑥√𝑎= 𝑏=√𝑧 𝑐andifa,b,careinG.P,thenx,y,zareinA.P.

(b) IfqthandrthtermsofanA.Pandbea,bandcrespectivelythen
𝑎 𝑏−𝑐 .𝑏 𝑐−𝑎 .𝑐 𝑎−𝑏=1.
(c) If𝑎 𝑥=𝑏 𝑦=𝑐 𝑧 andx,y,zareinG.P.,thenlog 𝑏𝑎=log𝑏.
𝑐

(d) If a,bandcareinA.P.and𝑎 2 ,𝑏 2and𝑐 2areinH.P.,theneithera=b=cor a, b and –c/2 are


inG.P.
Answer: a, b, c, d

GENERAL APTITUDE PAGE 131


Q502. Consider a small unit of a factory where there are 5 employees : a supervisor and
four labourers. The workers earn a salary of Rs. 5,000 per month each while the
supervisor gets Rs. 15,000 per month. What is relation between the mean, median
and mode of the salaries?
(a)Mean = Median = Mode
(b) Mean <Medin =Mode
(c) Mean > Median =Mode
(d) Mean > Median
>ModeAnswer:C
Solution:
Mean = (5000 + 5000 + 5000 + 5000 + 15000)/5 = 35000/5 = 7000
So, the mean salary is Rs. 7000 per month
To obtain the median, let us arrange the salaries in ascending order:
5000, 5000, 5000, 5000, 15000
Median = (n+1)/2 = (5+1)/2 = 6/2 = 3rd observation
Median=Rs.5000/-
Mode = Number of times an observation is repeated = Rs.5000/-
Here, Mean > Median = Mode.
So, correct answer is C.

Directionsfornextfivequestions:Followingbarchartrepresentsthenumberofpeoplein6
differentvillages(A,B,C,D,EandF)andthetabularcolumndepictstheratioofliterateto
illiteratepeopleandpercentageofmalelivinginthosevillages.

GENERAL APTITUDE PAGE 132


Q503. If40%ofthefemalefromvillageBisliterate,thenwhatisthepercentageofmale, who is
illiterate from villageB?
(a)38% (b)35% (c)37% (d) None ofthese
Answer: C
Solution:

Q504. What is the percentage of literate people in all the six villages together?
(a)55% (b)53% (c)51% (d)None of
theseAnswer:B
Solution:

Q505. What is the ratio between numbers of illiterate people from villages B, C & D to
number of female from villages A, E & F?
(a) 320: 527 (b) 527 : 330 (c) 330:527 (d) 527 :320
Answer: C
Solution:

GENERAL APTITUDE PAGE 133


Q506. If3%offemalefromvillageD&5%offemalefromvillageEareliteratethenwhatis
thetotalnumberofliteratemalefromD&Ftogether?
(a)1823 (b)1723 (c)1623 (d) Cannotdetermine
Answer: D
Solution: Since we don‟t have enough data for village F, hence we cannot determine
the answer.

Q507. The number of female from villages A & C is how much percentage more or less
than number of female from villages D & F?
(a)24.76% (b)24.72% (c) 25.76% (d)25.72%
Answer: A
Solution:

Directions for next five questions:


FollowingPiechartsrepresentthenumberofstudents,whohaveappearedandcleared
phaseIofCivilServiceExamfromvariousdepartmentsofanArtsandScienceCollege.

GENERAL APTITUDE PAGE 134


Q508. Approximately what is the difference between the number of students cleared phase
I from Maths department and number of students appeared from English
department?
(a)1350 (b)1240 (c) 1340 (d)1250
Answer: C
Solution:

Q509. The total number of students clearing phase I from commerce and other department
is what percentage of the number of students clearing phase I from physics
department?
(a) 340 %

GENERAL APTITUDE PAGE 135


(b) 310%
(c) 320%
(d) 330%
Answer: D
Solution:

Q510. WhathastheratiobetweennumbersofstudentsappearedfromMathsandphysics
departments to the number of students appeared from chemistry and commerce
departments?
(a) 16 :15
(b) 15 :17
(c) 16 :17
(d) 15 : 16
Answer: C
Solution:

Q511. From which department is the difference between the number of students cleared
and the number of students appeared is the second minimum?
(a) Chemistry
(b) Physics
(c) Commerce
(d) Maths
Answer: A
Solution:

GENERAL APTITUDE PAGE 136


Q512. What is the percentage of students, who did not clear the phase I of the exam?
(a) 84.45%
(b)84.55%
(c) 83.55%
(d)83.45%
Answer:B

GENERAL APTITUDE PAGE 137


Solution:

Directionsfornextfivequestions:Thefollowingpiechartshowsthedistancetravelledby
boatondifferentdays.Thespeedofthestreamissameinbothupstreamanddownstream
direction. The table shows the speed of stream at different days. Study the data given
carefullyandanswerthequestionsbasedonit.

Q513. On Wednesday the difference between speeds of the boat going downstream and
upstreamis7km/hrandalsotheratioofthespeedofboatgoingdownstreamand
upstream11:4.Ifthetotaltimetakenbyboattocoverdownstreamandupstream
distancesonWednesdayis26hours36minutes.Whatisthespeedofstreamonthat day?
(a)1kmph (b)1.5kmph (c)2kmph (d) 2.5kmph
Answer: A
Solution:
let speed of stream = v

GENERAL APTITUDE PAGE 138


Ratio of upstream: ratio of downstream= 4:11
Difference = 7km/hr
So upstream speed = 4
downstream speed=11
Now, {22% of 360/ (11+v) + 20%of 300/(4-v)}= 26.6
On solving we get v= 1km/hr
So option A is correct.

Q514. IfthetotalspeedofaboatinstillwateronSundayandTuesdayis12.5km/hrand
alsotimetakentocoverthedownstreamdistanceonSundayisequaltothetime
takentocovertheupstreamdistanceonTuesday.Findtheratioofspeedinstill water on
Sunday andTuesday.
(a) 9: 16 (b) 16: 17 (c) 16:9 (d) 19 : 16
Answer: C
Solution: Let speed of boat on Sunday =u
Speed of boat on Tuesday = v
(u + v)=12.5km/hr
According to question,
(v+2)/(u-2.5)=(15%of360)/(18%of300)
(Time same so ratio of speed = ratio of distance)
v + 2 = u - 2.5
u – v = 4.5
u = 8, v = 4.5
Therefore u : v = 16 : 9
Option C is correct.

Q515. On Thursday the boat takes total 30 hours to cover upstream and downstream
distances.Iftheratioofthespeedoftheboatinstillwatergoingdownstreamand
upstreamis5:3.Findthespeedoftheboatinstillwaterwhilegoinginanupstream
direction.
(a) 8km/hr
(b) 12km/hr
(c) 6km/hr
(d) 19km/hr
Answer: C
Solution:
(25%of360)/(5x+2)+(30%of300)/(3x-2)=30

GENERAL APTITUDE PAGE 139


 90/ (5x+2) +90/ (3x-2) = 30
 X=2
Hence speed in upstream = 3 * 2 = 6km/hr
Option C is correct.

Q516. TimetakentocovertheupstreamdistanceonMondayis8hoursmorethanthetime
takentocoverthedownstreamdistanceonThursdayandtotalspeedinstillwater on
Monday and Thursday is 23 km/hr. What is the ratio of speed in still water on
Monday andThursday?
(a) 5 :18
(b) 7 :16
(c) 10 : 13
(d) 11 : 12
Answer:B
Solution: {20% of 300/ (u1-2) – 20% of 360/ (u2+2)} =8
u1+u2 =23
On solving u1= 7km/hr, u2 =
16km/hrSo option B is correct.
Q517. OnSundaytheratioofthespeedoftheboatinstillwatergoingupstreamand downstream
is 5:7. The difference between the time to cover upstream and
downstreamdistanceis6hours.Findthetotaltimetocovertheupstreamand
downstreamdistance.
(a) 6hours
(b) 12hours
(c) 18hours
(d) 15hours
Answer: C
Solution:
{12% of 300/(5x-2) – 15%of 360/(7x+2) }= 6
On solving x=1
Total time= 12+6= 18 hours
So option C is correct.

GENERAL APTITUDE PAGE 140


Data for next five questions:

Q518. WhatistheincreaseinthenumberofCOVID-19casesinDelhiandRajasthan together from


Monday toWednesday?
(a)250 (b)230 (c) 225 (d)180
Answer: B
Solution:

GENERAL APTITUDE PAGE 141


From common explanation, we have
Total cases in Delhi and Rajasthan on Monday = 30 +10 = 40
TotalcasesinDelhiandRajasthanonWednesday=108+162=270
Increase = 270 – 40 =230
Hence, option B is correct.
Q519. The number of cases in UP on Thursday is what percent of the number of cases in
Delhi on Friday?
(a)9.33% (b)16.67% (c) 15% (d)12.5%
Answer: D
Solution:
From common explanation, we have
Number of cases in UP on Thursday = 144
NumberofcasesinDelhionFriday=1152,
Required %=144×100=12.5%
152
Hence, option D is correct.
Q520. What is the difference between the number of cases in Delhi and Rajasthan on
Friday?
(a)724 (b)792 (c) 1080 (d)856
Answer: B
Solution:
From common explanation, we have
Required difference = 1944 – 1152 = 792
Hence, option B is correct.

GENERAL APTITUDE PAGE 142


Q521. WhatistheratiooftheincreaseinthenumberofCOVID-19casesfromTuesdayto Thursday
in UP to that inDelhi?
(a) 5: 21 (b) 7: 20 (c) 11:25 (d) 5 :18
Answer: A
Solution:
From common explanation, we have
From Tuesday to Thursday,
IncreaseinDelhi=288–36=252
Increase in UP = 144 – 84 = 60
Required Ratio = 60 : 252 = 5 : 21
Hence, option A is correct.

Q522. The percentage increase in UP from Wednesday to Thursday is what percent of the
percentage increase in Delhi from Tuesday to Wednesday?
(a)50% (b)25% (c) 45% (d)30%
Answer: D
Solution:
From common explanation, we have
44−90
Percentage increase in UP from Wednesday to Thursday = × 100 = 60%
90
Percentage increase in Delhi from Tuesday to Wednesday
108−36
= × 100 = 200%
36
Required % = 60 /200× 100 = 30%

Data for next five questions:


The chart given below shows the number of positive COVID-19 cases reported in four
countries and percentage of people who died and those who recovered from the reported
cases. Rest of them are active cases.

GENERAL APTITUDE PAGE 143


Q523. What is the difference between the death count in Italy and Spain?
(a)890 (b)840 (c) 825 (d)850
Answer: D
Solution:

From common
explanation, we have
Requireddifference=3600–2750=850
Hence, option D iscorrect.

Q524. What is the total number of active cases in Spain?


(a)16560 (b)13750 (c) 17260 (d)16860
Answer: A
Solution:
From common explanation, we have
Active cases in Spain = 16560
Hence, option A is correct.

Q525. What is the total number of deaths in four countries?


(a)10012 (b)9457 (c) 9324 (d)9487
Answer: B

GENERAL APTITUDE PAGE 144


Solution:
From common explanation, we have
Required sum = 2750 + 3600 + 3075 + 32 = 9457
Hence, option B is correct.
Q526. If25%oftheactivecasesinIranarefemales,whatisthenumberofactivemale cases
inIran?
(a)7250 (b)7380 (c) 6450 (d)7460
Answer: B
Solution:

Q527. The number of people who recovered in India are what percent of the people who
recovered in Spain?
(a) 5.83%
(b)7.14%
(c) 8.25%
(d) 6.67%
Answer: D
Solution:

SPATIAL APTITUDE
SERIES
Q1. Select a figure from amongst the Answer Figures which will continue the same series
as established by the five Problem Figures.

GENERAL APTITUDE PAGE 145


(a)1 (b)2 (c)3 (d)4

Q2. Select a figure from amongst the Answer Figures which will continue the same series
as established by the five Problem Figures.

(a) 1

(b) 2

(c) 3

(d) 4

Q3. Select a figure from amongst the Answer Figures which will continue the same series
as established by the five Problem Figures.

GENERAL APTITUDE PAGE 146


(a)1 (b)2 (c)3 (d)4

Q4. Select a figure from amongst the Answer Figures which will continue the same series
as established by the five Problem Figures.

(a)1 (b)2 (c)3 (d)4

Q5. Select a figure from amongst the Answer Figures which will continue the same series
as established by the five Problem Figures

(a)1 (b)5 (c)3 (d)4

Q6. Select a figure from amongst the Answer Figures which will continue the same series
as established by the five Problem Figures.

GENERAL APTITUDE PAGE 147


(a)1 (b)2 (c)3 (d)4

Q7. Select a figure from amongst the Answer Figures which will continue the same series
as established by the five Problem Figures.

(a)1 (b)2 (c)3 (d)4

Q8. Select a figure from amongst the Answer Figures which will continue the same series
as established by the five Problem Figures.

(a)1 (b)2 (c)3 (d)4

Q9. Select a figure from amongst the Answer Figures which will continue the same series
as established by the five Problem Figures.

GENERAL APTITUDE PAGE 148


(a)1 (b)2 (c)3 (d)4
Q10. Select a figure from amongst the Answer Figures which will continue the same series
as established by the five Problem Figures.

(a)1 (b)2 (c)3 (d)4


Q11. Select a figure from amongst the Answer Figures which will continue the same series
as established by the five Problem Figures.

(a)1 (b)2 (c)3 (d)4


ANALYTICAL REASONING

GENERAL APTITUDE PAGE 149


Q12. Find the number of triangles in the given figure.

(a)8 (b)10 (c)12 (d)14


Q13. Find the number of triangles in the given figure.

(a)12 (b)18 (c)22 (d)26


Q14. Find the minimum number of straight lines required to make the given figure.

(a)13 (b)15 (c)17 (d)19


Q15. Find the number of triangles in the given figure.

(a)15 (b)16 (c)17 (d)18


Q16. Count the number of squares in the given figure.

(a)32 (b)30 (c)29 (d)28


Q17. Count the number of triangles and squares in the given figure.

(a) 28 triangles,3squares (b) 24 triangles, 5squares


(c) 28 triangles,5squares (d)24 triangles, 3squares

GENERAL APTITUDE PAGE 150


Q18. Count the number of parallelogram in the given figure.

(a) 20
(b)18
(c)16
(d)12
Q19. Count the number of triangles and squares in the given figure.

(a) 28 triangles, 10squares


(b) 28 triangles, 8squares
(c) 32 triangles, 10squares
(d)32 triangles, 8squares

GENERAL APTITUDE PAGE 151


PAPER FOLDING
Directions :
In each of the following problems, a square transparent sheet (X) with a pattern is given.
Figure out from amongst the four alternatives as to how the patter would appear when the
transparent sheet is folded at the dotted line.
Q20. Find out from amongst the four alternatives as to how the pattern would appear
when the transparent sheet is folded at the dotted line.

(a)1 (b)2 (c)3 (d)4


Q21. Find out from amongst the four alternatives as to how the pattern would appear
when the transparent sheet is folded at the dotted line.

(a)1 (b)2 (c)3 (d)4

Q22. Find out from amongst the four alternatives as to how the pattern would appear
when the transparent sheet is folded at the dotted line.

(a)1 (b)2 (c)3 (d)4

GENERAL APTITUDE PAGE 152


Q23. Find out from amongst the four alternatives as to how the pattern would appear
when the transparent sheet is folded at the dotted line.

(a)1 (b)2 (c)3 (d)4

Q24. Find out from amongst the four alternatives as to how the pattern would appear
when the transparent sheet is folded at the dotted line.

(a)1 (b)2 (c)3 (d)4


Q25. Find out from amongst the four alternatives as to how the pattern would appear
when the transparent sheet is folded at the dotted line.

(a)1 (b)2 (c)3 (d)4

Q26. Find out from amongst the four alternatives as to how the pattern would appear
when the transparent sheet is folded at the dotted line.

(a)1 (b)2 (c)3 (d)4

GENERAL APTITUDE PAGE 153


Q27. Find out from amongst the four alternatives as to how the pattern would appear
when the transparent sheet is folded at the dotted line.

(a)1 (b)2 (c)3 (d)4

Q28. Find out from amongst the four alternatives as to how the pattern would appear
when the transparent sheet is folded at the dotted line.

(a)1 (b)2 (c)3 (d)4


Q29. Find out from amongst the four alternatives as to how the pattern would appear
when the transparent sheet is folded at the dotted line.

(a)1 (b)2 (c)3 (d)4


Q30. Find out from amongst the four alternatives as to how the pattern would appear
when the transparent sheet is folded at the dotted line.

(a)1 (b)2 (c)3 (d)4

GENERAL APTITUDE PAGE 154


Q31. Find out from amongst the four alternatives as to how the pattern would appear
when the transparent sheet is folded at the dotted line.

(a)1 (b)2 (c)3 (d)4


Q32. Find out from amongst the four alternatives as to how the pattern would appear
when the transparent sheet is folded at the dotted line.

(a) 1

(b) 2

(c) 3

(d) 4

Q33. Find out from amongst the four alternatives as to how the pattern would appear
when the transparent sheet is folded at the dotted line.

(a) 1

(b) 2

(c) 3

(d) 4

GENERAL APTITUDE PAGE 155


Q34. Find out from amongst the four alternatives as to how the pattern would appear
when the transparent sheet is folded at the dotted line.

(a) 1

(b) 2

(c) 3

(d) 4

Q35. Find out from amongst the four alternatives as to how the pattern would appear
when the transparent sheet is folded at the dotted line.

(a) 1

(b) 2

(c) 3

(d) 4

GENERAL APTITUDE PAGE 156


GROUPING OF IMAGES
Directions to Solve:
In each of the following questions, group the given figures into three classes using each
figure only once.
Q36. Group the given figures into three classes using each figure only once.

(a) 1,4 ; 2,3;5,6 (b) 1,5 ; 2,6 ;4,3


(c) 1,6 ; 2,3;4,5 (d)1,2 ; 3,6 ;4,5
Q37. Group the given figures into three classes using each figure only once.

(a) 7,8,9 ; 2,4,3;1,5,6 (b)1,3,2 ; 4,5,7 ;6,8,9


(c)1,6,8 ; 3,4,7;2,5,9 (d)1,6,9 ; 3,4,7 ;2,5,8
Q38. Group the given figures into three classes using each figure only once.

(a) 1,5,7 ; 2,4,6;3,9,8 (b)1,5,7 ; 2,4,8 ;3,6,9


(c) 1,4,7 ; 2,5,8;3,6,9 (d) 1,7,9 ; 3,5,8 ;2,4,6

GENERAL APTITUDE PAGE 157


Q39. Group the given figures into three classes using each figure only once.

(a) 1,3,5 ; 2,6,9 ;4,7,8


(b) 2,3,4 ; 5,6,8 ;9,1,7
(c) 1,3,5 ; 2,6,8 ;4,7,9
(d) 3,2,4 ; 6,5,8 ; 7,9,1

GENERAL APTITUDE PAGE 158


ANALOGY
Figures A, B, C and D constitute the Problem Set while figures 1, 2, 3, 4 and 5 constitute the
Answer Set. There is a definite relationship between figures A and B. Establish a similar
relationship between figures C and D by selecting a suitable figure from the Answer Set that
would replace the question mark (?) in fig. (D).
Q40. Select a suitable figure from the Answer Figures that would replace the question
mark (?).Problem Figures:

(a)1 (b)2 (c)3 (d)4

Q41. Select a suitable figure from the Answer Figures that would replace the question
mark (?).

(a)1 (b)2 (c)3 (d)4

GENERAL APTITUDE PAGE 159


Q42. Select a suitable figure from the Answer Figures that would replace the question
mark (?).

Problem Figures:

(a) 1

(b) 2

(c) 3

(d) 4

Q43. Select a suitable figure from the Answer Figures that would replace the question
mark (?).

(a) 1

(b) 2

(c) 3

(d) 4

GENERAL APTITUDE PAGE 160


Q44. Select a suitable figure from the Answer Figures that would replace the question
mark (?).

(a)1 (b)2 (c)3 (d)5


Q45. Select a suitable figure from the Answer Figures that would replace the question
mark (?).Problem Figures:

(a)1 (b)4 (c)3 (d)5


Q46. Select a suitable figure from the Answer Figures that would replace the question
mark (?).Problem Figures:

(a)1 (b)2 (c)3 (d)4

GENERAL APTITUDE PAGE 161


MIRROR IMAGES
In each of the following questions you are given a combination of alphabets and/or numbers
followed by four alternatives (1), (2), (3) and (4). Choose the alternative which is closely
resembles the mirror image of the given combination
Q47. Choose the alternative which is closely resembles the mirror image of the given
combination.

(a)1 (b)2 (c)3 (d)4


Q48. Choose the alternative which is closely resembles the mirror image of the given
combination.

(a)1 (b)2 (c)3 (d)4


Q49. Choose the alternative which is closely resembles the mirror image of the given
combination.

(a)1 (b)2 (c)3 (d)4


Q50. Choose the alternative which is closely resembles the mirror image of the given
combination.

(a)1 (b)2 (c)3 (d)4

GENERAL APTITUDE PAGE 162


Directions to Solve
In each of the following questions, choose the correct mirror images of the given image of
the Fig.(X) from amongst the four alternatives (1), (2), (3) and (4) given along with it.
Q51. Choose the correct mirror image of the given figure (X) from amongst the four
alternatives.

(a)1 (b)2 (c)3 (d)4

Q52. Choose the correct mirror image of the given figure (X) from amongst the four
alternatives.

(a)1 (b)2 (c)3 (d)4

Q53. Choose the correct mirror image of the given figure (X) from amongst the four
alternatives.

(a)1 (b)2 (c)3 (d)4

GENERAL APTITUDE PAGE 163


Q54. Choose the correct mirror image of the given figure (X) from amongst the four
alternatives.

(a)1 (b)2 (c)3 (d)4


Q55. Choose the correct mirror image of the given figure (X) from amongst the four
alternatives.

(a)1 (b)2 (c)3 (d)4


Q56. Choose the correct mirror image of the given figure (X) from amongst the four
alternatives.

(a)1 (b)2 (c)3 (d)4


Q57. Choose the correct mirror image of the given figure (X) from amongst the four
alternatives.

(a)1 (b)2 (c)3 (d)4

GENERAL APTITUDE PAGE 164


PATTERN COMPLETION
Directions to Solve:
In each of the following questions, select a figure from amongst the four alternatives, which
when placed in the blank space of figure (X) would complete the pattern.
Q58. Identify the figure that completes the pattern.

(a)1 (b)2 (c)3 (d)4

Q59. Identify the figure that completes the pattern.

(a)1 (b)2 (c)3 (d)4

Q60. Identify the figure that completes the pattern.

(a)1 (b)2 (c)3 (d)4

GENERAL APTITUDE PAGE 165


Q61. Identify the figure that completes the pattern.

(a)1 (b)2 (c)3 (d)4


Q62. Identify the figure that completes the pattern.

(a)1 (b)2 (c)3 (d)4


Q63. Identify the figure that completes the pattern.

(a)1 (b)2 (c)3 (d)4


Q64. Identify the figure that completes the pattern.

(a)1 (b)2 (c)3 (d)4


Q65. Identify the figure that completes the pattern.

(a)1 (b)2 (c)3 (d)4

GENERAL APTITUDE PAGE 166


PAPER CUTTING
Directions to Solve:
Each of the following questions consists of a set of three figures X, Y and Z showing a
sequence of folding of a piece of paper. Figure (Z) shows the manner in which the folded
paper has been cut. These three figures are followed by four answer figures from which you
have to choose a figure which would most closely resemble the unfolded form of figure (Z).
Q66. Choose a figure which would most closely resemble the unfolded form of Figure (Z).

(a) 1

(b) 2

(c) 3

(d) 4

Q67. Choose a figure which would most closely resemble the unfolded form of Figure (Z).

(a) 1
(b) 2
(c) 3
(d) 4

GENERAL APTITUDE PAGE 167


Q68. Choose a figure which would most closely resemble the unfolded form of Figure
(Z).

(a) 1
(b) 2
(c) 3
(d) 4

Q69. Choose a figure which would most closely resemble the unfolded form of Figure (Z).

(a) 1
(b) 2
(c) 3
(d) 4

Q70. Choose a figure which would most closely resemble the unfolded form of Figure (Z).

(a) 1
(b) 2
(c) 3
(d) 4

GENERAL APTITUDE PAGE 168


Q71. Choose a figure which would most closely resemble the unfolded form of Figure (Z).

(a)1 (b)2 (c)3 (d)4


Q72. Choose a figure which would most closely resemble the unfolded form of Figure (Z).

(a)1 (b)2 (c)3 (d)4


Q73. Choose a figure which would most closely resemble the unfolded form of Figure (Z)

(a)1 (b)2 (c)3 (d)4


Q74. Choose a figure which would most closely resemble the unfolded form of Figure (Z).

(a)1 (b)2 (c)3 (d)4

GENERAL APTITUDE PAGE 169


Q75. Choose a figure which would most closely resemble the unfolded form of Figure (Z).

(a)1 (b)2 (c)3 (d)4


Q76. Choose a figure which would most closely resemble the unfolded form of Figure (Z).

(a)1 (b)2 (c)3 (d)4


Q77. Choose a figure which would most closely resemble the unfolded form of Figure (Z).

(a)1 (b)2 (c)3 (d)4


Q78. Choose a figure which would most closely resemble the unfolded form of Figure (Z).

(a)1 (b)2 (c)3 (d)4


Q79. Choose a figure which would most closely resemble the unfolded form of Figure (Z).

(a)1 (b)2 (c)3 (d)4

GENERAL APTITUDE PAGE 170


CUBES AND DICE
Directions to Solve:
The sheet of paper shown in the figure (X) given on the left hand side, in each problem, is
foldedtoformabox.Choosefromamongstthealternatives(1),(2),(3)and(4),theboxesthat
aresimilartotheboxthatwillbeformed.
Q80. Choose the box that is similar to the box formed from the given sheet of paper (X).

(a) 1 and2only (b) 2 and 4only


(c) 2 and3only (d) 1 and 4only

Q81. Choose the box that is similar to the box formed from the given sheet of paper (X).

(a) 1 and4only (b) 3 and 4only


(c) 1and2only (d) 2 and 3only
Q82. Choose the box that is similar to the box formed from the given sheet of paper (X).

(a) 1 and2only (b) 1 and 4only


(c) 2 and4only (d) 3 and 4only

GENERAL APTITUDE PAGE 171


Q83. Choose the box that is similar to the box formed from the given sheet of paper (X).

(a) 1 and 2only


(b) 2 and 3only
(c) 2 and 4only
(d) 1, 2, 3 and 4

Q84. Choose the box that is similar to the box formed from the given sheet of paper
(X).Choosetheboxthatissimilartotheboxformedfromthegivensheetofpaper(X).

(a)1only (b)2only (c)3only (d) 4only


Q85. Choose the box that is similar to the box formed from the given sheet of paper (X).

(a) 2 and 3only


(b) 1,3 and 4only
(c) 2 and 4only
(d) 1 and 4only

GENERAL APTITUDE PAGE 172


Q86. Choose the box that is similar to the box formed from the given sheet of paper (X).

(a) 1 and 2only


(b) 1,2 and 4only
(c) 1 and 4only
(d) 1,2 and 3only
Q87. Choose the box that is similar to the box formed from the given sheet of paper (X).

(a) 1only
(b) 2 and 3only
(c) 1 and 3 only
(d)1, 2 and 4only
Q88. When the following figure is folded to form a cube, how many dots lie opposite the
face bearing five dots?

(a)1 (b)2 (c)3 (d)4

GENERAL APTITUDE PAGE 173


Q89. Two positions of a dice are shown. When 4 is at the bottom, what number will be on
the top?

(a) 1

(b) 2

(c) 5

(d) 6

Q90. What will be the number at the bottom, if 5 is at the top; the two positions of the dice
being as given below

(a)1 (b)2 (c)3 (d)6

GENERAL APTITUDE PAGE 174


CLASSIFICATION
Q91. Choose the figure which is different from the rest.

(a)1 (b)2 (c)3 (d)4

Q92. Choose the figure which is different from the rest.

(a)1 (b)2 (c)3 (d)4

Q93. Choose the figure which is different from the rest.

(a)1 (b)2 (c)3 (d)4

Q94. Choose the figure which is different from therest.

(a)1 (b)2 (c)3 (d)4

Q95. Choose the figure which is different from the rest

(a)1 (b)2 (c)3 (d)4

Q96. Choose the figure which is different from the rest.

(a)1 (b)2 (c)3 (d)4

GENERAL APTITUDE PAGE 175


WATER IMAGES

Q97. Choose the alternative which is closely resembles the water-image of the given
combination.

(a)1 (b)2 (c)3 (d)4


Q98. Choose the alternative which is closely resembles the water-image of the given
combination.

(a)1 (b)2 (c)3 (d)4

Ineachofthefollowingquestions,choosethewaterimageoftheFig.(X)fromamongstthe
fouralternatives(1),(2),(3)and(4)givenalongwithit

Q99. Choose the correct water image of the given figure (X) from amongst the four
alternatives.

(a)1 (b)2 (c)3 (d)4


Q100. Choose the correct water image of the given figure (X) from amongst the four
alternatives.

(a)1 (b)2 (c)3 (d)4

Q101. Choose the correct water image of the given figure (X) from amongst the four

alternatives.

(a)1 (b)2 (c)3 (d)4

GENERAL APTITUDE PAGE 176


SHAPE CONSTRUCTION
Q102. SelectafigurefromthegivenfouralternativeswhichfitsexactlyintoFigure-Xto forma

(a)1 (b)2 (c)3 (d)4

Q103. SelectafigurefromthegivenfouralternativeswhichfitsexactlyintoFigure-Xto forma

(a)1 (b)2 (c)3 (d)4

Q104. SelectafigurefromthegivenfouralternativeswhichfitsexactlyintoFigure-Xto forma

(a)1 (b)2 (c)3 (d)4

Q105. SelectafigurefromthegivenfouralternativeswhichfitsexactlyintoFigure-Xto forma

(a) 1
(b) 2
(c) 3
(d) 4

GENERAL APTITUDE PAGE 177


Q106. SelectafigurefromthegivenfouralternativeswhichfitsexactlyintoFigure-Xto forma

(a) 1

(b) 2

(c) 3

(d) 4

Q107. SelectafigurefromthegivenfouralternativeswhichfitsexactlyintoFigure-Xto forma

(a) 1

(b) 2

(c) 3

(d) 4

Q108. SelectafigurefromthegivenfouralternativeswhichfitsexactlyintoFigure-Xto forma‟

(a) 1
(b) 2
(c) 3
(d) 4

GENERAL APTITUDE PAGE 178


VERBAL APTITUDE
SOLVED EXAMPLES

SPOTTING THE ERRORS


Q1. Some nouns are singular in form, but they are used as plural nouns and always take a
plural verb. Cattle, gentry, vermin, peasantry, artillery, people, clergy, company,
police
a) The cattle is grazing inthe ground. (Incorrect)
b) The cattle are grazing intheground. (Correct)
a) The clergy is inthechurch. (Incorrect)
b) The clergy are inthechurch. (Correct)

Q2. Some nouns are always used in a plural form and always take a plural verb. Trousers,
scissors,spectacles,stockings,shorts,measles,goods,alms,premises,thanks,tidings,
annals, chattels,etc
a) Where ismytrousers? (Incorrect)
b) Where aremytrousers? (Correct)
a) Spectacles is now acostlyitem. (Incorrect)
b) Spectacles are now acostlyitem. (Correct)

Q3. Collective nouns such as jury, public, team, committee, government, audience,
orchestra, company, etc. are used both as singular and plural depending on the
Meaning. When these words indicate a unit, the verb is singular, otherwise the verb
will be plural.
a) The jury was divided inthiscase. (Incorrect)
b) The jury were divided inthiscase. (Correct)
a) The team have not comeasyet. (Incorrect)
b) The team has not comeasyet. (Correct)

GENERAL APTITUDE PAGE 179


Q4. People are often confused or they commit mistakes in the use of certain nouns.
1) Lecturership is wrong: lectureship iscorrect.
a) There are twenty candidatesforlecturership. (Incorrect)
b) There are twenty candidatesforlectureship. (Correct)
2) Freeship is wrong; free-studentshipiscorrect. (Incorrect)
a) Ramesh has appliedfor freeship. (Incorrect)
b) Ramesh has appliedforfree-studentship. (Correct)
3) Familymembersiswrong;membersofthefamilyiscorrect.
a) Vivek and Ramesh are myfamilymembers. (Incorrect)
b) Vivek and Ramesh are the members ofmyfamily. (Correct)
4) Englishteacheriswrong;theteacherofEnglishiscorrect.
a) Dr. Raina is ourEnglishteacher. (Incorrect)
b) Dr. Raina is our teacherofEnglish. (Correct)
5) Cousin-brotherorsisteriswrong;onlycousiniscorrect.
a) Geeta is mycousinsister. (Incorrect)
b) Geeta ismycousin. (Correct)
6) Ours, yours, hers, theirs arecorrect.
a) This houseisour‟s. (Incorrect)
b) This houseisours. (Correct)
Note: Thesameprincipleappliesto„yours‟,„hers‟and„theirs‟.

Q5. A pronoun must agree with its antecedent in person, number and gender.
For example:
Every man must bring his luggage.
All students must do their home work.
Each of the girls must carry her own bag.
Each student must bringtheirbooks. (Incorrect)
Each student must bring hisbooks. (Correct)

GENERAL APTITUDE PAGE 180


Q6. While using „everybody‟, „anyone‟, „anybody‟, and „each‟ the pronoun of the
masculine or the feminine
gender is used according to the content.
I shall be happy to help each of the boys in this practice.
But when the sex is not mentioned, we use the pronoun of the masculine gender.
Anyone can do this job if he tries.
Eachofthesixboysintheclasshasfinishedtheirtask. (Incorrect)
Eachofthesixboysintheclasshasfinishedhistask. (Correct)

Q7. The pronoun „one‟ must be followed by „one‟s‟.


One must finish his taskintime. (Incorrect)
One must finish one‟s taskintime. (Correct)

Q8. „Who‟ denotes the subject and „whom‟ is used for the object?
Whom do you think wontheaward? (Incorrect)
Who do you think wontheaward? (Correct)
Who are youtalkingto? (Incorrect)
Whom are youtalkingto? (Correct)

Q9. When two or more singular nouns are joined together by „either or‟; „neither nor‟;and
„or‟, thepronounis singular.
Either Ram or Shyam will givetheirbook. (Incorrect)
EitherRamorShyamwillgivehisbook. (Correct)

Q10. „Whose‟isusedforlivingpersonsand„which‟forlifelessobjects?
Which book did youselect?
Whose photograph is lying there?
What book doyou read? (Incorrect)
Which book doyouread? (Correct)

GENERAL APTITUDE PAGE 181


Q11. „Eachother‟isusedwhentherearetwosubjectsorobjectsand„oneanother‟when there
aremorethan two.
Romeo and Juliet loved each other.
Those five friends, who are sitting there, love one another.
All the students of the class are friendly; they loveeachother. (Incorrect)
All the students of the class are friendly; they loveoneanother. (Correct)

Q12. Whenapronounstandsforacollectivenoun,itmustbeinthesingularnumberand
intheneutergenderifthecollectivenounisviewedasawhole.
The jury gave „its‟ verdict.
Herethe„jury‟givestheideaofonewhole.
If the collective noun conveys the idea of separate individuals comprising the whole,
the pronoun standing for it must beplural.
The jury were divided in their opinions.
Here the „jury‟ gives the idea of several individuals.
The team are divided in this opinion about playingonSunday. (Incorrect)
The team are divided in their opinion about playingonSunday. (Correct)

Q13. If pronoun of different persons are to be used together in a sentence, the serial order
of persons should be as follows: second person + third person + first person in a good
normal sentence. But if a fault is to be confessed, the order will be:
first person + second
person + third person.
You, he and I have finishedthework. (Normalsentence)
I, you and he aretoblame. (Confession)
Ram, I and you have finishedour studies. (Incorrect)
You, Ram and I have finishedourstudies. (Correct)

GENERAL APTITUDE PAGE 182


Q14. „Some‟ is used in affirmative sentences to express quantity or degree. „Any‟ is used in
negative or interrogative sentences.
I shall buy some applies.
I shall not buy any apples.
Have you bought any applies?
But „Some‟ may be correctly used in interrogative sentences which are, in fact,
requests. Will you please give me some milk?
I shall readanybook. (Incorrect)
I shall readsomebook. (Correct)
Have you boughtsomeapples? (Incorrect)
Have you boughtanyapples? (Correct)
Q15. Theuseof„few‟,„afew‟and„thefew‟shouldbeusedwithcare.Theydenote„number‟. „Few‟
means „not many‟. It is the opposite of many. A „few‟ is positive and means
„someatleast‟.Itistheoppositeofnone.„Thefew‟means„whateverthereis‟.
A few men are freefromfault. (Incorrect)
Few men are freefromfault. (Correct)
Here the sense is negative and thus „a few‟ is wrong.
Few boys will pass intheexamination. (Incorrect)
A few boys will pass intheexamination. (Correct)
Herethesenseispositiveandthus„few‟isincorrect.
I have already read a few books that are onthebookshelf. (Incorrect)
I have already read the few books that are onthebookshelf. (Correct)
Here the sense is „whatever thereis‟.

Q16. Use of „less‟ and „fewer‟.


„Less‟ denotes quantity and „fewer‟ denotes number.
No less than fifty personswerekilled. (Incorrect)
No fewer than fifty personswerekilled. (Correct)
There are no fewer than five litres of water inthejug. (Incorrect)
There are no less than five litres of water inthejug. (Correct)

GENERAL APTITUDE PAGE 183


Q17. Use of little, a little, the little.
„Little‟ means „hardly any‟.
There is a little hope ofhisrecovery. (Incorrect)
There is little hope ofhisrecovery. (Correct)
„A little‟ means „some‟, though notmuch.
Little knowledge is adangerousthing. (Incorrect)
A little knowledge is adangerous thing. (Correct)
„Thelittle‟means„notmuchbutallthereis‟.
A little milk that is in the pot may be used forthepatient. (Incorrect)
The little milk that is in the pot may be used forthepatient. (Correct)

Q18. Use of elder, older.


„Older‟ refers to persons as well as things and is followed by „than‟.
Ram is elder than all other boys ofthisarea. (Incorrect)
Ram is older than all other boys ofthisarea. (Correct)
„Elder‟ is used for members of thefamily.
Suresh is myolderbrother. (Incorrect)
Suresh is myelderbrother. (Correct)

Q19. Normally „than‟ is used in the comparative degree, but with words like superior,
Inferior,senior,junior,prior,anterior,posteriorandprefer„to‟isused.Shelleyisjunior
thanWordsworth. (Incorrect)
Shelley is juniortoWordsworth. (Correct)
I prefer readingthansleeping. (Incorrect)
I prefer readingto sleeping. (Correct)

GENERAL APTITUDE PAGE 184


Q20. When a comparison is made by using a comparative followed by „than‟, the word
„other‟mustbeusedtoexcludethethingcomparedfromtheclassofthingswithwhich it
iscompared.
He is stronger thananyman. (Incorrect)
He is stronger than anyotherman. (Correct)
„Any man‟ includes the man himself and thus the sentence will beabsurd.

Q21. „Many a‟ is always followed by the singular verb.


Many a man were drowned inthesea. (Incorrect)
Many a man was drowned inthesea. (Correct)

Q22. If the subject is „the number of‟ the singular verb is used.
The number of students arevery small. (Incorrect)
The number of students isverysmall. (Correct)
Ifthesubjectis„anumberof‟,thepluralverbisused.
A number of booksismissing. (Incorrect)
A number of booksaremissing. (Correct)

Q23. When„aswellas‟,„alongwith‟,„togetherwith‟,„nolessthan‟,„inadditionto‟and„not‟ and


„with‟ join two subjects, the verb will be according to the firstsubject.
Ram, as well as his ten friends,aregoing. (Incorrect)
Ram, as well as his ten friends,isgoing. (Correct)
The teacher, along with the students,weregoing. (Incorrect)
The teacher, along with the students,was going. (Correct)

Q24. „A great many‟ is always followed by a plural noun and a plural verb.
A great many student has beendeclaredsuccessful. (Incorrect)
A great many students have beendeclaredsuccessful. (Correct)

GENERAL APTITUDE PAGE 185


Q25. If two subjects are joined by „either or‟, „neither nor‟, the verb agrees with the subject
that is near.
Either Ramesh or I are to dothiswork. (Incorrect)
Either Ramesh or I am to dothiswork. (Correct)
Neither he nor his friendsisreading. (Incorrect)
Neither he nor his friendsarereading. (Correct)

Q26. When two singular nouns joined by „and‟ point out the same thing or person, the verb
will be singular.
Bread and butter make agoodbreakfast. (Incorrect)
Bread and butter makes agoodbreakfast. (Correct)
The Collector and the District Magistrate areonleave. (Incorrect)
TheCollectorandDistrictMagistrateisonleave. (Correct)

Q27. „One of‟ always takes a plural noun after it.


It is one of the important day inmylife. (Incorrect)
It is one of the important days inmylife. (Correct)

Q28. Use of „not only‟ and „but also‟: Examine the sentences given below.
He not only comes for swimming but also for coachingthelearners. (Incorrect)
He comes not only for swimming but also for coachingthelearners. (Correct)
Here the error is due tomisplacement.
Theexpression„notonly‟mustbeplacedbeforetherightword,otherwisethemeaning
changes. Here this expression is meant for swimming and not forcoming.

Q29. „Scarcely‟ and „hardly‟ are followed by „when‟ and not by „than‟.
I had scarcely entered the room than thephonerang. (Incorrect)
I had scarcely entered the room when thephonerang. (Correct)

GENERAL APTITUDE PAGE 186


Q30. Though‟ is followed by „yet‟ and not by „but‟.
Though he is poor but heishonest. (Incorrect)
Though he is poor, yet heishonest. (Correct)

Q31. „No sooner‟ is followed by „than‟.


No sooner had I entered the class when the studentsstoodup. (Incorrect)
No sooner had I entered the class than the studentsstoodup. (Correct)
Attentionmustalsobegiventotheverbformwhileusing„nosoonerthan‟.Thecorrect forms
are past perfect or pastindefinite.
No sooner had I entered the class than the studentsstandup. (Incorrect)
No sooner had I entered the class than the studentsstoodup. (Correct)

Q32. „Lest‟ must be followed by „should‟.


Read regularly lest youwillfail. (Incorrect)
Read regularly lest youshouldfail. (Correct)

Q33. „Such‟ is followed by „as‟.


He is such a writer that everybody should readhisbooks. (Incorrect)
He is such a writer as everybody should readhisbooks. (Correct)
„Such‟ is also followed by „that‟ if we emphasize the degree of something by
mentioning the result. The extent of the disaster was such that the authorities were
quite unable to cope.

Q34. „Unless‟ expresses a condition. It is always used in the negative sense. Thus „not‟ is
never used with „unless‟.
Unless you do not labour hard, you willnotpass. (Incorrect)
Unless you labour hard, you willnotpass. (Correct)

GENERAL APTITUDE PAGE 187


Q35. Until expresses time. It has a negative sense and thus „not‟ should never be used with
it.
Wait here until I donotreturn. (Incorrect)
Wait here untilI return. (Correct)
Q36. „Since‟ indicates a point of time and „for‟ stands for the length of time.
He has been reading the book sincetwo hours. (Incorrect)
He has been reading the book fortwo hours. (Correct)
Twohoursisalengthoftimeandthus„for‟iscorrect.
It has been raining forMondaylast. (Incorrect)
It has been raining sinceMondaylast. (Correct)
Q37. „As if‟ is used to convey the sense of pretension. When „as if‟ is used in this sense,
„were‟ is used in all cases, even with third person singular.
He behaves as if he wasaking. (Incorrect)
He behaves as if he wereaking. (Correct)
The sense of time and tense is also very important. Certain important rules are listed
below.
Q38. Past tense in the principal clause is followed by a past tense in the subordinateclause.
He saw that the clockhas stopped. (Incorrect)
He saw that the clockhadstopped. (Correct)
There are, however, two exceptions to thisrule:
(1) A past tense in the principal clause may be followed by a present tense in the
subordinate clause, when it expresses a universaltruth.
The teacher observed that the Earth moved roundthe Sun. (Incorrect)
The teacher observed that the Earth moves roundtheSun. (Correct)
(2) Whenthesubordinateclauseisintroducedby„than‟,evenifthereispasttensein the
principal clause, it may be followed by any tense required by the sense in the
Subordinateclause.
He helped him more than he helped hisownchildren. (Incorrect)
He helped him more than he helps hisownchildren. (Correct)

GENERAL APTITUDE PAGE 188


Q39. If two actions in a sentence are shown happening in the past, one after the other, the
tense of the action happening first should be past perfect and that of the second should
be past indefinite.
The patient died before thedoctorarrived. (Incorrect)
The patient had died before thedoctorarrived. (Correct)

Q40. Two actions in the past, one depending on the other, should have the sequence as
follows:
Past perfect + Future perfect
Had + past participle + would + have + past participle
If you had worked hard, you would have succeeded in the examination.
Or
Had you worked hard, you would have succeeded in the examination.
If you would have practised regularly, you would winthematch. (Incorrect)
If you had practised regularly, you would have wonthematch. (Correct)

Q41. If, in a sentence, two actions are indicated and both are to take place in future, the
sequence of tenses will be as follows:
The principal clause in present indefinite; and the subordinate clause in future
indefinite.
If I go to Delhi, I shall attend the seminar.
„If I go to Delhi‟ is the principal clause and „I shall attend the seminar‟ is the
subordinate clause.
If it will rain, I shall not attendthemeeting. (Incorrect)
If it rains, I shall not attendthemeeting. (Correct)

Q42. When an action has taken place in two clauses of a sentence, it is used in both the
clauses according to the requirement.
My brother has and is still doing excellent work forhisorganisation. (Incorrect)
My brother has done and is still doing excellent work for his organisation. (Correct)

GENERAL APTITUDE PAGE 189


Q43. When there is a sense of continuity, that is, when a thing has taken place in the past
and still continues in the present, the perfect continuous tense form of the verbshould
beused.
India is independent for the lastforty-six years. (Incorrect)
India has been independent for the lastforty-sixyears. (Correct)
The use of articles is also an important matter and one must be careful about it.

Q44. Before a consonant „a‟ is used.


A boy, a horse, a woman.
But„a‟isalsousedbeforewordslikeuniversity,useful,unicorn,union,European,and
one,etc.,becausethesewordsbeginwithaconsonantsound.
Here isanuniversity. (Incorrect)
Here isauniversity. (Correct)
An European lives inmyarea. (Incorrect)
A European lives inmyarea. (Correct)
It is anone-rupeenote. (Incorrect)
It is aone-rupeenote. (Correct)

Q45. Similarly, words like „hour‟, „honest‟, „heir‟, etc. take „an‟ before them as they begin
with a vowel sound.
I have been waiting for him forahour. (Incorrect)
I have been waiting for him foranhour. (Correct)

Q46. Some important points to remember regarding the omission of a/an/the


1. Before a common noun used in its widest sense,e.g.,
Man ismortal.
2. Before names ofmaterials.
Bronze is a usefulmetal.
3. Usually before proper names.
Patna is the capital ofBihar.
4. Before abstract nouns used in general sense.
Beauty fascinatespeople.
5. Beforelanguages.
English is a very popular language in the Delhi University.
6. Before„school‟,„college‟,„church‟,„bed‟,„table‟,„hospital‟,„market‟,and„prison‟,
When these places are visited or used for their common purpose. My Christian
Friends go to church every Sunday. But „the‟ is used with these words whenwe

GENERAL APTITUDE PAGE 190


refertothemasadefiniteplace,buildingorobjectratherthantothenormal
Activity that goes onthere.
I met my friend at the church.
7. Beforenamesofrelations,like„father‟,„mother‟,„aunt‟,„uncle‟,„cook‟and„nurse‟.
Father is very happytoday.
8. Before predicative nouns denoting a unique position that is normally held at one
time by one persononly.
He was elected chairman of the Board.

Q47. Uses of „the‟.


1. When the object isunique
The earth, the sky, the equator
2. Beforesuperlatives
The best, the finest, the most
3. With proper nouns like „seas‟, „rivers‟, „group of islands‟, „chains ofmountains‟,
„deserts‟,„newspapers‟,„buildings‟,„religiousbooks‟,„gulfs‟.
TheArabiansea,TheGanges,TheTajMahal,
The Times of India, TheRamayana.
4. Before a proper noun, when it is qualified by anadjective.
The immortalKalidas.
5. Before an adjective, when the noun is hidden.
The rich should be kind andhelping.
6. Before musicalinstruments.
I know how to play the harmonium.
7. With a class of things.
The crow is a cleverbird.
8. „The‟ should be used before both comparative degrees when they are used in a
sentence forproportion.
Higher we go, coolerwefeel. (Incorrect)
The higher we go, thecooler wefeel. (Correct)

Q48. „What to speak of‟ is incorrect; the correct expression is „not to speak of‟.
What to speak of running, he cannotevenwalk. (Incorrect)
Not to speak of running, he cannotevenwalk. (Correct)

GENERAL APTITUDE PAGE 191


Q49. When there is the sense of dislike, hesitation, risk, etc., in a sentence, we should use a
gerund instead of a verb. A gerund is that form of the verb which ends in „ing‟ and
can be used in the place of a noun.
I dislike to see a film late inthenight. (Incorrect)
I dislike seeing a film late inthenight. (Correct)
Young girls hesitate to talktostrangers. (Incorrect)
Young girls hesitate talkingtostrangers. (Correct)

Q50. „Cent per cent‟ and „word by word‟ are wrong. „Hundred per cent‟ and „word for
word‟ are correct expressions.
You are never cent per cent sure of your success in a competitive examination.
(Incorrect)
You are never hundred per cent sure of your success in a competitive examination.
(Correct)
I can reproduce this lesson word by word. (Incorrect)
I can reproduce this lesson word for word.(Correct)

Q51. „Since‟,„because‟,„as‟and„for‟–allmean„because‟,butthereisadifferenceintheir
degree.„Since‟and„because‟areusedforstrongercasesand„as‟and„for‟forweak cases.
I respect him as he ismyteacher. (Incorrect)
I respect him because he ismyteacher. (Correct)
I could not attend the meeting because it was veryhottoday. (Incorrect)
I could not attend the meeting as it was veryhottoday. (Correct)

Q52. Use of „when‟ and „while‟: Proper attention must be paid to these words. „When‟
indicatesageneralsenseand„while‟impliesatimeduringtheprocessofdoingawork. When
learning to swim, one of the most important things istorelax. (Incorrect)
While learning to swim, one of the most important things istorelax. (Correct)

GENERAL APTITUDE PAGE 192


Q53. Sometimestheerrorliesintheuseofwords.Properattentionmust,therefore,begiven to the
appropriateness ofwords.
Prakash was leading a happy and leisurely life after his retirement from his
service. (Incorrect)
Prakash was living a happy and leisurely life after his retirement from his
service. (Correct)
Here the word „leading‟ is inappropriate.

Q54. Proper, abstract and material nouns have no plural except when they are used as
common nouns.
The house is builtofbricks. (Incorrect)
The house is builtofbrick. (Correct)
When such nouns are used in the plural, they become common nouns with changed
meanings; as
Coppers =Coppercoins; Irons =fetters.
Tins = cans madeoftin; Woods =forest.
Helivesinthewoods.Itmeanshelivesinaforest.

Q55. The relative pronoun should be placed as near as possible to its antecedent so that no
ambiguity arises.
The boy is my cousin who stood first in themilerace. (Incorrect)
Theboy,whostoodfirstinthemilerace,ismycousin. (Correct)

Q56. When two singular nouns joined by „and‟ are preceded by „each‟ or „every‟ the
pronoun used for them is singular.
Each man and each boy is responsible fortheiraction. (Incorrect)
Each man and each boy is responsible forhisaction. (Correct)

Q57. Sometimes a pronoun is used where it is not required at all.


He being an M.A., he is qualified forthepost. (Incorrect)
He, being an M.A., is qualified forthepost. (Correct)

GENERAL APTITUDE PAGE 193


Q58. If a pronoun comes after a preposition it should be used in the objective case.
Between you and I neither of usiscorrect. (Incorrect)
Between you and me neither of usiscorrect. (Correct

Q59. When a pronoun comes after „like‟ and „unlike‟ it takes an objective case.
A man like I will notdoit. (Incorrect)
A man like me will notdoit. (Correct)
Q60. A pronoun takes an objective case after „let‟.
Let Idoit. (Incorrect)
Let medoit. (Correct)

Q61. Pronouns joined by „and‟ remain in the same case:


He and heraresister. (Correct)
He and me are friends. Brother andsister. (Incorrect)
He and she arebrotherand (Incorrect)
He and Iare friends. (Correct)

Q62. When pronouns of second person and third person are used as subjects, the pronoun
Following them will be according to the second person pronoun.
You and he must bringhisbooks. (Incorrect)
You and he must bringyourbooks. (Correct)

Q63. When pronouns of second person and first person are used as subjects, the pronoun
following them will be first person plural form.
You and I must finish your workintime. (Incorrect)
You and I must finish our workintime. (Correct)

Q64. „But‟ is also used as a relative pronoun. When a sentence has a negative noun or
pronoun, „but‟ can be used with it. In this case „but‟ means: who not; that not.
Here was none but laughed to see the joker.
Here the meaning of „but laughed‟ is „who did not laugh.‟
There is no bird but flies.
„But flies‟ means „that does not fly‟.

GENERAL APTITUDE PAGE 194


Q65. Therelativepronoun„that‟isusedinpreferenceto„who‟or„which‟afteradjectivesin the
superlativedegree.
The wisest man who ever livedmademistakes. (Incorrect)
The wisest man that ever livedmade mistakes. (Correct)
This is the best which wecando. (Incorrect)
This is the best that wecando. (Correct)
Q66. The relative pronoun „that‟ is also used in preference to „who‟ and „which‟ after the
words „all‟, „same‟, „any‟, „none‟, „nothing‟ and (the) „only‟.
He is the same man who hehasseen. (Incorrect)
He is the same man that hehasseen. (Correct)
Man is the only animal whocantalk. (Incorrect)
Man is the only animal thatcantalk. (Correct)
Q67. The same rule applies after the interrogative pronouns „who‟ and „what‟.
What is there which I donotknow? (Incorrect)
What is there that I donotknow? (Correct)

Q68. Whentherearetwoantecedents,amanandananimalortwothingsbeforetherelative
pronoun, we should use„that‟.
The man and his dog which passed through this roadwerekilled. (Incorrect)
The man and his dog that passed through this roadwerekilled. (Correct)

Q69. The case of the noun or pronoun preceding or succeeding the verb „to be‟ should be
the same.
It is him who came toseeus. (Incorrect)
It is he who came toseeus. (Correct)
It is me who caughtthethief. (Incorrect)
It is I who caughtthethief. (Correct)

GENERAL APTITUDE PAGE 195


Q70. When two qualities of a person or thing are compared „more‟ or „less‟ is used before
the adjective and the adjective following them takes positive degree.
Suman is better thanbrave. (Incorrect)
Suman is more goodthanbrave. (Correct)

Q71. When two or more adjectives are used to show the qualities of the same man or thing,
all the adjectives must be in the same degree.
Sita is more intelligent and wisethanRita. (Incorrect)
Sita is more intelligent and wiserthanRita. (Correct)
Ramesh is the wisest and strong boy oftheclass. (Incorrect)
Ramesh is the wisest and the strongest boy oftheclass. (Correct)

Q72. „Very‟ is used with adjectives in the positive degree and with present participles.
He is a muchstrong man. (Incorrect)
He is a verystrongman. (Correct)
It is a muchinterestingbook. (Incorrect)
It is a veryinterestingbook. (Correct)
„Much‟ is used with adjective in the comparative degree and with past participles.
He is very stronger thanIam. (Incorrect)
He is much stronger thanI am. (Correct)
I am very obliged tomyfriend. (Incorrect)
I am much obliged tomyfriend. (Correct)

Q73. To show equality „as‟ is used before and after the adjective.
I can run as fast, if not faster,thanyou. (Incorrect)
I can run as fast as, if not faster,thanyou. (Correct)

GENERAL APTITUDE PAGE 196


Q74. Certain adjectives do not admit of comparison and thus they always remain in the
positive degree:
„Absolute‟, „annual‟, „chief‟, „circular‟, „complete‟, „entire‟, „extreme‟, „excellent‟, „full‟,
„impossible‟, „perfect‟, „right‟, „round‟, „unique‟, „universal‟, „whole‟, etc.
This is the most unique building that Ihaveseen. (Incorrect)
This is the unique building that Ihaveseen. (Correct)

Q75. „Morethanone‟indicatesapluralsense,butitistreatedasasortofcompoundofone. Thus it


agrees with a singular noun and takes a singularverb.
More than one employees were killed intheaccident. (Incorrect)
More than one employee was killed intheaccident. (Correct)

Q76. „Morethanone‟indicatesapluralsense,butitistreatedasasortofcompoundofone. Thus it


agrees with a singular noun and takes a singularverb.
More than one employees were killed intheaccident. (Incorrect)
More than one employee was killed intheaccident. (Correct)

Q77. If the plural subject denotes a definite amount or quantity taken as a whole, the verb
is singular.
Forty miles are agooddistance. (Incorrect)
Forty miles is agooddistance. (Correct)
Two-thirds of the bookwererubbish. (Incorrect)
Two-thirds of the bookwasrubbish. (Correct)

Q78. Whenqualifiedby„each‟or„every‟twosingularsubjects,ifevenconnectedby„and‟, take a


singularverb.
Each boy and every girl were givenrewards. (Incorrect)
Each boy and every girl wasgivenrewards. (Correct)

GENERAL APTITUDE PAGE 197


Q79. The following verbs are always followed byan infinitive:
„decide‟, „plan‟, „expect‟, „fail‟, „hope‟, „intend‟, „learn‟, „promise‟, „refuse‟, „want‟,
„agree‟, „consent‟, „try‟, „love‟, etc.
I refusemeetinghim. (Incorrect)
I refuse tomeethim. (Correct)

Q80. The following verbs and phrases should be followed by a gerund which is a verbal
noun:„enjoy‟,„admit‟,„deny‟,„appreciate‟,„accustomedto‟,„isusedto‟,„donotmind‟,
„object to‟,etc.
I am looking forward to receiveyourreply. (Incorrect)
I am looking forward to receivingyour reply. (Correct)
He is used toworkhard. (Incorrect)
He is used toworkinghard. (Correct)

Q81. The word „to‟ is frequently used with the infinitive, but it is not an essential part ofit.
Forexample,aftercertainverbs(„bid‟,„let‟,„make‟,„need‟,„dare‟,„see‟,„hear‟)weuse the
infinitive without„to‟.
Bid him togothere. (Incorrect)
Bid himgothere. (Correct)
Make himtostand. (Incorrect)
Makehimstand. (Correct)

Q82. The infinitive is used without „to‟ after „had better‟, „had rather‟, „would rather‟,
„sooner than‟ and „rather than‟.
You had better to ask permissionfromhim. (Incorrect)
You had better ask permissionfromhim. (Correct)

Q83. „Than‟ should be used after „no other‟.


I met no other man butMr.Roy. (Incorrect)
I met no other man thanMr.Roy. (Correct)
Ram has no other claim excepthisdegrees. (Incorrect)
Ram has no other claim thanhisdegrees. (Correct)

GENERAL APTITUDE PAGE 197


Q84. Aftertheword„know‟,„how‟,or„when‟shouldbeusedbeforeusinganinfinitive. I know
towriteFrench. (Incorrect)
I know how towriteFrench. (Correct)
Q85. A verb may take an infinitive or a gerund according to its implication. If the verb
indicates a purpose, an infinitive should be used and if the verb indicates a cause, a
gerund should be used.
He went to school for seeingtheprincipal. (Incorrect)
He went to school to seetheprincipal. (Correct)
He was turned out tocopyanswers. (Incorrect)
He was turned out forcopyinganswers. (Correct)
Q86. Adverb„as‟isnotusedwithverbslike„appointed‟,„elected‟,„considered‟,„called‟but it is
used with„regard‟.
He was elected as President ofoursociety. (Incorrect)
He was elected President ofoursociety. (Correct)
I regard Rameshmyfriend. (Incorrect)
I regard Ramesh asmyfriend. (Correct)
Q87. After the verb „doubt‟ „that‟ should not be used. „If‟ or „whether‟ should be used in its
place.
I doubt that Raviwillcome. (Incorrect)
I doubt if Raviwillcome. (Correct)
Q88. Itisacommonpracticeinconversationtomakeastatementandaskforconfirmation;
as„It‟sveryhot,isn‟tit?‟Twopointsaretobekeptinmind.Ifthestatementispositive,
thepatternwillbeAuxiliary+n‟t+subject.
If the statement is negative, the pattern will be
Auxiliary + subject
Examples:
It is raining,isit? (Incorrect)
It is raining,isn‟tit? (Correct)
You are not busy,aren‟tyou? (Incorrect)
You are not busy,areyou? (Correct)

Q89. There are certain common errors which should be avoided.


(a) „Only‟shouldbeplacedimmediatelybeforetheworditqualifies.
He only lost his ticket inthestampede. (Incorrect)
Only he lost his ticket inthestampede. (Correct)
(b) An infinitive verb should not besplit.
I request you to kindlyhelpme. (Incorrect)
I request you kindly tohelpme. (Correct)

GENERAL APTITUDE PAGE 198


Miscellaneous Exercise
EXERCISE-1
Q1. (A)Oneofthefirmestbelief/(B)wasthatherfriendshoutedather/(C)withaview

to make her /(D) aware of her repeated lapses. /(E) No error.

Q2. (A)Theprettywoman/(B)sittinginthecaris/(C)thedaughterof/(D)theoneof

the headman of the village. /(E) No error.

Q3. (A)Youareamongthose/(B)luckiestmanwho/(C)attainednameandfamenotby

/(D) labour but by chance. /(E) No error.

Q4. (A)TheBoardofDirectors/(B)wantallpossiblefacilities/(C)andallowancesfor/

(D) itself. /(E) No error.

Q5. (A)YouandI/(B)havedone/(C)mybestin/(D)theexamination./(E)Noerror.

Q6. (A) His efforts /(B) will bring him /(C) more success /(D) than your. /(E) No error.

Q7. (A)Everyteacherand/(B)everystudentofthiscollege/(C)isdeterminedtodotheir

best for /(D) the glorious prospects of the college. /(E) No error.

Q8. (A)TheVice-PresidentofIndiaand/(B)theVice-Chancellorofthisuniversity/(C)

have given /(D) his consent to join the meeting. /(E) No error.

Q9. (A) She hates everybody /(B) and everything who /(C) reminds her /(D) of her

mistakes. /(E) No

Q10. (A)None/(B)ofthesetwoPrincipals/(C)hasbeenlooking/(D)afterhiscollegewell.

/(E) No error.

Q11. (A)Oneshould/(B)betruetohis/(C)wordinall/(D)circumstances./(E)Noerror.

GENERAL APTITUDE PAGE 199


Q12. (A)Thecriminal/(B)whichwaschained/(C)grumbledatthePoliceInspector/(D)

who was taking him to jail./(E) No error.

Q13. (A)Theofficer/(B)aswellastheassistants/(C)absentedthemselves/(D) fromthe

office. /(E) No error.

Q14. (A)Asastudent/(B)ofscience/(C)youarefarbetter/(D)thanhim./(E)Noerror.

Q15. (A)Thisisthesame/(B)dogwhichbarked/(C)athimbutfortunatelydidnot/(D)

bite him. /(E) No error.

Q16. Whomever /(B) comes late will not be allowed /(C) to make his presence /(D) in the

register. /(E) No error.

Q17. Youandmyself/(B)willenjoytheparty/(C)tobearrangedinhonourof/(D)thenew

Chief Justice. /(E) No error

Q18. (A) You say /(B) it‟s your problems /(C) but I say it‟s /(D) my also. /(E) No error.

Q19. (A) The hotels /(B) of Patna are more /(C) costly than /(D) Muzaffarpur. /(E) No

error.

Q20. (A)Shewasmore/(B)garrulousthan/(C)eitherofher/(D)threesisters./(E)No

error.

Q21. (A) When you will find out /(B) any solution to this problem, /(C) you will become

able /(D) to finalise the project. /(E) No error.

Q22. (A)Whenever/(B)youarecominghere,/(C)youbringalotof/(D)sweetsforme.

/(E) No error.

Q23. (A)Itisappearingtome/(B)thatyouareplotting/(C)againstyour/(D)friendsand

their parents. /(E) No error

GENERAL APTITUDE PAGE 200


EXERCISE-2
Q1. (A) A misogynist is a person /(B) who is hating woman /(C) but a philogynist is

aperson /(D) who loves woman. /(E) No error.

Q2. (A) The secret of his good /(B) health lies in the fact that /(C) he is getting up before

sunrise /(D) and has a two mile walk. /(E) No error.

Q3. (A) She says /(B) that she will take /(C) her umbrella /(D) in case it will rain. /(E)

No error.

Q4. (A) Before the alarm /(B) had stopped ringing /(C) Veera had pulled up /(D) the

shade. /(E) No error.

Q5. (A) I have been knowing /(B) him for ten years /(C) but I don‟t know /(D) wherehe

lives. /(E) No error.

Q6. (A) All the flowers /(B) in his garden will surely die /(C) before the rains /(D) will

fall. /(E) No error.

Q7. (A) If I would have done this, /(B) I would have done wrong /(C) and would have

disappointed /(D) many of my friends. /(E) No error.

Q8. (A) I wish /(B) he saw you /(C) when you were /(D) living in England. /(E) No

error.

Q9. (A)Thejudgehadnotgiven/(B)anydicision/(C)untilhehad/(D)studiedthe

case. /(E) No error.

Q10. (A) Radha came /(B) to the meeting /(C) much later /(D) than I expect. /(E) No

error

Q11. (A) If I was you /(B) I would have told /(C) the Chairman to keep /(D) his mouth
shut. /(E) No error.

GENERAL APTITUDE PAGE 201


Q12. (A)Thevictimtriedtotellus/(B)whathashappened/(C)buthiswords/(D)were

not audible. /(E) No error.

Q13. (A) The problems that our /(B) country has and is bound to face /(C) are the

outcome of the selfishness /(D) of our so calledsaviours. /(E) No error.

Q14. (A) Never before /(B) the students of India have so much been frustrated /(C) as

today when the problems /(D) of unemployment are dominant. /(E) No error.

Q15. (A)Beingarainyday/(B)Vijaydecidedtostay/(C)athomeandwork/(D)further

on the problem. /(E) No error

Q16. (A)Schooloffersmanyopportunitiesofmeeting/(B)helpfulpeople,readinguseful

books/(C)andobtaininformationabout/(D)avarietyofpubliccareers./(E)No error.

Q17. (A) Taking tea, /(B) he went to the office /(C) and disposed /(D) of a lot of work.

/(E) No error.

Q18. (A) Only one dispute /(B) has been settled /(C) and others left to both the parties

/(D) to settle. /(E) No error.

Q19. (A) Without taking proper care, /(B) the doctors could not have been saved /(C)this

patient who received /(D) a bullet injury in the chest. /(E) No error.

Q20. (A) Going towards them /(B) with some cups of /(C) tea in the tray, somebody

knocked /(D) at the door. /(E) No error.

Q21. (A) During the course of investigation /(B) the police learnt that not only /(C) the

house was looted but /(D) also the things taken away. /(E) No error.

GENERAL APTITUDE PAGE 202


Q22. (A) He does nothing /(B) but to find /(C) faults in others and laugh /(D) at them.

/(E) No error.

Q23. (A)Weadvisedhim/(B)tomarryhisdaughter/(C)becauseshehad/(D)cometo

marriageable age. /(E) No error.

Q24. (A) The criminal /(B) will certainly be hung /(C) because the charges brought upon

/(D) him are very serious. /(E) No error.

Q25. (A) Owing to his /(B) ill health, he will /(C) not be able to give this examination,

which means /(D) he will have to lose one year. /(E) No error.

GENERAL APTITUDE PAGE 203


EXERCISE-3
Q1. (A) Walking in the field /(B) late at night, a snake bit me /(C) and that was the

reason /(D) why I was in a hurry. /(E) No error.

Q2. (A) When she entered /(B) the room, she found the /(C) child sleeping peacefully

and /(D) the fan move slowly. /(E) No error.

Q3. (A) Two miles beyond /(B) that pasture was hundreds /(C) of cattle including /(D)

some lambs. /(E) No error.

Q4. (A)Itisnoticedthat/(B)theeliteclasshavenosoft/(C)cornerforthedowntrodden

who are the /(D) real victims of the present social set-up. /(E) No error.

Q5. (A) Ritu as well as /(B) some of her friends /(C) have fallen in love with Sonu who

/(D) is the eldest son of an S.D.O. /(E) No error.

Q6. (A) More than one successful candidate /(B) have given interviews for one /(C) of

the popular magazines /(D) being published from Delhi. /(E) No error.

Q7. (A) The leader as well as /(B) his followers are fatally injured in /(C) the train

accident which occurred last night /(D) near this railway crossing. /(E) No error.

Q8. (A) Just outside /(B) the hotel is /(C) two bars, extremely beautiful /(D) as well as

crowdy. /(E) No .

Q9. (A) We should not forget that /(B) we have a right to criticize but /(C) at the same

time each of us /(D) have to remember the duty also. /(E) No error.

Q10. (A)Oneof/(B)thebiggestindustrialhouses/(C)inBombayisonthevergeof/(D)

declaringalockout./(E)Noerror.

GENERAL APTITUDE PAGE 204


Q11. (A) Each of the students, /(B) whom I have chosen to take part /(C) in thecultural

programmestobeperformed/(D)intheCityHall,areuptothemark./(E)Noerror.

Q12. (A) Everybody among the businessmen /(B) were injoying drinking /(C) when the

Managerofthehotel/(D)wasshotdead./(E)Noerror.

Q13. (A) Although these buildings are /(B) in need of repair, /(C) there have been much

improvement /(D) in their appearance. /(E) No error.

Q14. (A)Notonlythedoctor/(B)butalsothenursesofthisnursing/(C)homeisverykind

and helpful /(D) to the attendants. /(E) No error.

Q15. (A)Eachfacultymemberas/(B)wellasmostofthestudents/(C)wereoftheview

that there should /(D) be many more new courses. /(E) No error.

Q16. (A) Either the manager /(B) or his assistants always try to misguide the public /(C)

regarding the vacancy /(D) in the factory. /(E) No error.

Q17. (A)Everymanandwoman/(B)ofthevillagehavecomeout/(C)toseethisstrange

child/(D)whoclaimstoknoweverythingabouthispre-natalexistence./(E)Noerror.

Q18. In our college, it /(B) was obligatory for each of /(C) the students to buy /(D) his own

instruments. /(E) No error.

Q19. (A) One of the developing or underdeveloped /(B) countries are in favour of

launching/(C)chemicalwarfarebecauseittrembles/(D)toimagineitsrepercussions.

/(E) No error.

Q20. (A)NeitherRajni/(B)norRaginiweretotaketoherheels/(C)whentheysawacobra

laying /(D) at the gate. /(E) No error.

GENERAL APTITUDE PAGE 205


Q21. (A) If Mahatma Gandhi /(B) was alive /(C) he would start weeping /(D) to see the

present condition of India. /(E) No error.

Q22. (A)Wassheabird/(B)shewoulddefinitelyflytoyou/(C)andsaythatshecouldnot

/(D) live without you. /(E) No error.

Q23. (A) He was not /(B) so well versed in /(C) English that we /(D) had expected. /(E)

No error.

Q24. (A) Mrs. Varun /(B) not only stopped coming /(C) here but also going to any place

/(D) which is related to her past tragedy. /(E) No error.

Q25. (A) Neither the doctor /(B) nor his assistants /(C) were asleep when /(D) the phone

rang up. /(E) No error.

EXERCISE-4
Q1. (A)Youmusteither/(B)informthepolice/(C)elsebeprepared/(D)tosufferanyloss.

/(E) No error.

Q2. (A) As soon as the peon /(B) rings the first bell, then all /(C) the students assemble

/(D) on the ground for prayer. /(E) No error.

Q3. (A) He not only comes /(B) here for shopping but also for /(C) having a glimpse of

/(D) the extremely beautiful sales girl. /(E) No error.

Q4. (A) Hardly had he /(B) come out of the bus /(C) then the bomb exploded /(D) and

shattered the bus into pieces. /(E) No error.

Q5. (A) Scarcely had he bought /(B) the ticket when the guard /(C) showed the flag and

/(D) the train started. /(E) No error.

GENERAL APTITUDE PAGE 206


Q6. (A)Thisisnotsuch/(B)amajorproblem/(C)whichcannotbesolved/(D)withalittle

effort. /(E) No error

Q7. (A)Thisisperhaps/(B)thesamedogwhich/(C)bitherwhileshewasreturning/(D)

from market. /(E) No error.

Q8. (A) Both Hari /(B) as well as his wife /(C) are determined to bring /(D) about some

changesintheplan./(E)Noerror.

Q9. (A)Asheis/(B)aperfectionist,/(C)sohealwaysinsists/(D)onregularpractice./

(E) No error.

Q10. (A)Althoughtheserooms/(B)areinneedofrepair,/(C)buttheownerdoesnottake

/(D) any notice of their condition. /(E) No error.

Q11. (A)Hesuggestedme/(B)thatIshould/(C)doalltheworkas/(D)quicklylikehim.

/(E) No error.

Q12. (A)Nosoonerdid/(B)hefindoutanysolution/(C)totheproblemwhen/(D)another

problem appeared. /(E) No error.

Q13. (A) The robbers had /(B) hardly put the ornaments /(C) in his bag than /(D) the

housewifewokeup./(E)Noerror.

Q14. (A)Scarcelyhadhe/(B)goneafewsteps/(C)thathewastoldbysomeone/(D)that

his mother was no more. /(E) No error.

Q15. (A) Mrs. Sen told me that though /(B) her son had worked hard but /(C) he failed to

make /(D) any mark in the last examination. /(E) No error.

Q16. (A) Dr. Sinha was not only /(B) sympathetic to the rich /(C) patients but also /(D) to

the poor ones. /(E) Noerror.

GENERAL APTITUDE PAGE 207


Q17. (A) Because he is intelligent, /(B) therefore he gets good /(C) marks in all the /(D)

examinations he takes. /(E) No error.

Q18. (A) Both the rich along with /(B) the poor are responsible for a /(C) great many vices

with which our society /(D) as well as country is inflicted. /(E) No error.

Q19. (A) He exclaimed with sorrow /(B) that his brother died /(C) just two /(D) months

before./(E)NoError.

Q20. (A)InsteadofgoingtoLondon/(B)hewenttoAmerica/(C)andstays/(D)therefor

a month. /(E) No error.

Q21. (A)Theprisonerswalkedslowly/(B)fortheyknewthatas/(C)soonastheycrossthe

gate, /(D) the jailor would ask them to jog. /(E) No error.

Q22. (A)Theprisonerswalkedslowly/(B)fortheyknewthatas/(C)soonastheycrossthe

gate, /(D) the jailor would ask them to jog. /(E) No error.

Q23. (A)WhenMeenasaid/(B)thatshewascomingtosee/(C)methenextday,Iwondered

/(D) what problems she will bring. /(E) No error.

Q24. (A) I suggested that Ragini should /(B) stay here at night if she /(C) got late but she

does /(D) not pay any attention to my suggestion. /(E) No error

Q25. (A)Heaskedme/(B)whyhadIgonetotheCinema/(C)lateatnightinspiteof/(D)

his clear instructions. /(E) No error.

GENERAL APTITUDE PAGE 208


EXERCISE-5
Q1. (A) On reaching the station /(B) he found that /(C) he left the papers /(D) in the

drawing room.

Q2. (A)Herequestedtheteacher/(B)toallowhimtogohome/(C)ashegot/(D)asevere

headache. /(E) No error.

Q3. (A) Jyoti asked me /(B) what could she do /(C) for me in that /(D) critical situation.

/(E) No error.

Q4. (A) The Director knowing of my /(B) interest in Environmental Science /(C) asked me

that I would /(D) like to attend the National Seminar. /(E) No error.

Q5. (A)TheGovernmentwarnedtheshopkeepersthatif/(B)theypersistin/(C)charging

high prices, their /(D) licences would be cancelled. /(E) No error.

Q6. (A)Iwassurprised/(B)toknowwhyhadheturned/(C)downsuchagoodoffer/(D)

of marriage.

Q7. (A) My father wanted to know /(B) that I had done /(C) all the work /(D) assigned to

me./(E)NoError.

Q8. (A) In his famous sonnet, /(B)Milton says that /(C) “They also serve /(D) who only

stand and wait” /(E) No error.

Q9. (A) Any step that the centre /(B) takes to establish /(C) peace and harmony in the

country /(D) will be appreciate. /(E) No error.

Q10. (A)Thepolicemen/(B)whowereondutyinthisarea/(C)werediscovered/(D)two

drug addicts. /(E) No error.

GENERAL APTITUDE PAGE 209


Q11. (A) As the first President /(B) of India, Dr. Rajendra Prasad /(C) was known by all

types /(D) of people in the world. /(E) No error.

Q12. (A) The teacher asked /(B) the students whether they could /(C) tell the name of the

man /(D) who had been discovered America. /(E) No error.

Q13. (A) She held something /(B) at her side which /(C) was totally hiding /(D) by the folds

of her sari. /(E) No error.

Q14. (A) I was surprising /(B) at not having seen /(C) her even though she was standing

/(D) just beside my wife. /(E) No error.

Q15. (A)Incaseyouapologise/(B)forhavingbrokenyour/(C)promisesyouwill/(D)

certainly be forgave. /(E) No error.

Q16. (A)Hadthepolicenot/(B)reachedhere/(C)ontime,Iwould/(D)havekilledbythe

robbers/(E)Noerror.

Q17. (A)Bywhom/(B)washehelped/(C)whenheseriouslywounded/(D)byaterrorist?

/(E)Noerror.

Q18. (A)APoliceInspectorsent/(B)tomyhouse/(C)andalltherooms/(D)werechecked.

/(E)Noerror.

Q19. (A)Theneedyman/(B)wasapproachedtothemoneylender/(C)andrequestedhim

to/(D)lendhimsomemoney./(E)NoError.

Q20. (A)Itwassuggested/(B)bythedoctor/(C)thatthepatient/(D)shouldbetakencare.

/(E)NoError.

Q21. (A) She was extremely /(B) annoyed by /(C) her lover‟s unreasonable /

(D)behavior./(E)NoError.

GENERAL APTITUDE PAGE 210


Q22. (A) Five persons killed /(B) and a baby was /(C) badly injured in the /(D) bus accident

whichtookplacelastnight./(E)NoError.

Q23. (A)Whenhelentme/(B)somebooks,heasked/(C)metoreturnthemback/(D)by

theendofthemonth./(E)NoError.

EXERCISE-6
Q1. (A)Hedenied/(B)thathewasnot/(C)presentatthespotof/(D)occurrence./

(E) No Error.

Q2. (A)Myfatheraskedme/(B)ifIhad/(C)sufficientenoughmoney/(D)topaythefees.

/(E)NoError.

Q3. (A)Thecandidate/(B)requestedthechairman/(C)torepeatthe/(D)questionagain.

/(E)NoError.

Q4. (A)Theprincipalforbade/(B)thestudentsnotto/(C)stagea/(D)walkout./€No
Error.
Q5. (A) “The patient is comparatively better /(B) today and I hope /(C) that he will

recoversoon”,/(D)saidthedoctor./(E)NoError.

Q6. (A)Sheaskedmepolitely/(B)whereIwasgoingto/(C)andwhatIwould/(D)bring

forher./(E)NoError.

Q7. (A)Probablybytheend/(B)oftheyearhewill/(C)leavefortheUSA/(D)withbag

andbaggage./(E)NoError.

Q8. (A) Throughout the whole year /(B) there was not /(C) a single day /(D) without
someviolence./(E)NoError.
Q9. (A)Shereimbursedback/(B)themoneywhichIhad/(C)spentduringour/(D)
journeyto Kathmandu./(E)NoError.

GENERAL APTITUDE PAGE 211


Q10. (A) Unless you do not /(B) go through your books /(C) you cannot get through the
examination. /(D) No Error.
Q11. (A) The last project of /(B) the company was /(C) equally as successful /(D) as the
firstproject./(E)NoError.
Q12. (A) In spite of his insincerity, /(B) he was awarded /(C) cent-percent marks /(D) in
mathematics./(E)NoError.
Q13. (A)Eachcompetitor/(B)triedhisbest/(C)todefeat/(D)theone-another./(E)No
Error.
Q14. (A)Heranasfastly/(B)ashecouldto/(C)wintherace/(D)butultimatelyhelost.
/(E)NoError.
Q15. (A)Heputhissign/(B)atthefootofthe/(C)letterandpostedit/(D)withoutdelay.
/(E)NoError.

GENERAL APTITUDE PAGE 212


EXERCISE-7 (ANTONYMS)
In the following questions choose the word which is the exact OPPOSITE of the given words.
Q1. ENORMOUS
(a) Soft (b) Average (c) Tiny (d) Weak

Q2. COMMISSIONED
(a)Started (b)Closed (c)Finished (d)Terminated

Q3. ARTIFICIAL
(a)Red (b)Natural (c)Truthful (d)Solid

Q4. EXODUS
(a)Influx (b)Home-coming (c)Return (d)Restoration

Q5. RELINQUISH
(a)Abdicate (b)Renounce (c)Possess (d)Deny

Q6. EXPAND
(a)Convert (b)Condense (c)Congest (d)Conclude

Q7. MORTAL
(a)Divine (b)Immortal (c)Spiritual (d)Eternal

Q8. QUIESCENT
(a) ACTIVE (b)Dormant (c)Weak (d)Unconcerned

Q9. OBEYING
(a) Ordering (b)Following (c) Refusing (d) Contradicting

Q10. FRAUDULENT
(a)Candid (b)Direct (c) Forthright (d)Genuine

GENERAL APTITUDE PAGE 213


EXERCISE-8 (SYNONYMS)
In the following the questions choose the word which best expresses the meaning of the given
word.
Q1. CORPULENT
(a) Lean (b)Gaunt (c)Emaciated (d)Obese

Q2. BRIEF
(a)Limited (b)Small (c)Little (d)Short

Q3. EMBEZZLE
(a) Misappropriate (b)Balance (c)Remunerate (d)Clear

Q4. VENT
(a)Opening (b)Stodge (c)End (d)Past tense of go

Q5. AUGUST
(a) Common (b) Ridiculous (c) Dignified (d)Petty

Q6. CANNY
(a) Obstinate (b)Handsome (c)Clever (d)Stout

Q7. ALERT
(a) Energetic (b) Observant (c)Intelligent (d)Watchful

Q8. WARRIOR
(a) Soldier (b)Sailor (c)Pirate (d)Spy

Q9. DISTANT
(a)Far (b)Removed (c)Reserved (d)Separate

Q10. ADVERSITY
(a) Failure (b)Helplessness (c)Misfortune (d)Crisis

GENERAL APTITUDE PAGE 214


EXERCISE-9(IDIOMSANDPHRASE
Directions for next Ten questions :Some proverbs/idioms are given below together with
Their meanings. Choose the correct meaning of proverb/idiom.
Q1. To make clean breast of
(a)To gain prominence
(b)To praise oneself
(c)To confess without of reserve
(d)To destroy before it blooms

Q2. To keeps one's temper


(a) To becomehungry
(b) To be in goodmood
(c) To preserve onesenergy
(d) To be alooffrom

Q3. To catch a tartar


(a) To trap wanted criminal with greatdifficulty
(b) To catch a dangerousperson
(c) To meet withdisaster
(d) Todealwithapersonwhoismorethanone'smatch

Q4. To have an axe togrind


(a) A private end toserve
(b) To fail to arouseinterest
(c) To have noresult
(d) To work for bothsides

Q5. To cry wolf


(a) To listeneagerly
(b) To give falsealarm
(c) To turnpale
(d) To keep offstarvation

GENERAL APTITUDE PAGE 215


Q6. To end in smoke
(a) To make completelyunderstand

(b) To ruinoneself

(c) To excite great applause

(d)To overcomesomeone

Q7. To be above board


(a) To have a goodheight

(b) To be honest in any businessdeal

(c) They have nodebts

(d) To try to bebeautiful

Q8. To pick holes


(a) To find some reason toquarrel
(b) To destroysomething
(c) To criticizesomeone
(d) To cut some part of anitem

Q9. To leave someone in the lurch


(a) To come to compromise withsomeone
(b) Constant source of annoyance tosomeone
(c) To put someone atease
(d) To desert someone in hisdifficulties

Q10. To beg the question


(a) To referto
(b) To take forgranted
(c) To raiseobjections
(d) To bediscussed

GENERAL APTITUDE PAGE 216


ANALYTICAL APTITUDE
Direction sense Test
Datafornexttwoquestions:AnkitstartsfromhishouseTOWARDSWestAfterwalkinga
distance of 30 metres, he again turned towards right and walked 20 metres. He then turned
leftandmovingadistanceof10mturnedtohisleftandwalked40metres,turnstotheleft
and walks 5 metres. Finally he turns to his left
Q1. In which direction is he walking now?
(a) North
(b) South
(c) East
(d) South-
WestAnswer:A
Q2. Now, how far is he from his starting point (in meters)?
(a)20m (b)25m (c)21.62m (d) 20.62 m
Answer: D
Data for next two questions: A rat runs 20 feet towards East and turns a right, runs 10 feet
and turns to right, runs 8 feet and again turns to left, runs 5 feet and then turns to left, runs 12
feet and finally turns to left and runs 8 feet.
Q3. Now, which direction is the rat facing?
(a)East (b)West (c) (d) South
NorthAnswer:C
Q4. Now, how far is he from his starting point (in meters)?
(a)31ft (b)25ft (c)24.62ft (d) none of these
Answer: B
Q5. Arushi leaves tram her home. She first walks 30 metres in North-west direction and
then30minSouth-Westdirection.Next,shewalks30metresinSouth-eastdirection.
Finally, she turns towards her house. In which direction is shemoving?
(a)North-east (b) North-west (c)South-east (d)South-west
Answer: A

GENERAL APTITUDE PAGE 217


Q6. I am facing South. I turn right and walk 20 in. Then I turn right again and walk 10m.
Then I turn left and walk 10 m and then turning right walk 20 m. Then I turn right
againandwalk60m.InwhichdirectionamIfromthestartingpoint?
(a)North (b) North-west (c)East (d)North-east
Answer: D
Q7. Amanwalks1kmtowardsEastandthenheturnstoSouthandwalks5km.Againhe
turnstoEastandwalks2km,afterthisheturnstoNorthandwalks9km.Now,how
farishefromhisstartingpoint(inkm)?
Answer: 5
Q8. Going50mtotheSouthofherhouse,Ankitaturns/leftandgoesanother20m.Then, turning
to the North, she goes 30 m and then starts walking to her house. In which direction
is she walkingnow?
(a)North-west (b)North (c)South-east (d)
EastAnswer:A

Q9. Manojwalks20mNorth.Thenheturnsrightandwalks30m.Thenheturnsrightand walks 35


m Then he turns left and walks 15 m. Then he again turns left and walks15
m. In which direction and how many metres away is he from his original position?
(a) 45 metreseast
(b) 30 metreseast
(c) 30 metreswest
(d) 45 metreswest
Answer:D
Q10. Five boys are standing in a row facing East. Deepak is to the left of Sameer, Tushar
and Shailendra. Sameer, Tushar and Shailendra are to the left of Sushil. Shailendra is
betweenSammerandTushar.IfTusharisfourthfromtheleft,howfarisSameerfrom
theright?
(a)First (b)Second (c)Third (d)
FourthAnswer:D

Read the following information carefully and answer the question given below it :

GENERAL APTITUDE PAGE 218


(i) There are six flat on a floor in two rows facing north and south are allotted to
allottedtoP,Q,R,S,TandU.
(ii) QgetsaNorthfacingflatandisnotnexttoS.
(iii) S and U get diagonally oppositeflats.
(iv) RnexttoU,getsasouthfacingflatandTgetsNorthfacingflat.

Q11. Which of the following combination get south facing flats?


(a)QTS (b)UPT
(c)URP (d)Data isinadequate
Answer: C

Q12. Whose flat is between Q and S?


(a)T (b)U (c)R (d)P
Answer: A
Q13. IftheflatsofPandTareinterchangedthenwhoseflatwillbenexttothatofU? (a)P
(b)Q (c)R (d)T
Answer: C
Q14. The flats of which of the other pair than SU, is diagonally opposite to each other?
(a)QP (b)QR (c)PT (d)
TSAnswer:A
Q15. Fromtheoriginalposition,BandDmoveoneandahalf-lengthofsidesclockwiseand
anticlockwiserespectively.Whichoneofthefollowingstatementsistrue?
(a) BandDarebothatthemidpointbetweenAandC
(b) D is at the midpoint between A and C, and B at the corner originally occupied by
A.
(c) B is at the midpoint between A and C, and D at the corner originally occupied by
A.
(d) BandDarebothatthemidpointbetweenAandD.
Answer: A

GENERAL APTITUDE PAGE 219


Q16. Anoop starts walking towards South. After walking 15 m he turns towards North.
After walking 20 m, he turns towards East and walks 10 m. He then turns towards
south and walks 5 m. How far is he from his original position and in which direction?
(a)10m,North (b) 10 m, South (c)10m,West (d)10m,East
Answer: D
Q17. Village„C‟is20kmtotheNorthofvillage„R‟.Village„B‟is18kmtotheEastofvillage „R‟.
Village „A‟ is 12 km to the West of „C‟. If Sanjay starts from village „B‟ and goes to
village „A‟, in which direction is he from his startingpoint?
(a)North (b)North-West (c)South (d)South-East
Answer: B
Q18. From a point, Rajneesh started walking East and walked 35 m. He then turned on his
right and walked 20 m and he again turned to right and walked 35 m. Finally, he
turned his left and walked 20 m and reached his destination. Now, how far is he from
the starting point?
(a)50m (b)55m (c)20m (d)40m
Answer: D
Q19. A rat runs 20 m towards East and turns to right, runs 10 m and turns to right run 9 m
and again turns to left, runs 5 m and then turns to left, runs 12 m and finally turns to
left and runs 6 m. Now, which direction is the rat facing?
(a)East (b)North (c)West (d)South
Answer: B
Q20. Vijayan started walking towards South. After walking 15 m, he turned to the left and
walked 15 m. He again turned to his left and walked 15 m. How far is he from his
original position and in which direction?
(a)30m,East (b)15 m, South (c)15m,East (d)15 m,West
Answer: C
Q21. Rakeshisstandingatapoint.Hewalks20mtowardstheEastandfurther
10mtowardstheSouth,thenhewalks35mtowardstheWestandfurther

GENERAL APTITUDE PAGE 220


5 m towards the North, then he walks 15 m towards the East. What is the straight
distance in metres between his starting point and the point where he reachedlast?
(a)0m (b)5m (c)10m (d)None ofthese
Answer: B
Q22. Rahim started from point X and walked straight 5 km West, then turned left and
walked straight 2 km and again turned left and walked straight 7 km. In which
direction is he from the point X?
(a)North-East (b)South-West (c)South-East (d)North-West
Answer: C

Q23. Ravitravelled4kmstraighttowardsSouth.Heturnedleftandtravelled6kmstraight, then


turned right and travelled 4 km straight. How far is he from the starting point?
(a)8km (b)10km (c)12km (d)18km
Answer: B
Q24. Mohan started from point „A‟ and proceeded 7 km straight towards East, then he
turned left and proceeded straight for a distance of 10 km. He then turned left again
and proceeded straight for a distance of 6 km, and then turned left again and
proceeded straight for another 10 km. In which direction is Mohan from his starting
point?
(a)East (b)West (c)North (d)South
Answer: A
Q25. B is to the South-West of A, C is to the East of B and South-East of A and D is to the
North of C in line with B and A. In which direction of A is D located?
(a) North
(b)East
(c)South-East
(d)North-East
Answer: D

GENERAL APTITUDE PAGE 221


Q26. Starting from a point, Raju walked 12 m North, he turned right and walked 10 m, he
again turned right and walked 12 m, then he turned left and walked 5 m. How far is
he now and in which direction from the starting point?
(a)27 m towards East
(b)5 m towards East
(c)10 m towards West
(d)15 m towards East
Answer: D

Q27. A watch reads 4.30. If the minute hand points East, in what direction will the hour
hand point?
(a) South-East
(b) North-East
(c) North
(d)North-West
Answer:B

Q28. „A‟ walks 10 m, towards East and then 10 m to his right. Then every time turning to
his left, he walks 5, 15 and 15 m, respectively. How far is he now from his starting
point?
(a)5m (b)10m (c)15m (d) 20m
Answer:A

Q29. A postman was returning to the post office which was in front of him to the north.
When the post office, was 100 m away from him, he turned to the left and moved 50
mtodeliverthelastletterattheShantivilla.He,thenmovedinthesamedirectionfor 40 m,
turned to his right and moved 100 m. How many metres was he away from the
postoffice?
(a)0m (b)150m (c)90m (d) 100m
Answer:C

BLOOD RELATION

GENERAL APTITUDE PAGE 222


Q30. AisB'ssister.CisB'smother.DisC'sfather.EisD'smother.Then,howisArelated toD?
(a) Grandfather
(b) Grandmother
(c) Daughter
(d) Granddaughter
Answer: D
Solution:
A is the sister of B and B is the daughter of C.
So,AisthedaughterofC.Also,DisthefatherofC. So, A
is the granddaughter ofD.

Q31. Inafamily,therearesixmembersA,B,C,D,EandF.AandBareamarriedcouple, A being the


male member. D is the only son of C, who is the brother of A. E is the sister of D. B
is the daughter-in-law of F, whose husband has died. How is E related to C?
(a)Sister (b)Daughter (c)Cousin (d)
MotherAnswer:B

Q32. Q is the brother of R;


P is the sister of Q;
T is the brother of S;
S is the daughter of R.
Who are the cousins of Q ?
(a) RandP (b) PandT (c) QandT (d) S
andTAnswer:D

Q33. IfA+BmeansAisthemotherofB;A-BmeansAisthebrotherB;A%BmeansA
isthefatherofBandAxBmeansAisthesisterofB,whichofthefollowingshows that P is
the maternal uncle ofQ?
(a) Q - N + Mx P (b) P + S x N -Q
(c) P - M + NxQ (d) Q - S %P

GENERAL APTITUDE PAGE 223


Answer: C
Solution:

Q34. Introducing a man, a woman said, 'He is the only son of my mother's mother." How
is the woman related to the man?
(a)Mother (b)Aunt (c)Sister (d)
NieceAnswer:D

Data for next two questions:


A + B means A is daughter of B
A × B means A is son of B
A − B means A is wife of B
Q35. If T-S x B-M, which of the following is not true /
(a)B is motherofS (b)M is husband ofB
(c)T is wifeofS (d)S is daughter of B .
Answer:D

Q36. If Z x T –S x U +P, what is U to Z.


(a)Mother (b)GrandMother
(c)Father (d)Can‟t bedetermined
Answer: B

Q37. (i) InafamilyofsixpersonsA,B,C,D,EandF,therearetwomarriedcouples.


(ii) DisgrandmotherofAandmotherofB.
(iii) C is wife of B and mother ofF.
(iv) F is the grand daughter ofE.
Who among the following is one of the couples ?

GENERAL APTITUDE PAGE 224


(a)CD (b)DE (c)EB (d)DF
Answer :- D
Q38. IfDisthebrotherofB,howBisrelatedtoC?Toanswerthisquestionwhichofthe statements
is/arenecessary?

1. The son of D is the grandson ofC.


2. B is the sister ofD.
(a)Only1 (b)Only2
(c)Either 1or2 (d)1 and 2 both are required
Solution:Given:DisthebrotherofB.
From statement 1, we can detect that D is son of C (son of D is the grandson of C).
Fromstatement2,wecandetectthatBis'Female'(sisterofD).
Therefore, B is daughter ofC.

Q39. If'A$B'means'AisbrotherofB','A@B'means'AiswifeofB','A#B'means'Ais daughter of


B' and 'A &B'means 'A is father of B', then which of the following
expressionsindicatestherelationship'K'isfather-in-lawofH'?
(a) H@J$L#P&K
(b) H@J$P&L#K
(c) H @ J $ L # K &P
(d) H@P$J&L#K
Answer:C
Solution:
K is the father-in-law of H means H is the wife of the brother (say, J) of the daughter
(say, L) of K and K is a male (brother or father of some person, say P) i.e H @ J $ L #
K$PorH@J$L#K&P.

Q40. IfAisthebrotherofBandK,DisthemotherofBandEisthefatherofA.Whichone of the


following statements is not definitelytrue?
(a)BisthebrotherofK (b)A is the father ofK
(c)A is the sonofD (d)D is the wife ofE

Q41. Lookingataportraitofaman,Sanjaysaid,“Hismotheristhewifeofmyfather‟sson.
Brothers and sisters I have none.” At whose portrait was Sanjay looking?

GENERAL APTITUDE PAGE 225


(a)Hisson (b) Hisnephew
(c)Hiscousin (d) Hisuncle

Q42. Old man‟s son is my son‟s uncle, then what relation has the old man to me?
(a)Brother (b)Father
(c)Grandfather (d)Uncle
Q43. If(1)„S+K‟means„SisthesisterofK‟;(2)„S–K‟means„SisthefatherofK‟;(3)„S× K‟ means „S
is the brother of K‟; which of the following means „P is the aunt of D‟? (a)P + M–D
(b)P – M ×D
(c)P – M+D (d)P × M –D
Directions (Q.45 – 46): Read following statements carefully and answer questions given
below:
(A) P×QmeansPisthebrotherofQ
(B) P+QmeansPisthefatherofQ
(C) P÷QmeansPisthesisterofQ
Q44. Which of the following represents A is uncle of M?
(a) A + D÷M (b)A × D +M
(c) A + D×M (d)A ÷ D +M
Q45. Which of the statements is superfluous to answer the above question?
(a)Conly (b)B or Conly
(c)Aonly (d)None ofthese
Q46. P is brother of Q. R is sister of Q. How P is related to R?
(a)Uncle (b)Datainadequate
(c)Brother (d)None ofthese
Q47. If P $ Q means P is father of Q, P # Q means P is mother of Q, P * Q means P is sister
of Q, then how is Q related to N in N # L $ P *Q?
(a)Noneofthese (b)Granddaughter
(c)Nephew (d)Datainadequate

Q48. A$B means B is father of A, A#B means B is mother of A, A*B means B is sister of A,
A@B means B is husband of A, then which of the following indicates the relationship
„N‟ is grandmother of Q?

GENERAL APTITUDE PAGE 226


(a) P * Q # M$N (b)P @ Q $ M #N
(c) P # Q $ N*M (d)P $ Q # M @ N

Q49. If „A + B‟ means „A is brother of B‟, „A – B‟ means „A is sister of B‟, „A × B‟ means „A is


wife of B‟, and „A * B‟ means „A is father of B‟, then which of the following indicates „S
is son ofP‟?
(a) P×Q*R+S–T (b) P × Q * S – R +T
(c) P × Q * R – T+ S (d) P × Q * R – S +T

Directions (Q.51 – 52): Study the following information carefully and answer the given
questions based on it.
i. „P × Q‟ means „Q is motherofP‟ ii. „P + Q‟ means „P is father ofQ‟.
iii. „P – Q‟ means „P is brotherofQ. iv. „P † Q‟ means „Q is sister ofP‟.

Q50. Which of the following means „M is niece of T‟?


(a)M ÷ D + T×R (b)T – D + R ÷M
(c)T×D+R÷M (d)Datainadequate

Q51. Which of the following statements is superfluous to answer the above questions?
(a)i only
(b) iionly

(c) Either i or ii only

(d)Either iii or ivonly

Directions (Q.53–54): Study meaning of given symbols and answers questions based on it.
i. „P×Q‟means„QismotherofP.
ii. „P + Q‟ means „P is brother ofQ‟.
iii. „P – Q‟ means „P is sister ofQ‟.
iv. „P † Q‟ means „Q is father ofP‟.
Q52. Which of the following definitely means R is grandson of K?
(a)R × T ÷ K
(b)R + M × T ÷ K

GENERAL APTITUDE PAGE 227


(c) M – R × T ÷K

(d) None ofthese

Q53. Which of the following statements is superfluous to answer the above questions?
(a)ivonly
(b)i only
(c)ii only
(d)iiionly

Q54. If „P + Q‟ means „P is mother of Q‟, „P × Q‟ means „P is father of Q‟, „P – Q‟ means „P is


brother of Q‟ and „P†Q‟ means „Q is wife of P‟ then which of following means „G is
paternal uncle of S‟?
(a) G – M ÷ T +S
(b) G – M +S
(c) T × G – Q +S
(d) S + M ÷ N –G

Directions (Q.56 – 57): Study the following information carefully and answer the given
questions following it.
i. „P × Q‟ means „P is brother ofQ‟.
ii. „P + Q‟ means „Q is mother ofP‟.
iii. „P – Q‟ means „Q is sister ofP‟
iv. „P † Q‟ means „P is father ofQ‟.
Q55. Which of the following means T is nephew of R?
(a)R × J ÷ T
(b) R – M + T – J
(c)R × J ÷ T × K
(d) None of these
Q56. Which of the following means is superfluous to answer the above questions?
(a)ionly (b)iionly

GENERAL APTITUDE PAGE 228


(c)ii andiiionly (d)iiionly

Q57. „A + B‟ means „B is brother of A‟, „A × B‟ means „B is husband of A‟, „A – B‟ means „A


is mother of B‟ and „A † B‟ means „A is father of B‟. Then which of the following
expressions indicates „P‟ is grandmother of „T‟?
(a)Q – P + R ÷T
(b)P × Q ÷ R –T
(c)P × Q ÷ R +T
(d)P + Q ÷ R –T

Q58. If„A+B‟means„AisthesisterofB‟,„A×B‟means„AisthewifeofB‟,„A†B‟means „A is the


father of B‟ and „A – B‟ means „A is the brother of B‟, then which of following expresses
the relationship that „T is the daughter ofP‟?
(a) P×Q÷R+S–T
(b)P×Q÷R–T+S
(c) P × Q ÷ R + T –S
(d) P × Q ÷ R + S +T

ANALYTIC PUZZLES
Directions (Q.1 – 4): Read the following information and answer the questions givenbelow
it:SixstudentsA,B,C,D,EandFaresittinginthefield.AandBarefromNehruHousewhile
therestbelongtoGandhiHouse.DandFaretallwhiletheothersareshort.A,CandDare
wearing glasses while the others are not.
Q59. Which two students, who are not wearing glasses are short?
(a) AandF (b)CandE (c)BandE (d) E andF

Q60. Which short student of Gandhi House is not wearing glasses?


(a)F (b)E (c)B (d)A

Q61. Which tall student of Gandhi House is not wearing glasses?


(a)B (b)C (c)E (d)F

GENERAL APTITUDE PAGE 229


Q62. Onashelfareplacedsixvolumesside-by-sidelabelledA,B,C,D,EandF.B,C,E,F
havegreencoverswhileothershaveyellowcovers.A,D,Barenewvolumeswhilethe
restareoldvolumes.A,C,Barelawreportswhiletherestaremedicalextracts.Which two
volumes are old medical extracts and have greencovers?
(a)B,C (b)C,D (c)C,E (d) E,F

Directions(Q.64–68):Readfollowinginformationcarefullyandanswerthequestionsgiven
belowit:TherearesixpersonsA,B,C,D,EandFinaschool.Eachoftheteachersteachestwo subjects,
one compulsory subject and the other optional subject. D‟s optional subject was
Historywhilethreeothershaveitascompulsorysubject.EandFhavePhysicsasoneoftheir
subjects.F‟scompulsorysubjectisMathematicswhichisanoptionalsubjectofbothCandE. History
and English are A‟s subjects but in terms of compulsory and optional subjects, they
arejustreverseofthoseofD‟s.Chemistryinanoptionalsubjectofonlyoneofthem.Theonly
female teacher in the school has English as her compulsory subject.
Q63. What is C‟s compulsory subject?
(a) History (b)Physics
(c)Chemistry (d) English

Q64. Who is a female member in the group?


(a)A (b)B (c)C (d)D

Q65. Which of the following has same compulsory and optional subjects as those of F‟s?
(a)D (b)B (c)A (d)C

Q66. Disregarding which is the compulsory and which is the optional subject, who has the
Same two subject combination as F?
(a)A (b)B (c)E (d)D

Q67. Which of the following groups has History as the compulsory subject?
(a) A,C,D (b) B,C,D (c)C,D (d)A, B,C

Q68. Compare the knowledge of persons X, Y, Z, A, B and C in relation to each other:

GENERAL APTITUDE PAGE 230


1. X knows more thanA
2. Y knows as much asB
3. Z knows less thanC
4. A knows more thanY
The best knowledgeable person amongst all is:
(a)X (b)Y (c)A (d)C

Q69. Five children were administered psychological tests to know their intellectual levels.
In the report, psychologists pointed out that the child A is less intelligent than the
child B. The child C is less intelligent than the child D. The child B is less intelligent
than the child C and child A is more intelligent than the child E. Which child is the
most intelligent?
(a)A (b)B (c)D (d)E

Q70. If(i)PistallerthanQ;(ii)RisshorterthanP;(iii)SistallerthanTbutshorterthanQ, then who


among them is thetallest?
(a)P (b)Q (c)S (d)T

Q71. Five boys participated in a competition. Rohit was ranked lower than Sanjay. Vikas
wasrankedhigherthanDinesh.Kamal‟srankwasbetweenRohitandVikas.Whowas
rankedhighest?
(a)Sanjay (b)Vikas (c)Dinesh (d)Kamal

Q72. Ashish is heavier than Govind. Mohit is lighter than Jack. Pawan is heavier than Jack
but lighter than Govind. Who among them is the heaviest?
(a)Govind (b)Jack (c)Pawan (d)Ashish

Q73. Rohan is taller than Anand but shorter than Seema. Krishna is taller than Pushpa but
shorter than Anand. Dhiraj is taller than Krishna but shorter than Seema. Who among
them is the tallest?

GENERAL APTITUDE PAGE 231


(a) Rohan
(b) Seema
(c) Krishna
(d) Datainadequate
Directions (Q.75 – 77): Read the following information carefully and answer the questions
given below it:
(A) GopalisshorterthanAshokbuttallerthanKunal

(B) Navin is shorter thanKunal

(C) Jayesh is taller than Navin


(D)Ashok is taller thanJayesh
Q74. Who among them is the tallest?
(a)Gopal (b)Ashok (c)Kunal (d)Navin

Q75. Which of the given information is not necessary to answer the above question?
(a)A (b)B (c)C (d)D

Q76. BistwiceasoldasAbuttwiceyoungerthanF.CishalftheageofAbuttwicetheage
ofD.Whichtwopersonsformthepairofoldestandyoungest?
(a) FandA (b) FandD (c) BandF (d) F andC
Directions(Q.78–82):Readthefollowinginformationcarefullyandanswerthequestionsthatfollow:
P,Q,R,S,TandUaretravellinginabus.Therearetworeporters,twotechnicians,onephotographer
andonewriterinthegroup.ThephotographerPismarriedtoSwhoisareporter.Thewriterismarried
toQwhoisofthesameprofessionasthatofU.P,R,Q,Saretwomarriedcouplesandnobodyinthe
group has same profession. U is brother of R.
Q77. Which of the following is a pair of technicians?
(a)RS (b)SU (c)PT (d)QU

Q78. Which of the following is a pair of reporters?


(a)PQ (b)RT (c)ST (d)SU

Q79. How is R related to U?


(a) Brother

GENERAL APTITUDE PAGE 232


(b) Sister
(c) Uncle
(d) Datainadequate

Q80. Which of the following pairs is a couple?


(a)PQ (b)QR (c)QS (d)PT

Q81. Which of the following is a pair of husbands?


(a)PQ (b)PR (c)QS (d) Datainadequate

Q82. Four girls are sitting on a bench to be photographed. Shikha is to the left of Reena.
Manju is to the right of Reena. Rita is between Reena and Manju. Who would be
second from the left in the photograph?
(a)Reena (b)Shikha (c)Manju (d)Rita

Q83. Fivechildrenaresittinginarow.SissittingnexttoPbutnotT.KissittingnexttoR
whoissittingontheextremeleftandTisnotsittingnexttoK.Whoaresittingadjacent toS?
(a) KandP (b)RandP (c)OnlyP (d)P andT

Q84. Fivegirlsaresittinginarow.RashiisnotadjacenttoSulekhaorAbha.Anuradhais not


adjacent to Sulekha. Rashi is adjacent to Monika. Monika is at the middle in the
row.Then,Anuradhaisadjacenttowhomoutofthefollowing?
(a)Rashi (b)Sulekha (c)Abha (d)Monika

Q85. There are five friends. They are standing in a row facing South. Jayesh is to the
immediate right of Alok. Pramod is between Bhagatand Subodh. Subodh is
between Jayesh and Pramod. Who is in themiddle?
(a)Bhagat (b)Jayesh (c)Pramod (d)Subodh

Q86. FivepersonsA,B,C,DandEaresittinginarowfacingyousuchthatDisontheleft of C and B


is on the right of E. A is on the right of C and B is on the left of D. If E
occupiesacornerposition,thenwhoissittinginthecentre?
(a)A (b)B (c)C (d)D

GENERAL APTITUDE PAGE 233


Directions (Q.7– 9): Study given information carefully and answer the questions that follow:
(i) A,B,C,D,E,FandGaresittingonawallandallofthemarefacingeast.
(ii) C is on the immediate right ofD.
(iii) B is at an extreme end and has E as hisneighbour.
(iv)G is between E andF.
(v) D is sitting third from the south end.
Q87. Who is FOURTH to the right of „B‟?
(a) H
(b)D
(c)A
(d)None of these

Q88. In which of the following pairs, second person is to the immediate left of the first
person?
(a) „H, E‟
(b)„D, A‟
(c) „B,G‟
(d) „B,C‟
Q89. Which of the following is definitely true?
(a) „D‟issecondtotheleftof„H‟
(b) „A‟ is second to the right of„E‟
(c) „C‟ is to the immediate right of„B‟
(d) „E‟istotheimmediateleftof„C‟

Q90. Who is the immediate left of „E‟?


(a)„D‟ (b)„A‟ (c)„H‟ (d)„F‟

Q91. Who is third to the right of „G‟?


(a) „E‟ (b)„A‟ (c)„C‟ (d)„D‟

GENERAL APTITUDE PAGE 234


Directions(Q.93–97):Studyfollowinginformationcarefullyandanswerthequestionsbelow:
B, C, D, E, F, G, H and J are sitting around a circle facing at the centre. B is an immediate
neighbour of F and C. E is second to the left of F. G is second to the right of C. D is not an
immediate neighbour of G. There are two persons between C and J.
Q92. Who is to the immediate left of F?
(a)J (b)D (c)E (d)H

Q93. Who is to the immediate right of C?


(a) J
(b) D
(c) H
(d) Datainadequate

Q94. Who is sitting between D and J?


(a)E (b)G (c)Datainadequate (d)None ofthese

Q95. What is J‟s position with respect to B?


I. Fourth to theright
II. Fourth to theleft
(a)Ionly (b)IIonly (c) Either IorII (d)Both I andII
Q96. How many persons are between C and D?
(a)2only (b)4only (c)5only (d) Either 2 or4

Q97. Fiveboystookpartinarace.RajfinishedbeforeMohitbutbehindGaurav.Ashishfinished
beforeSanchitbutbehindMohit.Whowontherace?
(a)Raj (b)Gaurav (c)Mohit (d)Ashish

Directions(Q.99–104):Studyfollowinginformationcarefullyandanswerthequestions
given below. A, B, C, D, E, F, G and H are eight students of a school. They study in Std. VI,
VII and VIII with not more than three in any Std. Each of them has a favourite subject from
Physics, Geography, English, Marathi, Mathematics, Chemistry, Biology and Economics not
necessarily in the same order.D likes Chemistry and studies in Std. VIII with only H. B does
notstudyinStd.VII.EandAstudyinthesameStd.butnotwithB.CandFstudyinthesame
Std.ThosewhostudyinStd.VIdonotlikeMathematicsorBiology.FlikesPhysics.Theone

GENERAL APTITUDE PAGE 235


who studies in Std. VIII likes English. C does not like Geography. A‟s favourite subject is
Marathi and G does not like Biology.
Q98. Which subject does H like?
(a)English (b)Marathi
(c)Science (d)Datainadequate

Q99. What is G‟s favorite subject?


(a)Biology (b)Physics
(c)Marathi (d)None ofthese

Q100. What is C‟s favourite subject?


(a)Economics (b)Biology
(c)English (d)Geography

Q101. Whichofthefollowingcombinationsofstudent-Std-Subjectiscorrect?
(a)C-VII-Economics (b)D-VI-Chemistry
(c)G-VII-Physics (d) None iscorrect

Q102. Which of the following group of students study in VIIStd?


(a)EAF (b)EGC
(c)EAG (d) Datainadequate

Syllogism
Directions (Q.104 – 105): In each of the following questions, select one alternative in which
the third statement is implied by the first two statements.
Q103. (a) Allelephantsarewild.Alllionsarewild.So,alllionsareelephants.
(b) Allmangoesarered.Someapplesaremangoes.So,allapplesarered.
(c) All roads are boxes. All foxes are roads. So, all boxes arefoxes.
(d) All XYZ can run. All ABC are XYZ. So, all ABC canrun.

Q104. (a) Alldogsaremad.Allsickpersonsaremad.So,allsickpersonsaredogs.


(b) Allorangesareblack.Allfigsareoranges.So,allfigsareblack.

GENERAL APTITUDE PAGE 236


(c) All windows are dogs. Some doors are dogs. So, all windows are doors.
(d) No man can fly. No kite can fly. So, all men are kites
Directions (Q.106 – 108): Each of these questions consists of two statements P and Q. They
May look factually absurd. You have to ignore this absurdity and concentrate only upon the
Logic involved therein. Mark your answer as
(a) IfbothPandQaretrue.
(b) If both P and Q are false ordoubtful
(c) If P is true and Q is false ordoubtful
(d) If P is false or doubtful and Q istrue
Q105. P: Jackalsliveinforests.Citiesareinforests.Thereforejackalsliveincities.
Q: Sometablesaregrass.Allstoolsaregrass.Thereforealltablesarestools

Q106. P: Cups play chess. Chess is a difficult game. Therefore cups play a difficult game.
Q: Ritu is a girl. All girls are timid. ThereforeRitu is timid.

Q107. P: Some musicians are not rich. All musicians are polite. Therefore not all polite
persons are rich.
Q: All musicians are rich. No rich person is polite. Therefore musicians are not
polite.

Directions (Q.109 – 111): In each of these questions, two statements P and Q are given. You
May look into their logical acceptability and mark your answer as
(a) if both P and Q aretrue
(b) If P is correct and Q is wrong ordoubtful
(c) If P is wrong or doubtful and Q iscorrect
(d) IfbothPandQarewrongordoubtful
Q108. P: All A‟s are B‟s. All B‟s are C‟s. Therefore all C‟s are A‟s.
Q: Some A‟s are B‟s. All B‟s are C‟s. Therefore some A‟s are C‟s

Q109. P: Some A‟s and some B‟s are C‟s. Some C‟s are both A‟s and B‟s. Therefore some

GENERAL APTITUDE PAGE 237


A‟s are B‟s.
Q: All A‟s are B‟s. Some C‟s are A‟s. Therefore, some C‟s areB‟s.

Q110. P: AllA‟sareB‟s.SomeB‟sareC‟s.ThereforesomeA‟sareC‟s.
Q: SomeA‟sareB‟s.AllC‟sareA‟s.ThereforesomeC‟sareB‟s.

Directions(Q.112–115):Eachquestiongivenbelowhasasetofthreeorfourstatements.Each
setofstatementsisfurtherdividedintothreesegments.Choosethealternativewherethethird
segment in the statement can be logically deduced using both the preceding two, but not just
from one of them.
Q111. A: X is an actor. Some actors are pretty. X is pretty.
B: Some men are cops. All cops are brave. Some brave people are cops.
C: All actors are brave. Some men are actors. Some men are brave.
D: All actors are pretty. X is not an actor. X is not pretty.
(a) Aonly (b)BandC (c)Conly (d) Donly

Q112. A: All beautiful things are sad. She is beautiful. She is sad.
B: All nice things are flat. TVs are flat. TVs are nice things.
C: Potatoes are stems. All stems are fruits. Potatoes are fruits.
(a) Aonly (b)AandB (c)Conly (d) A andC

Q113. A: All mammals are viviparous. Some fish are viviparous. Some fish aremammals.
B: Allbirdsareoviparous.Somefisharenotoviparous.Somefisharebirds.
C: No mammal is oviparous. Some creatures are oviparous. Some creatures are not
mammals.
D: Somecreaturesaremammals.Somecreaturesareviviparous.Somemammalsare
viviparous.
(a)Aonly (b)Bonly (c)Conly (d) Donly

Q114. A: All good people are knights. All warriors are good people. All knights are
warriors.

GENERAL APTITUDE PAGE 238


B: Nofootballersareministers.Allfootballersaretough.Someministersareplayers. C:
Allpizzasaresnacks.Somemealsarepizzas.Somemealsaresnacks.
D: Some barkers are musk deer. All barkers are sloth bears. Some sloth bears are
muskdeer.
(a)Aonly (b)BandC (c)Conly (d) C andD

Directions(Q.116–121):Eachquestiongivenbelowconsistsoffiveorsixstatementsfollowed
By Options consisting of three statements put together in a specific order. Choose the option
which indicates a valid argument containing logically related statements that is, where the
Third Statement is a conclusion drawn from the preceding two statements.
Q115. A: All synopses are poets. B: Some synopses are mentors.
C: Some X are notmentors. D: All X are poets.
E: All synopses are mentors. F: All synopses are X.
(a)ACB (b)AEC (c) FEC (d)DFA

Q116. A: All synopses are poets. B: Some synopses are mentors.


C: Some X are notmentors. D: All X are poets.
E: All synopses arementors. F: All synopses are X.
(a) ACB (b)AEC (c) FEC (d)DFA

Q117. A: All cushions are pillows. B: Some pillows are sheets.


C: Some sheets arecushions. D: Some pillows are not cushions.
E: All pillows are sheets. F: No pillows are sheets.
(a)ABC (b)BCD (c) EAC (d)EDC

Q118. A: No window is a door. B: All poles are doors.


C: No pole is awindow. D: Some poles are not windows.
E: Some windows are poles. F: Some doors are not windows.
(a)ABC (b)ACB (c) BDA (d)DFA

GENERAL APTITUDE PAGE 239


Q119. A: Apples are not sweets. B: Some apples are sweet.
C: All sweets aretasty. D: Some apples are not tasty.
E: No apple is tasty.
(a)EAC (b)CEA (c)BDC (d)CBD

Q120. A: Apples are not sweets. B: Some apples are sweet.


C: All sweets aretasty. D: Some apples are not tasty.
E: No apple is tasty.
(a)EAC (b)CEA (c)BDC (d)CBD

Directions (Q.122 – 123): Each of these questions is based on a set of given propositions
I to IV.
Selectthebestanswertoeachofthesequestions. I:
All P areQ.
II: Some P are notQ.
III: Some, but not all, P are Q.
IV: No P isQ
Q121. Considered only by themselves, which of the following pairs of statements might
either both be true or both be false?
(a)IandII (b)IIandIII (c)I and III (d)I andIV
Q122. Considered only by themselves, all of the following pairs of statements might both be
false but could not both be true except
(a) IandII (b)IandIII (c)Iand IV (d)II andI

Series Completion
Directions: find the missing term in each of the following series:
Q123. 1,6,15,?,45,66,91
(a)25 (b)26 (c)27 (d)28

Q124. 2, 5, 9, 19, 37, ?


(a)73 (b)75 (c)76 (d)78

Q125. 4, 8, 28, 80, 244, ?


(a) 278 (b) 428 (c) 628 (d) 728

GENERAL APTITUDE PAGE 240


Q126. 0, 6, 24, 60, 120, 210, ?
(a)240 (b)290 (c)336 (d)504

Q127. 1, 3, 3, 6, 7, 9, ?, 12, 21
(a)10 (b)11 (c)12 (d)13

357
Q128. Whichfractioncomesnextinthesequence1, , , ,?
2 4 8 16
9 10 11 12
(a) (b) (c) (d)
32 17 34 35

Q129. 2, 3, 8, 27, 112, ?


(a)226 (b)339 (c)452 (d)565

Q130. 2, 2, 5, 13, 28,?


(a)49 (b)50 (c)51 (d)52

Q131. 2, 7, 27, 107, 427,?


(a)1262 (b)1707 (c)4027 (d)4207

Q132. 24, 60, 120, 210,?


(a)300 (b)336 (c)420 (d)625

Q133. Which term comes next in the sequence: AC, FH, KM, PR,?
(a)UW (b)VW (c)UX (d)TV

Q134. Find the next term in the series: BMO, EOQ, HQS,?
(a)KSU (b)LMN (c)SOV (d)SOW

Q135. Which term comes next in the series: YEB, WFD, UHG, SKI,?
(a)QOL (b)QGL (c)TOL (d)QNL

Q136. Whichtermwillreplacethequestionmarkintheseries:ABD,DGK,HMS,MTB, SBL,?


(a)ZKU (b)ZKW (c)ZAB (d)XKW

GENERAL APTITUDE PAGE 241


Q137. Find the next term in the alpha-numeric series :
Z1A, X2D, V6G, T21J, R88M, P445P,?
(a)N2676S (b)N2676T (c)T2670N (d)T2676N

Q138. Find the term which does not fit into the series given below:
G4T, J10R , M20P, P43N, S90L,
(a)G4T (b)J10R (c)M20P (d)P43

CODING- DECODING
Directions (Q.140 – 151): In each of the following questions below, find out the correct
Answer from the given alternatives.
Q139. In a certain code, GIGANTIC is written as GIGTANCI. How is MIRACLES written in
that code?
(a)MIRLCAES (b)MIRLACSE (c)RIMCALSE (d)RIMLCAES

GENERAL APTITUDE PAGE 242


Q140. Inacertaincode,GOODNESSiscodedasHNPCODTR.HowisGREATNESScoded in
thatcode?
(a)HQFZUODTR (b)HQFZUMFRT (c)HQFZSMFRT (d)FSDBSODTR

Q141. In a certain code, EXPLAINING is written as PXEALNIGNI. How is PRODUCED


written in that code?
(a)ORPBUDEC (b)ROPUDECD (c)ORPUDECD (d)None ofthese

Q142. Inacertaincode,RIPPLEiswrittenas613382andLIFEiswrittenas8192.Howis
PILLER written in thatcode?
(a)318826 (b)318286 (c)618826 (d)328816

Q143. If ROSE is coded as 6821, CHAIR is coded as 73456 and PREACH is coded as 961473,
what will be the code for SEARCH?
(a)246173 (b)214673 (c)214763 (d)216473

Q144. In a certain code, PRODUCTS is written as NPMBSARQ. How is COMPREHENSION


written in that code?
(a)AMKNPCFCLOMLQ
(b) AMKNPCFCLQGML
(c)AMKNPCFCLQGNL
(d) AMKNPCFCKOML

Q145. In a certain code language, QUEUE is written as Q22, and CHURCH is written as
1UR1. Which of the following would be most appropriate code for BANANA in that
language?
(a)B5A5 (b)5N5A (c) B55A (d)BA5A5A

Q146. IfPAINTERiswritteninacodelanguageas NCGPRGP, then REASON wouldbe


written as

GENERAL APTITUDE PAGE 243


(a)PCYQMN (b)PGYQMN (c)PGYUMP (d)PGYUPM

Q147. If CARPET is coded as TCEAPR, then the code for NATIONAL would be
(a)NLATNOIA (b)LANOITAN (c)LNAANTOI (d)LNOINTAA

Q148. If FESTIVAL is coded as MBWJUTFG, then OPIUM would be coded as


(a)NOHTL (b)NTHNO (c)NVJQP (d)MUIPO

Q149. In a code language if POSE is coded as OQNPRTDF, then the word TYPE will be
coded as
(a)SUXZOQFD (b)SUXZQOFD (c)SUXZOQDF (d)SUXZQODE

Q150. Inacertaincode,ORGANISMiswrittenasROAGINMS.HowisBOARDINGwritten in
thatcode?
(a)RAOBIDGN (b)OBRAGNID (c)OBRAIDGN (d)OBIDRAGN

Q151. Inacertaincode,BRIGHTENiswrittenasHJSCMDSG.HowisCOMPLETEwritten in
thatcode?
(a)DSDKQNPD (b)QNPDDSDK (c)QNPDFUFM (d)OLNBFUFM

Q152. InacertaincodeRAINiswrittenas8$%6andMOREiswrittenas7#8@.Howis
REMAIN written in thatcode?
(a)8@7$%6 (b)7@#$%6 (c)#@&$%6 (d)#@7$%

Directions(Q.154–155):Ineachofthefollowingquestionsfindthecorrectanswerfromthe
given alternatives.
Q153. Inacertainlanguage„prenotbis‟means„smokingisharmful‟,„vogdornot‟means
„avoidharmfulhabit‟,and„dorbisyel‟means„pleaseavoidsmoking‟,whichofthe
following means „habit‟ in thatlanguage?
(a)vog (b)not (c)dor (d)bis

Q154. In a certain code,

GENERAL APTITUDE PAGE 244


 „brightfreshsunnyday‟means„cinbinzinhin‟,
 „scent of fresh flower‟ means „din zin linbin‟,
 „brightlightoftrucks‟means„linminhinrin‟,
 „trucks loaded with flowers‟ means „fin nin dinmin‟.
Whatisthecodefor„bright‟inthatlanguage?
(a) cin (b)bin (c)hin (d)zin

Number, Ranking & Time sequence


Q155. Inarowoftrees,onetreeisfifthfromeitherendoftherow.Howmanytreesarethere in
therow?
(a)8 (b)9 (c)10 (d)11

Q156. In a queue, Amrita is 10thfrom the front while Mukul is 25thfrom behind and Mamta
is just in the middle of the two. If there be 50 persons in the queue, what position does

GENERAL APTITUDE PAGE 245


Mamta occupy from the front?
(a)20th (b)19th (c)18th (d)17th

Q157. Raman ranks sixteenth from the top and forty ninth from the bottom is a class. How
many students are there in the class?
(a)64 (b)65 (c)66 (d)DataInadequate

Q158. Sanjeevranksseventhfromthetopandtwentyeighthfromthebottominaclass.How
manystudentsarethereintheclass?
(a)37 (b)36 (c)35 (d)34

Q159. If Atul finds that he is twelfth from the right in a line of boys and fourth from theleft,
how many boys should be added to the line such that there are 28 boys in the line?
(a)12 (b)13 (c)14 (d)20

Q160. Manish ranked sixteenth from the top and twenty ninth from the bottom among those
who passed an examination. Six boys did not participate in the competition and five
failed in it. How many boys were there in the class?
(a)40 (b)44 (c)50 (d)55

Q161. Some boys are sitting in a row. P is sitting fourteenth from the left and Q is seventh
fromtheright.IftherearefourboysbetweenPandQ,howmanyboysarethereinthe row?
(a)25 (b)23 (c)21 (d)19

Q162. Arunarankstwelfthinaclassofforty-sixfromtheleft.Whatwillbeherrankfromthe right?


(a)33 (b)34 (c)35 (d)37

Q163. Raviis7ranksaheadofSumitinaclassof39.IfSumit‟srankisseventeenthfromthe last, what


is Ravi‟s rank from thestart?
(a)14th (b)15th (c)16th (d)17th

GENERAL APTITUDE PAGE 246


Q164. Inaclassof60,wheregirlsaretwicethatofboys,Kamalrankedseventeenthfromthe
top.Ifthereare9girlsaheadofKamal,howmanyboysareafterhiminrank?
(a)3 (b)7 (c)12 (d)23

Q165. Inarowoftenboys,whenRohitwasshiftedbytwoplacestowardstheleft,hebecame
seventhfromtheleftend.Whatwashisearlierpositionfromtherightendoftherow? (a)First
(b)Second (c)Fourth (d)Sixth

Q166. In a queue, Vijay is fourteenth from the front and Jack is seventeenth from the end,
while Mary is in between Vijay and Jack. If Vijay be ahead of Jack and there be 48
persons in the queue, how many persons are there between Vijay and Mary?
(a)8 (b)7 (c)6 (d)5

Q167. In a row of girls, Shilpa is 8thfrom left and Reena is 17thfrom the right. If they
Interchange their positions, Shilpa becomes fourteenth from the left. How many girls
are there in the row?
(a) 25

(b) 27

(c) 29

(d) none ofthese

Q168. In a queue of children, Kashish is fifth from the left and Mona is sixth from the right.
When they interchange their places among themselves, Kashish becomes thirteenth
from the left. Then, what will be Mona‟s position from the right?
(a) 4th (b)8th (c)14th (d)15th
Q169. In a row of boys, Kapil is eighth from the right and Nikunj is twelfth from the left.
When Kapil and Nikunj interchange positions, Nikunj becomes twenty first from the
left. Which of the following will be Kapil‟s position from the right?
(a)8th (b)17th (c)21st (d)None ofthese

Q170. Sangeeta remembers that her father‟s birthday was certainly after eighth but before
thirteenth of December. Her sister Natasha remembers that their father‟s birthday was

GENERAL APTITUDE PAGE 247


definitely after ninth but before fourteenth of December. On which date of December
was their father‟s birthday?
(a)10th (b)11th (c)12th (d)None ofthese

Q171. Standing on a platform, Amit told Sunita that Aligarh was more than ten kilometres
but less than fifteen kilometres from there. Sunita knew that it was more than twelve
butlessthenfourteenkilometresfromthere.Ifbothofthemwerecorrect,whichofthe
followingcouldbethedistanceofAligarhfromtheplatform?
(a)11km (b)12km (c)13km (d)14km

Q172. ReachingtheplaceofmeetingonTuesday15minutesbefore08.30hours,Anujfound
himself half an hour earlier than the man who was 40 minutes late. What was the
scheduled time of themeeting?
(a)8.00hrs (b)8.05hrs (c)8.15hrs (d)8.45hrs

Q173. Thepriesttoldthedevotee,“Thetemplebellisrungatregularintervalsof45minutes. The


last bell was rung five minutes ago. The next bell is due to be rung at 7.45 a.m.”
Atwhattimedidthepriestgivethisinformationtothedevotee?
(a)7.40a.m. (b)7.05a.m. (c)7.00a.m. (d)6.55a.m.

Q174. ThetrainforLucknowleaveseverytwoandahalffromNewDelhiRailwayStation. An
announcement was made at the station that the train for Lucknow had left 40
minutes ago and the next train will leave at 18.00 hrs. At what time was the
announcementmade?
(a)15.30hrs (b)17.10hrs (c) 16.00 hrs (d)None ofthese

STATEMENT AND CONCLUSIONS(LOGICAL REASONING )


Directions(Q.1to13):Ineachofthefollowingquestions,astatement/groupofstatementsis
given followed by some conclusions. Without resolving anything yourself, choose the
conclusion which logically follows from the given statement(s).

Q175. Statement: Most dresses in that shop are expensive.

GENERAL APTITUDE PAGE 248


Conclusions:
(a) There are no cheap dresses available in thatshop.
(b) Handloomdressesinthatshoparecheap.
(c) There are cheap dresses also in thatshop.
(d) Some dresses in that shop areexpensive.

Q176. Statement: Every library has books.


Conclusions:
(a) Books are only inlibrary.
(b) Libraries are meant for booksonly.
(c) No library is withoutbooks.
(d) Some libraries do not havereaders.

Q177. Statement: The government is soon going to introduce a bill which would permit the
instituting of private universities under very strict direction.
Conclusions:
(a) We have some private universities in our country evennow.
(b) The demand for more universities is being steppedup.
(c) Such directions can also be issued without informing theParliament.
(d) The government gives directions to establish anything in privatesector.
(e) Unless and until the directions are given, the private universities can charge
exorbitantfees.

Q178. Statement: To pass the examination, one must work hard.


Conclusions:
(a) Examination is related with hardwork.
(b) All those who work hard,pass.
(c) Examination causes some anxiety and those who work hard overcomeit.
(d) Withouthardwork,onedoesnotpass.

GENERAL APTITUDE PAGE 249


(e) Hard-working person is a satisfiedperson.

Q179. Statement: In the university examination, most of the candidates write in Hindi
medium.
Conclusions:
(a) Some candidates of this examination write inHindi.
(b) MostlycandidateswithHindimediumappearinthisexamination.
(c) In this examination no candidate writes answers in medium other thanHindi.
(d) All the candidates who appear in this examination write answers in Hindi.

Q180. Statement: This book can help because all good books help.
Conclusions:
(a)This is not a good book.
(b)This is a good book.
(c)No good book helps.
(d)Some good books help.

Q181. Statement: All that glitters is not gold.


Conclusions:
(a)Non-metals also glitter.
(b)Only gold glitters.
(c)Not all metals glitter.
(d)Glittering things may be deceptive.

Q182. Statement: A forest has as many sandal trees as it has Ashoka trees. Three-fourth of
the trees are old ones and half of the trees are at the flowering stage.
Conclusions:
(a) All Ashoka trees are at the floweringstage.
(b) All sandal trees are at the floweringstage.
(c) Atleastone-halfoftheAshokatreesareold.

GENERAL APTITUDE PAGE 250


(d) One-halfofthesandaltreesareatthefloweringstage.
(e) None ofthese
Q183. Statement: Many business offices are located in buildings having two to eight floors.
If a building has more than three floors, it has lift.
Conclusions:
(a) All floors may be reached bylifts.
(b) Only floors above the third floor havelifts.
(c) Seventh floors havelifts.
(d) Second floors do not havelifts.

Q184. Statement:Inthiscompany,60%oftheemployeesaremales,40%arefemales,80%of
the employees are sincere and 40% of the employees are from this city – Rawalpura.
Conclusions:
(a) All male employees are from outstation.
(b) All male employees aresincere.
(c) 20% of female employees are notsincere.
(d) All female employees are fromRawalpura.
(e) None ofthese
Q185. Statement: In a class, three-fourth of the boys play football, one-half play cricket, one
fourth of those who play cricket do not play football.
Conclusions:
(a) Two-third of the boys play onlyfootball.
(b) One-fourth of the boys play neither cricket norfootball.
(c) One-thirdoftheboysplayneithercricketnorfootball.
(d) One-eighthoftheboysplayneithercricketnorfootball.
(e) Two-fifth of the boys play onlyfootball.
Directions(Q.188–190):Inthefollowingquestionstherearegivensomestatementsfollowed
by conclusions that can be drawn from them. Choose the conclusion which appeals to you to
be the most correct.
Q186. The Taj is in Agra. Agra is in India. Therefore, the Taj is in India.
(a)True (b)Probablyfalse (c)False (d) Can‟tsay

GENERAL APTITUDE PAGE 251


Q187. Thepresenceofcalciuminmilkmakesitwhite.Rice,too,iswhite.Therefore,ricealso
containscalcium.
(a)False (b)Probablytrue (c)True (d) Can‟tsay

Q188. Hydrogen is lighter than oxygen. Hydrogen is lighter than helium. Therefore, oxygen
is the heaviest of the three gases.
(a) False (b)Probablyfalse (c)True (d) Can‟tsay

Q189. People who are bald are generally of the intellectual type. Arun is bald. Therefore
Arun is an intellectual.
(a)False (b)Probablytrue (c)True (d)Can‟tsay

Directions (Q.192– 196): In each of the following questions, certain statements are given
followed by a conclusion based upon them. Choose the alternative which best applies to the
given statements and conclusion.
Q190. Statements:
1. A triangle has threeangles.
2. A square has fourangles.
Conclusion: A polygon has many angles.
The conclusion drawn is
(a)definitely true
(b)definitely false
(c) either probably true or probablyfalse

(d) irrelevant

Q191. Statements:
1. Some persons are weak inMathematics.
2. Allthose,whoareweakinMathematics,aremusicians.
Conclusion: Some musicians are weak in Mathematics.
The conclusion drawn is
(a)definitelytrue (b)irrelevant

GENERAL APTITUDE PAGE 252


(c)probablytrue (d)definitelyfalse
Q192. Statements:
1. Someveryeffectivemedicinesaremadefromspidervenom.
2. Poisonofsnakeisalsoneedforcuringcertaindiseases.
Conclusion: All poisons cure some or the other disease.
The conclusion drawn is
(a) definitely follows from the givenstatements
(b) does not follow from the givenstatements
(c) is probablytrue
(d) Can‟tsay

Q193. Statements:
1. Waterboilsat100C.
2. Waterfreezesat0C.
Conclusion: At low pressure, water boils at lower temperatures.
The conclusion drawn is
(a) definitely true (b) definitelyfalse
(c) either probably true orprobablyfalse (d)irrelevant

Q194. Statements:
1. During volcanic eruptions, molten lava oozes out in astream.
2. Thelavacomesfromunderthecrustoftheearth.
Conclusion: The inside of the earth must be very hot.
The conclusion drawn is
(a) definitely true (b) probablytrue
(c) definitelyfalse (d)irrelevant

Q195. Statements:
1. Oxygen is agas.
2. This cylinder containsgas.
Conclusion: This cylinder contains oxygen.
The conclusion drawn is
(a)irrelevant (b)definitelytrue

GENERAL APTITUDE PAGE 253


(c)either probably true orprobablyfalse (d)definitelyfalse
Directions(Q.198–201):Ineachofthefollowingquestions,aconclusionisgivenfollowedby
two statements labeled A and B. Give your answer as:
(a) IfstatementAaloneisneededtodrawtheconclusion;
(b) IfstatementBaloneisneededtodrawtheconclusion;
(c) IfbothAandBareneededtodrawtheconclusion;
(d) IfbothAandBarenotsufficienttodrawtheconclusion.
Q196. He looks dangerous.
A. Hehasaleanandhungrylook.
B. He thinks toomuch.
Q197. Shaving is smoother if some soap is applied with warm water.
A. Brushingthesoaptomakelatherincreasesthetimeittakestoshave.
B. Thefacealsogetsawashasitisshaved.
Q198. Spices deteriorate after prolonged boiling.
A. The essential oils in spices easilyevaporate.
B. Spicesareputinpicklestoaddtotheflavor.
Q199. Sometimes a single observation makes us recall innumerable events which have
occurred in the past.
A. Man has the gift ofmemory.
B. Man is basically adreamer.
Q200. Plants can also be sick.
A. Only mammals can beill.
B. Improper nutrition will create aberrations in a plant‟sgrowth.

(THEME DETECTION)
Directions:Eachofthefollowingquestionscontainsasmallparagraphfollowedbyaquestion
on it. Read each paragraph carefully and answer the question given below it:
Q201. The virtue of art does not allow the work to be interfered with or immediately ruled
by anything other than itself. It insists that it alone shall touch the work in order to

GENERAL APTITUDE PAGE 254


bring it into being. Art requires that nothing shall attain the work except through art
itself.
This passage best supports the statement that:
(a) art is governed by external rules andconditions.
(b) artisforthesakeofartandlife.
(c) art is for the sake of artalone.
(d) artist realizes his dreams through his artisticcreation.
(e) artistshouldusehisartforthesakeofsociety.

Q202. Though the waste of time or the expenditure on fashions is very large, yet fashions
have come to stay. They will not go, come what may. However, what is not required
is that strong efforts should be made to displace the excessive craze for fashion from
the minds of these youngsters.
The passage best supports the statement that:
(a) fashion is the need of theday.
(b) theexcessivecrazeforfashionisdetrimentaltoone‟spersonality.
(c) the hoard for fashion should be done away with so as not to let down the
constructivedevelopment.
(d) workandotheractivitiesshouldbevaluedmorethantheoutwardappearance

Q203. Due to enormous profits involved in smuggling, hundreds of persons have been
attracted towards this anti-national activity. Some of them became millionaires
overnight. India has a vast coastline both on the Eastern and Western Coast. It has
been a heaven for smugglers who have been carrying on their activities with great
impunity. There is no doubt that from time to time certain seizures were made by the
enforcement authorities, during raids and ambush but even allowing these losses the
smugglers made huge profits.
The passage best supports the statement that:
(a) smuggling hampers the economic development of anation.
(b) smuggling ought to becurbed.

GENERAL APTITUDE PAGE 255


(c) authorities are taking strict measures to curbsmuggling.
(d) smuggling is fast increasing in our country owing to the quick profit itentails.
Q204. The only true education comes through the stimulation of the child‟s powers by the
demands of the social situations in which he finds himself. Through these demands
he is stimulated to act as a member of a unity, to emerge from his originalnarrowness
ofactionandfeeling,andtoconceivehimselffromthestandpointofthewelfareofthe group
to which hebelongs.
The passage best supports the statement that real education:
(a) willtakeplaceifthechildrenimbibeactionandfeeling.
(b) willtakeplaceifthechildrenarephysicallystrong.
(c) isnotprovidedinourschoolstoday.
(d) comes through the interaction with socialsituations.
(e) comes from the self-centred approach of thestudents.

Q205. Emerson said that the poet was landlord, sealord, airlord. The flight of imagination
made the poet master of land, sea and air. But a poet‟s dream of yesterday becomes
today an actual achievement and a reality for all men. Even those who invented,
improved and perfected the aeroplane could hardly have dreamt of the possibility of
flight into outer space.
The passage best supports the statement that:
(a) seemingly impossible imaginations make one a goodpoet.
(b) all imaginations become a reality someday.
(c) what man imagined has never been impossible; he has always turned it a reality
through his conception of ideas and sheer hardlabour.
(e)manhasreachedtheclimaxoftechnologicaldevelopmentwithhisexplorationinto
outer space.
Q206. The prevention of accident makes it necessary not only that safety devices be used to
guard exposed machinery but also that mechanics be instructed in safety rules which
they must follow for their own protection, and that lighting in the plant beadequate.
The passage best supports the statement that industrial accidents:
(a) are alwaysavoidable.
(b) may be due toignorance.

GENERAL APTITUDE PAGE 256


(c) cannot be entirelyovercome.
(d) can be eliminated with the help of safetyrules.
(e) usually result from inadequatemachinery.
Q207. It is upto our government and planners todevise ways and means for the mobilization
of about ten crore workers whose families total up about forty crore men, women and
children. Our agriculture is over manned. A lesser number of agriculturists would
mean more purchasing or spending power to every agriculturist. This will result in
theshortageofman-powerformanycommoditiestobeproducedforwhichtherewill be a
new demand from a prosperous agrarian class. This shortage will be removed by
surplus man-power released from agriculture as suggestedabove.
The passage best supports the statement that:
(a) employment in production is more fruitful than employment inagriculture.
(b) Indianeconomyisinapoorshapebasicallyduetoimpropermobilizationofman-
power.
(c) a shift of labour from agricultural sector to the industrial sector would uplift the
livingstandard.
(d) theindustrialsectorislabour-deficientwhiletheagriculturalsectorisover-
manned in our country.

Q208. To forgive an injury is often considered to be a sign of weakness; it is really a sign of


strength. It is easy to allow oneself to be carried away by resentment and hate into an
act of vengeance; but it takes a strong character to restrain those natural passions.The
manwhoforgivesaninjuryproveshimselftobethesuperiorofthemanwhowronged himself
and puts the wrong-doer toshame.
The passage best supports the statement that:
(a) the sufferer alone knows the intensity of hissufferings.
(b) peopletendtoforgivethethingshappenedinthepast.
(c) natural passions are difficult tosuppress.
(d) mercy is the noblest form ofrevenge.
(e) a person with calm and composed nature has depth of thought andvision.

GENERAL APTITUDE PAGE 257


Q209. Exports and imports, a swelling favourable balance of trade, investments and bank
balances, are not an index or a balance sheet of national prosperity. Till the beginning
of the Second World War, English exports were noticeably greater than what they are
today. And yet England has greater national prosperity today than it ever had.
Because the income of average Englishmen, working as field and factory labourers,
clerks, policemen, petty shopkeepers and shop assistants, domestic workers and other
low-paid workers, has gone up.
The passage best supports the statement that:
(a) a country‟s economic standard can be best adjudged by per capitaincome.
(b) a country‟s balance of trade is the main criteria of determining itseconomic
prosperity.
(c) a nation‟s economy strengthens with the increase inexports.
(d) EnglishtradehascontinuallyincreasedsincetheSecondWorldWar.

Q210. Throughout the ages the businessman has helped build civilisation‟s great cities,
providedpeoplewithluxuriesandartistswithpatronage,andlifthisfellowcitizens
tounderstandthestandardofliving.Inthelastfewcenturiesthebusinessmanhas
seededtheIndustrialRevolutionaroundtheworld.
The passage best supports the statement that the businessman:
(a) is accountable to thesociety.
(b) lives luxurious and comfortablelife.
(c) is the beneficiary of the industrialRevolution.
(d) is capable of raising his standard ofliving.
(e) has contributed to the growth ofcivilization.

Q211. One of the important humanitarian by products of technology is the greater dignity
and value that it imparts to human labour. In a highly industrialized society, there is
no essential difference between Brahmin and Dalit, Muslim and Hindu; they are
equally useful and hence equally valuable, for in the industrial society individual
productivity fixes the size of the pay cheque and this fixes social status.
The passage best supports the statement that:

GENERAL APTITUDE PAGE 258


(a) technology decides individual‟s socialstatus.
(b) castes and religions areman-made.
(c) human labour has dignity andvalue.
(d) all individuals, irrespective of caste and creed, are bornequal.
(e) industrial society is a great leveler ofmen.
Q212. The paragraph above also supports the statement that:
(a) there are individual differences in industrialproductivity
(b) there are numerous side-effects oftechnology.
(c) sizeofthepayandsocialstatuscontradicteachother.
(d) labour force is treated well in modern dayindustries.
(e) division of labour based on caste wasjustified.

Q213. The press should not be afraid of upholding and supporting a just and righteous cause.
It should not be afraid of criticizing the government in a healthy manner. The press
has to be eternally vigilant to protect the rights of the workers, backward and
suppressed sections of the society. It should also give a balanced view of the things so
that people can be helped in the formation of a healthy public opinion.
The passage best supports the statement that:
(a) presshasagreatroletoplayinademocracy.
(b) thepressistheonlymeanstoprojecttothemassesthepoliciesofthegovernment.
(c) the freedom of press is essential for the proper functioning ofdemocracy.
(d) thepresscanbeusedbythegovernmentsasaneffectivemediafortheupliftment of the
backward sections ofsociety.
(e) all the information given by the press should be well articulated so as to gain a
good opinion towards the rulingparty

Q214. There is a shift in our economy from a manufacturing to a service orientation. The
increase in service-sector will require the managers to work more with people rather
than with objects and things from the assembly line.
This passage best supports the statement that:
(a) Managers should have a balancedmind.
(b) Assembly line will exist in serviceorganizations.

GENERAL APTITUDE PAGE 259


(c) Interpersonal skills will become more important in the future workplace.
(d) Manufacturing organizations ignore importance ofpeople.
(e) Service organizations will not deal with objects andthings.
(READING COMPREHENSION)
Q215. After the attacks on the World Trade Centre in America, security at airports has
become a thorough nuisance and a waste of time and a lot of money all over the
world. So many ordinary objects have been banned, and so much time is spent
checking people and their baggage but air travel has not really become safer.
According to the author of this passage, if security is a waste of time and money,
which of the following studies will support his claim?
(a) Spending money on newer gadgets to detect explosives during checks should be
apriority
(b) Airport security staff has never really dealt with a security breach as serious as
9/11.
(c) We should spend money on intelligence, investigation and emergencyresponse.
(d) Airport staff at security checks are prepared for any hint of terrorist attack while
screening.

Q216. America is one country that has no shortage of guns. Almost anyone can lay his
hands on them. It isn't surprising then that the number of gun-related crimes is
always on the rise. Which is why law-abiding citizens must be allowed to keep a gun
on their person or property and therefore protect themselves? Which of the
following statements will weaken theargument?
(a) Using guns for protection is everyone's fundamentalright
(b) Policemen cannot be available whenever a crime isunderway
(c) 55%ofallgunsusedincrimesarestolenfromthehomesoflaw-abidingcitizens
(d) Guns are dangerous regardless of who hasthem
Q217. All over the world, there is a severe shortage of organs for those requiring a
transplant.Afamilymembercandonateanorganonlyifmedicaltestsshowthatthe
organwillbeacceptedbypatient'sbody.Insomecases,thefamilymemberdonates
anorgantosomeoneelse,whoarrangesfortherequiredorgantobedonated;this
can be a complicated chain of donors and organs. In some countries selling organs is

GENERAL APTITUDE PAGE 260


illegal. Considering the huge requirement for organs, and the long waitlist of those
who require it, organ donation should be made compulsory.
Which of the following arguments will weaken the case for making organ donation
compulsory?
(a)People who pledge to donate their organs are medically fit.
(b)Donated organs are a huge boost for medical research.
(c)Scientists have discovered a quick and easy way to create organs artificially.
(d)Both A and C

Q218. Since we pay for food and shelter, which are necessary for life, it is perfectly morally
acceptable to pay for water and allow private entities to market it.
Which of the following statements strengthens the argument that water usage should
be payable?
(a) Profit-oriented private sector will ensure regular watersupply.
(b) Water is a renewable resource and need not becommodified.
(c) Water is a basic human right and should not be paid for by the commonman.
(d) Private companies have always wanted to sellwater.

Q219. Banning books that can ruffle minorities is the right approach in a democratic
country like India. Which of the following is an assumption?
(a) Indiaisstillhometoahugesocialistmovement.
(b) India has experienced a violent partition on religiousgrounds
(c) In a democratic set-up, minorities' interests need to be safeguarded.
(d) India has 28 states and 9 unionterritories.

Q220. Topreventitsenvironmentalexploitation,theAntarcticTreatyof1959wassigned.It has


been agreed that Antarctica should be used exclusively for peaceful purposes and
that military activities would be prohibited. It also guaranteed continued freedom for
scientific research and promoted international scientificcooperation.
Which of the following statements will weaken the argument for preserving
Antarctica as a site for scientific research and cooperation?

GENERAL APTITUDE PAGE 261


(a) The scientists themselves are damaging toAntarctica.
(b) The climate and terrain of the Antarctica doesn't match any other place on earth.
(c)Scientists are unable to correlate results of experiments here with the rest of the
world.
(d) Not many people have visited the Antarctica.

Q221. The rise of inflammatory bowel diseases could be down to our shifting diets causing
a "boom in bad bacteria", according to researchers. The researchers said the high-fat
diet changed the way food was digested and encouraged harmful bacteria.
Microbiologists said modifying gut bacteria might treat the disease.
Which of the following statements best summarizes the proposed treatment for
IBDs?
(a) Reduction in consumption of monosaturated milk fats can ease thepain
(b) A balanced diet comprising fiber and proteins is the bestsolution.
(c) ReshapingthemicrobialbalanceofthegutcouldsolvetheIBDproblem.
(d) Strengtheningthebowelmusclesisthemosteffectivesolution.

Q222. Racist attacks are on the rise in most countries prompting Indian students to do a
rethink on their decision to study abroad. However, governments in the countries
concerned have promised to take up active campaigns to educate the public and
ensure that students are provided a safe and healthy atmosphere to live in. The fact
that hate crimes are on the rise, does not reflect positively on their efforts. The
argument "hate crimes are increasing despite efforts by the authorities" would
weaken if:
(a) Students have to study abroad due to lack of the right teaching facilities inIndia.
(b) The police have reported 30 hate crimes as compared to last year'sfigures.
(c)Racist crimes are now a punishable offence and can lead toimprisonment.
(d) 911 call centers report an 80% reduction in SOS calls received by them in this
category.

GENERAL APTITUDE PAGE 262


Q223. Delaying fatherhood may offer survival advantages, say Asian scientists who have
found children with older fathers and grandfathers appear to be "genetically
programmed" to live longer.
The argument that late fatherhood is beneficial for future generations can be
strengthened greatly by which of the following statements?
(a) Delayed fatherhood increases chances of miscarriages but also healthbenefits.
(b) Older fatherhood translates into stronger immune systems and better tissue
growth for thechild.
(c) Thehealthstatusoftheoffspringhasnotbeenanalysed.
(d) Scientistsaretryingtofigureouttheidealfatherhoodageforthechild.
Q224. In a recent study, scientists discovered that the new miracle cure for headaches,
Tagoprax, can be addictive. For those who are addicted to the pill to ward off
migraines, common side-effects include seeing colorful dancing spots, decrease in
colour sensitivity, etc. The scientists say that if one is having the pill more than once
a week, the dose of Tagoprax needs to be increased if the effect on the migraine has
to be constant. All this points to the fact that, Tagoprax, is a helpful little pill, but to
get hooked on to it, can lead to unpleasant vision-related problems.
Which of the following assumptions does the above argument make?
(a) Tagoprax is notdangerous.
(b) People who have migraines suffer from itregularly.
(c) Those who get hooked on to Tagoprax will find it hard to have only one pill a
week.
(d) AstudyhasfoundthatTagopraxcanbeaddictive.

Q225. In the early twentieth century, the biggest crisis in America was addiction to liquor.
It was leading to a rise in domestic violence, anger-fuelled crimes, and absenteeism
from work, and pushed society towards self-destruction. Prohibition in the 1930s just
made matters worse, by pushing the business underground, and into the hands of
those who were aggressively pushing for profits, used nefarious means to get their
way, and weren't averse to selling badly brewed liquor, for money. Today, societyis
hurtling towards self-destruction again, and this time the reason is marijuana

GENERAL APTITUDE PAGE 263


addiction. But enforcing a Prohibition on the drug could just make the problem
worse as it did in the 1930s.
Which of the following statements is the underlying assumption?
(a) ThebiggestcrisisinAmericawasaddictiontoliquor.
(b) Prohibitionmademattersworse,bypushingthebusinessunderground.
(c) Since prohibition against alcohol failed, prohibition against marijuana willfail
too.
(d) None ofthese

Q226. After the huge public rally in support of Hitesh Shah, a businessman and an
independent candidate, whose squeaky clean image is attracting the attention of
voters, the government is in a quandary. The problem was made worse by a media
blitz following an interview with Mr. Shah, who said, 'Before a thief starts stealing,
he gets elected'. This seems to have swung public opinion heavily in his favour, and
the election this time will be a very different affair. Which statement is central to the
argument that Mr. Shah is making?
(a) Politicians are no match for articulatebusinessmen.
(b) The media favours theunderdog.
(c) All politicians arecorrupt.
(d) Theelectionswillbeaffectedbytherecession.

Q227. Huge problems are heading our way. With global warming spiralling out of control
and extreme weather-related phenomena causing unheard of destruction regularly,
we are now at a stage where we have to live to eat. Food is going to become more
and more expensive; we will work in our offices way past our retirement age to be
able to afford all the basic meals.
Which of the following statements is not stated in the passage?
(a) Food is going to become more and moreexpensive
(b) Wewillworkinourofficeswaypastourretirementage.

GENERAL APTITUDE PAGE 264


(c) We are now at a stage where we have to live to eat.
(d)Huge problems are heading out of our way.

Q228. Team meetings convened to find solutions to problems are largely silent affairs. But,
brainstorming is the modern manager's answer to getting employees to open up.-
Gather the team in a room, place a whiteboard in front of them, and eventually, after
a series of free associations, the solution everyone is looking for will appear. The
rules are that nobody must scoff at even the most unlikely idea. Which of the
following statements is the underlying assumption of this argument?
(a) Team meetings areuseless
(b) Brainstorming sessions need to bechallenging
(c) If people are scared of saying the wrong thing, they'll hardly openup.
(d) CEOs prefer to think about solutionstogether

Q229. In the last five years, the number of students seeking admission to the government-
funded Godhpur University has been falling by 68%. During a recent budget review,
the state's home minister proposed shutting down the university and using the funds
for other purposes. He has recently shut down two other Universities based on the
minister's claims that the state's funds were being poured into a loss-making
institution. But, even a quick look at the University's distance education programme
shows that the university has about 2.3 lakh students enrolled for various online
courses, this number has been growing steadily for the last five years, filling the
University's coffers.
What can you conclude from the above passage?
(a) MinistersfeelthattheUniversityisadrainoffunds
(b) TherearemorestudentsinGodhpurthaninDelhi
(c) The University has more virtual students than regularones
(d) Offering distance education courses has been a monetary boon for theUniversity
Q230. A new television advert for Diet Bhujia featuring singer D has been cleared by the
Advertising Standards in an Indian state after they received 22complaints.

GENERAL APTITUDE PAGE 265


The advert depicts D cycling through a supermarket during a concert she was
performing in. Eighteen viewers complained because she was not wearing reflective
clothing and her bicycle had no lights, and that children could copy her actions.
ASupheldDietBhujia'sargumentthattheadwassupposedtoreflectD'sfantasy
andherescapefromthepressuresofbeingacelebrity.Whichofthefollowing
statementsistheunderlyingassumptionoftheAS'sjudgment?
(a) Children will not copy an ad based on afantasy.
(b) Disanextremelycarefulpersoninreallife.
(c) Ads do notexercise any influence on people.
(d) ThebrandteamofdietBhujiaunderstandsthepressurescelebritiesareunder.
Q231. An advert by fast food restaurant Burger King could be banned from British
television over claims that it is misleading. The campaign for the new TenderCrisp
chicken burger received two complaints over the fact that the burger appears
significantly larger on the advert than in real life. The Advertising Standards
Authority (ASA) investigated the complaints and purchased three burgers from a
Burger King restaurant. They reported that both the thickness and the height were
"considerably less" than in the advertisement.
Which of the following statements can strengthen Burger King's case?
(a) Advertisements are actually supposed to show the real size ofproducts.
(b) Normally, people do measure the product on the TVscreen.
(c) The advert needed to make all the ingredients in the product clearlyvisible.
(d) Both 1 and2

Q232. IntheUnitedStates,TheWaltDisneyCompanyhasannounceditsintentionsto prohibit


junk food adverts from being shown during programming on their
televisionandradiochannels,aswellasontheInternet.USFirstLadyMichelle Obama
described the move as ''a game changer for the health of our children",
adding:"Foryears,peopletoldusthatnomatterwhatwedidtogetourkidstoeat
well and exercise, we would never solve our childhood obesity crisis until companies

GENERAL APTITUDE PAGE 266


changed the way they sell food to our children. We all know the conventional
wisdom about that". Which of the following statements can be inferred from this
passage?
(a) Walt Disney really doesn't cares aboutchildren.
(b) Michelle Obama's fight against irresponsible advertising has finally bornefruit.
(c) Childhoodobesitywillsoonbeathingofthepast.
(d) TheWaltDisneyisanexceptiontoconventionalwisdom.

Q233. Dr. Mira Nair has collected data about student obesity. Her goal is to make students
aware of maintaining healthy eating habits before lifestyle diseases have a chance to
set in. Which of the following statements can be inferred from this passage?
(a) Dr Mira Nair is working towards the immediate well being of theadults.
(b) Dr Mira Nair is working towards the immediate well being of theaged.
(c) DrMiraNairisworkingtowardsthewellbeingofthestudents.
(d) DrMiraNairisworkingtowardsthewellbeingofherself.

Q234. President Vladimir Putin of Russia today signed a new law increasing the fines
against those involved in unlawful protests, overriding concerns from his human
rights advisor and the Council of Europe. The people of Russia believe this law is not
only draconian, but will set Russia apart from the rest of the world by making it look
like a nation still living in the middle ages. The new legislation increases maximum
fines for individuals involved in illegal protests from 100 roubles to 10,000 roubles.
Protestors are banned from concealing their faces and nobody with a criminal record
may organize a protest. Which of the following will weaken the protestor's stand of
'living in the middle ages'?
(a) The government decides to replace a fine with communitywork.
(b) The law matched similar legislation in other Europeannations.
(c) If the protestors manage to get the fine reduced by a thousandroubles.
(d) The Americans decide to campaign against thelaw.

GENERAL APTITUDE PAGE 267


Q235. Summer temperatures and a desire to save energy usually expended on air-
conditioning have left many South Koreans hot and uncomfortable this year - but
while attempts have been made to relax dress codes, many office workers find
conservative clothing habits die hard. The air-conditioning in all government offices
remainsresolutelyoffduringthehotsummersinabidtocombatelectricityshortages, which
last summer led to blackouts in major cities and red faces in the cabinet room. The
president himself has been setting an example by conducting cabinet meetings in his
shirt-sleeves -a shocking state of undress for a Korean president. Despite their
leader's example, senior officials in the president's office are still walking around in
suit jackets, even in the sauna-like conditions. One of the president's staff told me the
older men had complained that T-shirts and jeans left them nowhere to put their
wallet. The real reason may be more complex - a fear that dressing down in front of
their junior colleagues might diminish their authority. Which of the following
statements is snyc with the data provided in thepassage?
(a) South Koreans are a stickler for appearance more thanperformance.
(b) Dress codes determine how much respect a superior willget.
(c) TodresscasuallymeanstotakeworklightlyinSouthKorea.
(d) Without proper protocol, South Koreans cannot workefficiently.
Q236. Opponents of euthanasia erroneously argue that human death is always wrong;
exceptions exist. Which of the following can be concluded?
(a) Human death is not alwayswrong.
(b) Euthanasia is the act of putting to deathpainlessly.
(c) The body dies but the soul livesforever.
(d) Memoriesofapersonwhochooseseuthanasiaarepainful..
Q237. A team from Kyoto University, Japan, tested how macaque monkey calls affected the
feeding behaviour of the Sika deer that live on Yakushima Island.
Which of the following is a logical deduction?
(a) Interdependence was never proposed in Darwin's EvolutionTheory.
(b) Yakushimaishometomorethanonespeciesofanimals.

GENERAL APTITUDE PAGE 268


(c) Japanese scientists believe in fundamentalresearch.
(d) Kyoto university is financed by researchers themselves..

Q238. Teachers are now in touch with their students on Facebook. This now means social
sites will soon be accepted as a compulsory means of communication by all schools.
Soon all students and teachers should compulsorily have an account with Facebook.
Which of the following is analogous to the logic provided above?
(a) Olive oil has been proven to have huge benefits for people suffering from heart
problems. This would mean that soon olive oil would be an integral part of every
doctor's medication list for their heartpatients.
(b) Cyclinghasproventohaveapositiveeffectontheenvironment.Soallcyclists
willsoonhavetheirownfreewayswithplentyofgreencover.
(c) Swimming with dolphins helps children lose their fear of water faster. So all
aquariums will offer swimming classessoon.
(d) A house with pets is more friendly that one with none. So all housingsocieties
will have it as a norm to help foster better relations between inhabitants.
Q239. In a recent survey it was found that 80% of meat eaters lived longer as compared to
vegetarians. Hence it is safe to assume that meat provides us with nutrients that
cannot be replicated by any other food source.
Which of the following is not stated in the passage?
(a) 80% of meat eaters lived longer than non-meat eaters.
(b)A survey was conducted in the not so distant past.
(c) Nutrientsprovidedbyavegetariandietarenotprovidedbyameatdiet.

(d) None ofthese

LOGICAL DEDUCTION
Directions (Q.1 – 11): In each of these questions, there are two statements A and B followed
by two conclusion numbered I and II. Decide which of the two given conclusions logically
follows from the two given statements, disregarding commonly known facts. Mark your
answer as:

GENERAL APTITUDE PAGE 269


(a) If only conclusion Ifollows
(b) If only conclusion IIfollows
(c) If either I or IIfollows
(d) If neither I nor IIfollows
(e) If both I and IIfollows
Q240. Statements: I. All graduatesarechairs. II. All chairs aretables.
Conclusion: A:All graduatesaretables. B: All tables aregraduates.

Q241. Statements: I. No batisball. II. No ball iswicket.


Conclusion: A: No batiswicket. B: All wickets arebats.

Q242. Statements: I. All fisharetortoise. II. No tortoise is acrocodile.


Conclusion: A: No crocodile is a fish. B: No fish is acrocodile.

Q243. Statements: I. All playersarechair. II. Some chairs are Table.


Conclusions: I. All chairs are players. II. Some Table arechair.

Q244. Statements: I. All women are ministers. II. All ministers arefruit.
Conclusions: I. All womenarefruit. II. All ministers arewomen.

Q245. Statements: I. Alljailsareguesthouses.II.Allguesthousesarebench.


Conclusions: I. All jailsarebench. II. No jail isbench.

Q246. Statements: I. Some doctorsarehospital. II. Some hospital arerich.


Conclusion: I. Some doctorsarerich. II. Some rich are doctors.

Q247. Statements: I. All milkiswater. II. No water isliquid.


Conclusion: I. No milkisliquid. II. No liquid ismilk.

Q248. Statements: I. Some pens are tables. II. No table isblue.


Conclusions: I. No penisblue. II. No blue ispen.

GENERAL APTITUDE PAGE 270


Q249. Statements: I. Some trees are horses. II. Some ships aretrees.
Conclusions: I. Some horses are ships. II. Some trees are notships.

Q250. Statements: I. All poets are readers. II. No reader iswriter.


Conclusions: A: No poetiswriter. B: All readers arepoets.

Directions(Q.12–24):Ineachofthequestionsbelowaregiventhreestatementsfollowedby
someconclusions.Youhavetotakethegivenstatementstobetrueeveniftheyseemtobeat variance
with commonly known facts. Read all the conclusions and then decide which of the
givenconclusionslogicallyfollowsfromthegivenstatementsdisregardingcommonlyknown
facts.
Q251. Statements:
A. Some chairs areflowers.
B. All flowers aretrees.
C. Some trees areleaves.
Conclusions:
I. some trees are chairs.
II.Some leaves are flowers.
III. No chair is aleaf.
(a)Nonefollows
(b) Only Ifollow
(c) Only IIfollows
(d) Only I and IIIfollow
(e) Allfollow

Q252. Statements:
A. All buildings aremountains.
B. All glasses aremountains.
C. Some mountains arebook.
Conclusions:

GENERAL APTITUDE PAGE 271


I. Some book areglasses.
II. Some buildings arebook.
III. Some mountains areglasses.
(a) OnlyIfollow (b)OnlyIIfollows (c)Only IIIfollows
(d) Nonefollows (e)Allfollow

Q253. Statements:
A. All trains areroom.
B. No room is abus.
C. All boats arerooms.
Conclusions:
I. No boat isatrain. II. No bus is a boat. III. No train is aroom.
(a) Nonefollows (b) Only I and II follow(c)Only II and III follow
(d) Only I andIIIfollow (e)Only IIfollow

Q254. Statements:
A. Some rings arephones.
B. Some phones arecomputers.
C. Some computers arestations.
Conclusions:
I. Some stationsarerings. II. Some phones arestations.
III. Some computersarerings. IV.Allringsarestations.
(a) Nonefollows (b) Only I and IIfollow
(c)Only I, II andIIIfollow (d) Only II and III follow
(e)Allfollows

Q255. Statements:
A. All needles arethreads.
B. All threads areboxes.
C. All treesboxes.
Conclusions:
I. No needleistree. II. Some trees arethreads.

GENERAL APTITUDE PAGE 272


III. Some boxesareneedles. IV.Sometreesareneedles.
(a) Only either I orIVfollow (b) Only either I or IV and IIfollow
(c) OnlyIIIfollows (d) Nonefollows
(e) Only either I or IV and IIIfollow

Q256. Statements:
A. All trees arehills.
B. Some hills areanimals.
C. All animals arebirds.
Conclusions:
I. Some birdsarehills. II. Some birds aretrees.
III. Some animalsaretrees. IV.Noanimalistree.
(a) Nonefollows (b) Only I and IIIfollow
(c) Only I andIVfollow (d) Only either III or IV and Ifollow
(e) None ofthese

Q257. Statements:
A. All lions arejungles.
B. Some jungles arerabbits.
C. All rabbits are elephants.
Conclusions:
I. Some rabbitsarelions. II. Some elephants arejungles.
III. Some elephantsarelions. IV.Someelephantsarerabbits.
(a) Only I andIIIfollow (b) Only I and IIfollow
(c) Only II andIIIfollow (d)OnlyIIIandIVfollow
(e)None ofthese

Q258. Statements:
A. All books arepens.
B. No pens arehouses.
C. All houses aredoors.
Conclusions:

GENERAL APTITUDE PAGE 273


I. No booksarehouses. II. No books aredoors.
III. Some doorsarepens. IV.Somehousesarebooks.
(a) OnlyIfollows (b) Only I and IIfollow
(c) Only II andIIIfollow (d) Only III and IVfollow
(e) None ofthese

Q259. Statements:
A. Some dogs arerats.
B. All rats aretrees.
C. Some trees are notdogs.
Conclusions:
I. Some treesaredogs. II. All dogs aretrees.
III. All ratsaredogs. IV.Alltreesaredogs.
(a) Nonefollows (b) Only Ifollow
(c) Only I andIIfollow (d) Only II and IIIfollow
(e) Allfollows

Q260. Statements:
A. Some boys arerains.
B. All rains areclouds.
C. Some clouds arecars.
Conclusions:
I. Some cloudsareboys. II. Some cars areboys.
III. Some carsarerains. IV.Somerainsareboys.
(a) Nonefollows (b) Only IVfollows
(c)OnlyIfollow (d) Both I and IVfollow

Q261. Statements:
A. All bricks areflowers.
B. Some houses areflowers.
C. All pens arehouses.
Conclusions:
I. Some housesarebricks. II. Some pens areflowers.

GENERAL APTITUDE PAGE 274


III. Some flowersarebricks. IV.No pen is flowers.
(a) Only either II or IV andIIIfollow (b) Only either II or IV and Ifollow
(c) OnlyeitherIorIIandIVfollow (d) Nonefollows
(e) Allfollows
Q262. Statements:
A. All lions areducks.
B. No duck is ahorse.
C. All houses arefruits.
Conclusions:
I. No lion isahorse. II. Some fruits arehorses.
III. Some ducksarelions. IV.Somelionsarehorses.
(a) Allfollows
(b) OnlyeitherIorIIandbothIIIandIVfollow
(c) OnlyeitherIorIVandbothIIandIIIfollow
(d) OnlyeitherIorIVandIIfollow
(e) None ofthese
Q263. Statements:
A. Some mountains arerivers.
B. Some rivers areroads.
C. Some roads arewindows.
Conclusions:
I. Some windows are roads. II. Some rivers aremountains.
III. Some roadsare mountains. IV.Some windows are rivers.
(a) Allfollows (b) Only I and IIfollow
(c) Only III andIVfollow (d) Only I and IVfollow
(e) Nonefollow

CLASSIFICATION
Directions In each of the following questions, five words are given, out of which four are
same in one way or the other and the fifth one is different from the these four. Select the odd
one.
Q264. (a)Sun (b)Moon (c)Venus (d)Mars

GENERAL APTITUDE PAGE 275


Q265. (a)Diamond (b)Ruby (c)Emerald (d)Turquoise

Q266. (a)Green (b)Violet (c)Brown (d)Yellow

Q267. (a)Writer (b)Printer (c)Publisher (d)Reader

Q268. (a)Under (b)Near (c)Beside (d)Where

Q269. (a)Fragrance (b)Smell (c)Aroma (d)Incense

Q270. (a)Ink (b)Pen (c)Pencil (d)Brush

Q271. (a)Guitar (b)Piano (c)Harmonium (d)Banjo

Q272. (a)Freeze (b)Simmer (c)Bake (d)Boil

Q273. (a)Mother (b)Sister (c)Brother (d)Aunt

Q274. (a)Lemon (b)Orange (c)Citron (d)Banana

Q275. (a)Manganese (b)Rubber (c)Salt (d)Gold

Q276. (a)Rectangle (b)Rhombus (c)Square (d)Circle

Q277. (a)Aluminum (b)Copper (c)Brass (d)Brick

Q278. (a)Metre (b)Yard (c)Litre (d)Inch

Q279. (a)Mars (b)Jupiter (c)Saturn (d)Sun

GENERAL APTITUDE PAGE 276


Q280. (a)Onion (b)Tomato (c)Potato (d)Garlic

Q281. (a)Sparrow (b)Parrot (c)Cuckoo (d)Duck

Q282. (a)Sparrow (b)Parrot (c)Cuckoo (d)Duck

Q283. (a)Sparrow (b)Parrot (c)Cuckoo (d)Duck

Q284. (a)Gold (b)Silver (c)Bronze (d)Iron

Q285. (a)Gold (b)Silver (c)Bronze (d)Iron

Q286. (a)Milk (b)Orange (c)Cotton (d)Snow

Q287. (a)Yen (b)Lira (c)Dollar (d)Ounce

Q288. (a)Huge (b)Tiny (c)Heavy (d)Small

Q289. (a)Rigveda (b)Yajurveda (c)Atharveda (d)Ayurveda

Q290. (a)Violin (b)Guitar (c)Sitar (d)Piano

Q291. Three of the following words-form a group, while one does not belong to the group.
Find out that word.
(a)Intestines (b)Eyes (c)Hands (d)Ears

Q292. Four of the following five are alike in a certain way and so form a group. Which is the
one that does not belong to the group?
(a) Volume

(b) Size
(c) Large

GENERAL APTITUDE PAGE 277


(d) Shape
Q293. Four of the following five are alike in a certain way and so form a group. Which is the
One that does not belong to the group?
(a)Guava
(b)Orange
(c)Apple
(d)Lichy
Q294. Four of the following five pairs of alphabets and numerals have same relationship
betweentheirelementsasinthecaseofthepairPROBLEM:2948375andhenceform
agroup.Whichonedoesnotbelongtothegroup?
(a) BORE :8497
(b) MOEP :5972

(c) LBOR :3849

(d) OMEP :4572

Q295. Four of the following five pairs are alike in a certain way and hence form a group.
Which one does not belong to the group?
(a)DONE : NOED
(b)WANT : NATW
(c)WITH : TIHW
(d)HAVE : AVEH

Q296. (a)144
(b)169
(c)256
(d)288
ANALOGIES
Directions: A good way to figure out the relationship in a given question is to make up
asentence that describes the relationship between the first two words. Then, try to use
thesame sentence to find out which of the answer choices completes the same relationship
with
the third word
Q1. Odometer is to mileage as compass is to

GENERAL APTITUDE PAGE 278


(a) speed
(b) hiking
(c)needle
(d)direction

Q2. Marathon is to race as hibernation is to


(a)winter
(b)bear
(c)dream
(d)sleep

Q3. Window is to pane as book isto


(a)novel
(b)glass
(c)cover
(d)page

Q4. Cup is to coffee as bowl is to


(a) dish
(b)soup
(c) spoon
(d)food

Q5. Yard is to inch as quart is to


(a) gallon
(b)ounce
(c)milk
(d)liquid

GENERAL APTITUDE PAGE 279


GENERAL APTITUDE PAGE 280

You might also like

pFad - Phonifier reborn

Pfad - The Proxy pFad of © 2024 Garber Painting. All rights reserved.

Note: This service is not intended for secure transactions such as banking, social media, email, or purchasing. Use at your own risk. We assume no liability whatsoever for broken pages.


Alternative Proxies:

Alternative Proxy

pFad Proxy

pFad v3 Proxy

pFad v4 Proxy